FRCS (TR & Orth) 2014 PDF

October 15, 2022 | Author: Anonymous | Category: N/A
Share Embed Donate


Short Description

Download FRCS (TR & Orth) 2014 PDF...

Description

 

 

 

FRCS Tr Tr & Or Orth MCQs and Clinical Cases

 

FRCS Tr Tr & Or Orth MCQs and Clinical Cases

Vikas Khanduja MA MSc FRCS (Orth) Consultant Orthopaedic Surgeon Addenbrooke’s Hospital, Cambridge University Hospitals NHS Foundation Trust Cambridge Biomedical Campus, Cambridge, UK 

London • Philadelphia • Panama City • New Delhi

 

© 2014 The British Editorial Society of Bone & Joint Surgery. Published by JP Medical Ltd 83 Victoria Street, London SW1H 0HW, UK   Tel:  T el: +44 (0)20 3170 8910 Fax: +44 (0)20 3008 6180 Email: info@jpmedpub [email protected] .com Web: www.jpmedpub www.jpmedpub.com .com  The rights of Vikas Khanduja to be identied as the editor of this work have have been asserted by him in accordance with the Copyright, Designs and Patents Act 1988. All rights reserved. No part par t of this publication may be reproduced, stored or transmitted in any form or by any means, electronic, elec tronic, mechanical, photocopying, recording recording or otherwise, other wise, except as permitted by the UK Copyright, Designs and Patents Act 1988, without the prior permission in writing of the publishers. Permissions may be sought directly from JP Medical Ltd at the address printed above. All brand names and product names used in this book are trade names, service marks, trademarks or registered trademarks of their respective owners. The publisher is not associated with any product or vendor mentioned in this book. Medical knowledge and practice change constantly. This book is designed to provide accurate, authoritative information about the subject matter in question. However, readers are advised to check the most current information available on procedures included and check information from the manufacturer of each product to be administered, to verify the recommended dose, formula, method and duration of administration, adverse eects and contraindications. It is the responsibility of the practitioner to take all appropriate safety precautions. Neither the publisher nor the editor assume any liability for any injury and/or damage to persons or property arising from or related to use of material in this book.  This book is sold on the understanding understanding that the publisher publisher is not engaged in providing professional professional medical services. If such advice or services are required, the services of a competent medical professional should be sought. Every eort has been made where necessary to contact holders of copyright to obtain permission to reproduce copyright material. If any have been inadvertently inadver tently overlooked, the publisher will be pleased to make the necessary arrangements at the rst opportunity. oppor tunity. ISBN: 978-1-907816-93-2 British Library Cataloguing in Publication Data A catalogue record for this book is available from the British Library Library of Congress Cataloging in Publication Data A catalog record for this book is available from the Library of Congress

JP Medical Ltd is a subsidiary of Jaypee Brothers Medical Publishers (P) Ltd, New Delhi, India Publisher: Editorial Assistant: Design:

Richard Furn Sophie Woolven Designers Collective Ltd

Indexed, typeset, printed and bound in India.

 

Preface

 The FRCS (Tr & Orth) examination in the UK comprises a written test test composed of Multiple Choice Questions, a clinical component and four 30 minute orals. The clinical component is divided into cases that focus on the Upper and Lower limb. The The orals, or vivas, are divided into four sections comprising Adult pathology, Children’s Children’s orthopaedics or thopaedics and Hands, Trauma Trauma and Basic science. Most other countries, including Australia, follow a similar system of examination wherein can didates are assessed on their ability to examine a patient in a clinical setting and then interpret a clinical case and present a logical management plan in the viva setting.  The goal of this book is to focus specically on the needs of trainees preparing for for the exit exam in any part of the world and to cater to all the sections of the exam. It is laid out in a format which is helpful for revision. Chapter 1 includes the MCQs, written in the single best answer (SBA) format

-

used in the exam, and Chapter 2 provides the answers. The The following chapters cover Adult pathol ogy, Trauma, Trauma, Hands, Children’s Children’s orthopaedics or thopaedics and Basic science. For each of these themes there are separate chapters that include, rstly, clinical cases and, secondly, detailed model answers with references. A signicant amount of work by a number of busy clinicians and educationalists has gone into the preparation of this book and we have enjoyed the process; we hope you enjoy reading it as well.

Vikas Khanduja December 2013

v

 

Acknowledgements

 The editor would like to to express his sincere sincere thanks to Matt Henderson, Henderson, Jennifer Burns and Emma Emma Vodden from The Bone & Joint Journal  for  for their editorial assistance and continual support in completion of this project.

vi

 

Contents

     

Preface Acknowledgements Acknowledgemen ts Contributors Contribut ors

v vi viii

Chapter 1  1 

SBA Questions

Chapter 2  2 

SBA Answers

15

Chapter 3  3 

Viva Questions: Adult pathology

31

Chapter 4  4  Chapter 5  5 

Viva Answers: Adult pathology Viva Questio Questions: ns: Trauma

41 61

Chapter 6  6 

Viva Answers: Trauma

71

Chapter 7  7 

Viva Questions: Hands

89

Chapter 8  8 

Viva Answers: Hands

99

Chapter 9  9 

Viva Questions Questions:: Children’ Children’ss orthop orthopaedics aedics

1

117

Chapter 10  10  Viva Answers: Children’ Children’ss orthop orthopaedics aedics

133

Chapter 11  11  Viva Questions: Basic science Chapter 12  12  Viva Answers: Basic science

145 149

Index

169

vii

 

Contributors

Timothy N Board MD Board MD FRCS (Tr&Orth) Consultant Hip and Knee Surgeon Wrightington Hospital Wigan, UK  Henry Budd MBChB Budd MBChB (Hons) FRCS (Tr&Orth) ( Tr&Orth) Specialist Registrar Addenbrooke’s Hospital Cambridge, UK  Emmet Griffiths MBBS Griffiths MBBS BSc MRCS MSc Specialist Registrar Addenbrooke’s Hospital Cambridge, UK  Sim Johal BM (Hons) BM (Hons) BSc(Hons) MRCSEng Specialist Registrar Addenbrooke’s Hospital Cambridge, UK  Karan Johal BMedSci Johal BMedSci BMBS (Hons) MRCS MSc Specialist Registrar Northwest London Hospitals NHS Trust

Ajay Malvyia MS Malvyia MS MSc FRCS Ed (Orth) Consultant Orthopaedic Surgeon Wansbeck General Hospital Ashington, Northumberland, UK  Paul M Robinson MBChB Robinson MBChB (Hons) BMedSci  MRCS (Eng) Specialist Registrar Addenbrooke’s Hospital Cambridge, UK  Sumedh C Talwalkar MS(Orth) Talwalkar MS(Orth) MCh(Orth)  FRCS (Tr & Orth) Consultant Hand and Upper Limb Surgeon Wrightington Wigan and Leigh NHS Trust Senior Lecturer, Edge Hill University   Ormskirk, UK  Malin Wijeratna BSc Wijeratna BSc (Hons) MBBS FRCS (Tr&Orth) Shoulder & Elbow Fellow Melbourne Shoulder & Elbow Centre

London, UK 

Australia

David Jones FRCS FRCSEd (Orth) Consultant Adviser  The Bone & Joint Journal London, UK 

viii

 

Chapter 1 SBA Questions For each question, select the single best answer from the five options listed. 1. Which of the follow following ing investig investigation ationss is the the most most specific specific scanning scanning method to detect infection in a total joint replacement?  A. Bone scintigraphy  B.  A combination of white blood cell scan and a technetium bone scan C.  White blood cell scan scan D. Sequential gallium-67 citrate scans E. Sequential technetium bone scans 2. What propo proportion rtion of patient patientss will develo develop p significan significantt back or ipsilate ipsilateral ral knee knee pain 25 years after arthrodesis of their hip?

 A. 0% B. 10% C. 45% D. 60% E. 75% 3. With refer reference ence to to “pulled “pulled elbow elbow”” injury injury in child children, ren, which of the follow following  ing  statements is true?  A. Peak incidence is between 3 and 5 years of age B. It results from traction on the extended and supinated forearm C. Recurrence is seen in 5% of cases D. Pain is due to tearing of the lateral collateral ligament complex  E.  X-r  X-ray ay is essential for for diagnosis 4. During During fractur fracture e healing, healing, differ differenti entiation ation of the proge progenitor nitor cells depen depends ds on local oxygen tension and strain. Based on this theory, the following promotes formation of woven bone during fracture healing:  A. High strain and low oxygen tension B. Low strain and high oxygen tension C. Intermediate strain D. Low oxygen tension E. Intermediate strain and low oxygen tension

 

2

Chapter 1 SBA Questions

5. Which of the follow following ing statem statements ents rega regardin rding g the effects of glucocortico glucocorticoids ids on bone mineral metabolism is false?  A. Supraphysiologic Supraphysiologic glucocorticoid levels do not cause secondary  hyperparathyroidism B. Glucocorticoids inhibit gastrointestinal absorption of calcium C. Glucocorticoids preferentially preferentially affect trabecular bone and the cortical rim of the  vertebral bodies D. Glucocorticoids inhibit osteoclastic activity  E. Bisphosphonates Bisphosphonates are successful in the prevention of glucocorticoid-related osteoporosis 6. Te intera interaction ction betwee between n water water and and hyalin hyaline e cartilage cartilage is beneficial beneficial becau because: se:  A. Cartilage repels water B.  Water is absorbed by cartilage cartilage C.  Water lubricates the joint joint D.  Water is imbibed by cartilage cartilage and has a passive role in resisting compression compression

E.  Water prevents the the degradation of cartilage by matrix metalloproteinases metalloproteinases 7. Te loca locatio tion n of Ka Kapla plan’ n’ss cardi cardinal nal lin line e is:  A.  Along the ulnar border of the ring finger (parallel (parallel to the metacarpophalangea metacarpophalangeall  joints) B. From the first web space to the hook of the hamate C. From the hook of the hamate perpendicular to the proximal transverse palmar skin crease D. From the first carpometacarpal joint to the pisiform E.  Along the distal palmar palmar skin crease crease 8. What type of affer afferent ent peripher peripheral al nerve nerve fibre is responsib responsible le for tran transmit smitting  ting   vibration sense to the spinal cord?  A.  Aα (A alpha) B.  Aβ (A beta) C.  Aγ (A gamma) D.  Aδ (A delta) E. C 9. Which of the the followin following g investig investigation ationss best disti distinguis nguishes hes osteopor osteoporosis osis from from osteomalacia?  A. Bone biopsy from the iliac crest B. Serum calcium C. Serum phosphate D. Urinary calcium E. etracycline-label etracycline-labelled led bone biopsy 

 

SBA Questions

10. Te nerve supplyin supplying g to teres minor minor is a branch branch of which of of these nerves?  A. Suprascap Suprascapular ular nerve (C5, C6) B. Lower subscapular nerve (C5, C6) C. Upper subscapular nerve (C5, C6) D.  Axillary nerve (C5, C6) E. Medial supraclavicular supraclavicular nerve (C3, C4) 11. Which of the following clinical findings is least least likely to be associated with with a pre-ganglionic brachial plexus injury?  A. Bruising in the anterior triangle of the neck  B. Pain in an insensate hand C. Loss of sensation above the clavicle D. Ipsilateral Horner’s syndrome

 

3

E. Loss of muscle function of branches direct from the roots of the brachial plexus 12. Following a latissimus dorsi transfer for chronic chronic irreparable tears of the rotator rotator cuff, which of the following factors has NO been associated with a poor clinical outcome?  A. Male gender B. Poor pre-operative shoulder function C. Generalised muscle weakness D.  Absence of electrical electrical activity at follow-up follow-up E. Previous failed rotator cuff repair 13. Te alpha angle on the ultrasound of an infant’s infant’s hip is defined as:  A. Te angle between the acetabular roof and the midline of the pelvis B. Te acute angle between the lateral wall of the ilium and the bony acetabular roof  C. Te angle between the centre of the femoral head and the lateral wall of the ilium D. Te angle of the thigh required to produce subluxation of the hip on the sonogram E. Te angle between the acetabular roof and the transverse plane 14. Which of these values values reflects the normal tibiofemoral tibiofemoral axis for a child aged three  years?  A.  V  Varus arus of 20° B.  V  Varus arus of 10° C. Neutral alignment D.  V  Valgus algus of 10° E.  V  Valgus algus of > 20°

 

4

Chapter 1 SBA Questions

15. Which of the following constituent constituent accounts for 65–80% of the dry mass of flexor tendons?  A. Collagen type I B. Collagen type II C. Collagen type III D. Collagen type IV  E. Elastin 16. Which of the following statements is correct with regard regard to a therapeutic therapeutic intraarticular facet joint injection in the lumbar region for low back pain? A. Pain relief on two occasions after a facet joint injection is an indication for

 A. Pain relief on two occasions after a facet joint injection is an indication for facet joint ablation B. It aims to relieve back pain during flexion of the lumbar spine C. Steroid injection in the facet joint gives good pain relief, beyond six months D. Te best visualisation of the facet is on the lateral image intensifier view  E. It has a high risk of haematoma and infection 17. What percentage percentage of success success would you quote to patients being offered a coccygectomy for coccygodynia that has been refractory to conservative management?  A. < 20% B. 20% to 30% C. 40% to 50% D. 60% to 70% E. > 80% 18. Which one of the following statements is false with regard to to the clinical presentation of patients with tarsal tunnel syndrome?  A. Symptoms are variable B. Tere is sensory disturbance along the big toe C. Tere is atrophy of the intrinsic muscles of the foot D. Tere is hind-foot varus deformity  E. Symptoms accentuate accentuate on eversion and dorsiflexion of the foot

 

SBA Questions

19. Meta-analysis Meta-analysis comparing the the intermediate and long-term outcome after total ankle replacement and ankle arthrodesis has shown all of the following except:  A. Mean AOFAS AOFAS (American Orthopaedic Foot and Ankle Society) S ociety) Ankle–Hindfoot Scale score is higher for patients with wi th ankle arthrodesis B.  A greater number of patients patients have poor results after after ankle replacement replacement compared with ankle arthrodesis C. Te revision rates for both procedures are similar D. Five and ten year survival survi val after ankle replacement are more than 75%

 

5

E. Below knee amputation rate is higher for patients with ankle arthrodesis 20. With regar regard d to bearing bearing surfaces surfaces in artificial artificial joints: joints:  A. Equatorial bearing is ideal B. Polar bearing is more conducive to fluid film lubrication C. Subhemispherical sockets are always advantageous D. Sacrificial bearings will always fail before hard-on-hard bearings E. Te synovial fluid in prosthetic joints is indistinguishable from normal synovial fluid 21. Which of the following statements statements regarding regarding spinal tumours is false?  A.  About 15% of all all bone tumours are are primary spine tumours B. Combined surgical decompression and radiotherapy is generally superior to radiotherapy radiothera py alone in the treatment of metastatic spinal cord compression C. Spinal metastases from renal cell carcinoma tend to be hypervascular D. Spinal tumours often present with pain and weakness w eakness E. Ependymom Ependymoma a is the most common type of intramedullary tumour in adults 22. Which of the the following following is NO a cause cause of leg length length shortening? shortening?  A. Epiphysiodesis B. Coxa vara C. Fracture D. Klippel–rénaunay–Weber syndrome E. Poliomyelitis 23. In monostotic fibrous dysplasia, dysplasia, the prevalence of malignant malignant transformation transformation (chondrosarcoma or osteosarcoma) is about:  A. 0.4% B. 0.8% C. 1% D. 2% E. 4%

 

6

Chapter 1 SBA Questions

24. Which of the following statements statements is true true regarding regarding an open repair of the ruptured ulnar collateral ligament of the thumb?  A. It is unusual to see the superficial radial nerve B. One should aim to identify the extensor pollicis brevis tendon C.  A Stener lesion l esion will w ill be found, f ound, if i f present, proximal prox imal to the t he adductor adducto r

aponeurosis D. Te ligament will have ruptured from its insertion into the metacarpal in most cases E.  A Stener lesion will be found found in 50% of the cases 25. Which one one of the following following is necessary necessary for a good key grip? grip?  A. Extension of the thumb metacarpophalangeal metacarpophalangeal joint B. Extension of the thumb interphalangeal joint C. Function of the first interosseous muscles D. Function of the radial nerve E. Function of the median nerve 26. All of these are example exampless of enchondral enchondral bone formation formation except: except:  A. Embryonic long-bone formation B. Fracture callus C. Bone formation after use of demineralised bone matrix  D. During distraction osteogenesis E. Longitudinal growth 27. All of these are diseases that involve the proliferative proliferative zone of the growth growth plate except:  A. Kniest syndrome B.  Achondropl  Achondroplasia asia C. Gigantism D. Malnutrition E. Irradiation injury  28. Which one of these regulates cartilage and bone formation in the fracture callus?  A. Bone morphogenic protein B. ransforming growth factor-β C. Insulin-like growth factor II D. Platelet derived growth factor E. Endothelial derived growth factor

 

SBA Questions

29. What is the diagnosis in a patient who presents with reduced serum calcium, raised serum phosphate, normal alkaline phosphatase and parathormone level and a reduced urinary calcium excretion?

 

7

 A. Nutritional rickets B. Hypoparathyroidism C. Pseudohypoparathyroidism D. Secondary hyperpar hyperparathyroidism athyroidism E. Nutritional calcium deficiency  30. During revision knee arthroplasty arthroplasty with the trial components in place, the knee is tight in extension and loose in flexion. Correction involves which of the following changes?  A. Use of a thinner tibial insert B. Use of a thinner distal femoral augmentation wedge C. Use of a smaller femoral component D. Resection of more proximal tibia E. Shifting the femoral component anteriorly with an offset stem 31. A 26-year-old 26-year-old basketball basketball player was noted to have have an absent anterosuperior labrum during a shoulder MRI arthrogram. Which of the following would be true for a Buford complex?  A. Te patient needs further assessment with shoulder arthroscopy  B. Biceps insertion site will usually be involved C. Presence of a cord-like middle glenohumeral ligament actually represents represents the torn labrum D. Sites of insertion of anterior and middle glenohumeral ligament ligament are often abnormal E. Reattaching the complex will lead to painful restriction of rotation 32. Which one of the following exercise regime would would you recommend to someone  who wishes to improve his/her muscle bulk? bulk?  A. Isometric B. Isotonic C. Isokinetic D. Plyometric E.  Aerobic

 

8

Chapter 1 SBA Questions

33. With regards regards to the Bunnell-Littler Bunnell-Littler test, which of the following following statements statements is false:  A.  An increase in proximal proximal interphalangeal interphalangeal (PIP) joint flexion with flexion of  metacarpophalangeal metacarpoph alangeal (MCP) joint indicates i ndicates intrinsic tightness B. Reduced flexion of PIP joint in extended and flexed attitude of the MCP joint  would indicate a capsular contracture contracture of the joint C. Extension of PIP joint with attempted flexion of MCP joint may indicate lumbrical tightness D. Reduced PIP joint flexion with w ith flexion of MCP joint may indicate contracture of the extensor tendons E. Flexor digitorum profundus laceration distal to lumbrical origin may give a false positive test 34. A 13-year-old 13-year-old girl presents with a flexion deformity deformity of the little little finger PIP joint joint (camptodactyly). (campt odactyly). Which one of the following statements is true?  A. Te deformity is usually due to an abnormality in the lumbrical or flexor digitorum superficialis insertion B. Te deformity is commonly associated with Down’s syndrome C. Te deformity will usually respond to splinting and stretching D. Capsular release and tendon transfer is indicated if full PIP extension cannot be achieved with MCP held in flexion E. Corrective osteotomy is indicated 35. Which one one of the following following is true true about arthrogryp arthrogryposis? osis?  A. It results from defect in the motor unit uni t B. It is commonly myopathic in origin C. It leads to a contracture of the joint D. Joint contractures are progressive E. Te involvement of other organ systems is unusual 36. Which one of the following statements is false regarding regarding septic arthritis arthritis in children?  A. Males are affected twice as often as females B.  A lower extremity extremity (hip) is affected in 80% of patients patients C. Polyarticular involvement involvement is in i n fewer than 10% of patients D. Loss of proteoglycan starts at five days from the bacteria entering the joint E.  An aspirate white white blood cell count count of > 50 000/mm3 with 75% polymorphonuclearr leucocytes is diagnostic of sepsis in 60% to 70% of patients polymorphonuclea

 

SBA Questions

37. Which one of the following following statements statements is true regarding regarding the use of forest forest plots in systematic reviews? Tey:  A. Summarise treatment efficacies (risk ratio) across trials B. Detect publication bias in literature C.  Are a quantifiable quantifiable way to test prospective studies for homogeneity  D.  Are a measure measure of odds of failure against against the size of the study  E. Detect outliers in the literature in terms of study inclusion criteria 38. Which of the following following statem statements ents is false for the AAOS AAOS grades grades of  recommendation for summaries or reviews of orthopaedic studies?  A.  A high-quality prospective prospective prognostic study study investigating investigating the effect of a patient patient characteristic charact eristic on the outcome of disease would constitute a good evidence study (Grade A) B.  A systematic review of Level I studies studies investigating the the results of treatment with inconsistent results would constitute a fair evidence evi dence study (Grade B) C.  A prospective comparative comparative study study investigating investigating the results of a surgery with consistent findings would constitute a fair evidence study (Grade B) D.  A retrospective retrospective comparative comparative study investigating investigating the results results of an intervention  would constitute a fair fair evidence study (Grade (Grade B) E.  An expert opinion from someone who has performed performed more than than 1000 complex surgical intervention would constitute a poor evidence study (Grade C) 39. All of the following have have been shown to slow disease disease progress in osteoarthritis except:  A. Glucosamine sulphate B. Chondroitin sulphate C. Diacetylrhein D. Hyaluronic acid E. Doxycycline 40. Which of the following statements is true regarding regarding the use of tranexamic tranexamic acid in orthopaedic surgery?  A. It increases the prothrombin time B. Meta-analysis Meta-analysis has shown that it increases the risk of thromboembolic events C. It reduces the need for transfusion after joint replacemen replacementt D. It is only effective when given intravenously  E. It should only be used in high risk cases

 

9

 

10

Chapter 1 SBA Questions

41. Which of the following statements statements is true true when performing performing a unilateral unilateral wrist arthrodesis?  A. 15° ulnar deviation is preferred B. 10° to 20° of dorsiflexion is ideal to preserve grip strength C. Ulna–triquetral abutment cannot be avoided D.  An open epiphyseal plate in the distal distal radius is not a contraindicat contraindication ion E. Te most common surgical approach is volar 42. In the surgical treatment treatment of de Quervain’s Quervain’s stenosing tenosynovitis, tenosynovitis, which one of  the following tendons should be decompressed?  A.  Abductor pollicis longus longus B.  Adductor pollicis C. Extensor pollicis longus D. Flexor pollicis longus E. Opponens pollicis 43. Which one of the following is true when comparing non-anatomical (reattachment to the brachialis muscle) with anatomical reinsertion of the distal biceps brachii tendon following rupture?  A. No difference in strength of flexion or supination B. Improved strength strength of flexion and supination C. Improved strength of supination D. Decreased strength of flexion and supination E. Decreased strength of supination 44. Which of the following is not not a recognised technique for delayed posterolateral corner reconstruction?  A. Popliteal bypass (Muller’s procedure) B. Figure of eight reconstruction (Larsen’s procedure) C. wo-tailed (Warren’s procedure) D. Tree-tailed (Warren/Miller procedure) E. Mumford procedure 45. Which of the following is not not a cause cause for failed anterior anterior cruciate ligament reconstruction?  A.  Associated posterolateral posterolateral corner corner injury  B. Cyclops lesion impingement C. ibial tunnel placement 10 mm to 11 mm anterior to posterior cruciate ligament insertion D.  Arthrofibrosis of graft E. Returning to full sporting activities at three months

 

SBA Questions

46. Which one of the following statements is false with regards to to hyperbaric oxygen therapy for the treatment of chronic osteomyelitis?  A. It promotes collagen formation B. It improves healing of ischaemic wounds C. It promotes angiogenesis D.  An 85% remission rate has been reported E. It has no direct bactericidal effect 47. With regards regards to metacarpophalangea metacarpophalangeall joint arthritis arthritis of the fingers, which of the following statements is true?  A. It is more common in osteoarthritis than rheumatoid arthritis B. Silicone prostheses have a high rate of fracture requiring revision surgery  C. Silicone prostheses have been shown to improve range of movement in the longer term D. Overall long-term patient satisfaction after implantation i mplantation of silicone prostheses is below 50% E. Unconstrained metal–polyethylene metal–polyethylene prostheses have good predictable results in all patients 48. Which of the following statements statements is false false with regards regards to tibial tibial malunion? malunion?  A. Defined as an angulation of more than 10° in the coronal or the sagittal plane B. Coronal plane malalignment is more symptomatic C. Up to 20° of malalignment can be tolerated without significantly increasing the pressure on the cartilage D. ibial lengthening should be considered for leg-length discrepancy of more than one inch E. Rotational malunion of more than 10° has been shown in up to 22% after tibial nailing 49. Which one of the following is true with regards to the stages stages of posterior tibial tibial tendon insufficiency?  A. Foref Forefoot oot abduction is noted in Stage I B. Dynamic hind foot deformity is noted in Stage II C. Hindfoot valgus is correctable in Stage III D. Correctable ankle valgus is noted in Stage III E. Foref Forefoot oot pronation is present in Stage IV 

 

11

 

12

Chapter 1 SBA Questions

50. Which one of the following is the strongest factor leading to symptomatic symptomatic degenerative disc disease?  A. Heavy lifting B.  Vibration forces forces C. orsional loads D. obacco exposure E. Genetic predisposition 51. In relation relation to orthopaedic screws:  A. Pitch is directly proportional to thread angle B. Locking screws achieve stability by cold welding to the plate C. Te correct drill bit size for the threaded hole for a 4.5 mm cortical screw is 3.5 mm D. Reducing shank diameter increasing the risk of fatigue failure E.  A larger ratio ratio of core diameter diameter to outer diameter diameter leads to higher higher pull out strength 52. Which one of the following statements statements is false with regards to the use use of autotransfusion in orthopaedic surgery?  A. Should be used only if the transfusion risk is more than 10% B. Should not be used if bone cement is being used during joint replacement surgery  C. Contraindicated in infected cases D. Contraindicated in the presence of malignancy  E. Is a good source of clotting factors 53. What is the probable probable mechanism of failure of a cemented total total hip replacement  with radiolucent lines on the anteroposterior anteroposterior radiograph radiograph in Gruen zones two and six?  A. Medial stem pivot B. Calcar pivot C. Cantilever failure D. Pistoning between cement and bone E. Pistoning between cement and implant

54. Regarding genetic transmission, transmission, which one of the following inheritance patterns patterns is seen in patients with familial hypophosphataemic rickets?  A.  Autosomal recessive recessive B.  Autosomal dominant dominant C.  X-link  X-linked ed recessive D.  X-link  X-linked ed dominant E. Mixed pattern

 

SBA Questions

55. Delayed gadolinium-enhanced gadolinium-enhanced MR imaging imaging to detect articular articular cartilage degeneration relies on the content and depletion of which one of the following  f ollowing  in the hyaline cartilage?  A. Proteoglycan B. Chondroitin sulphate C. Keratin sulphate D. Collagen E.  Water 56. During the process of nerve regeneration, regeneration, which one of the following modalities is the first to return?  A. Fine touch B. Deep touch C.  Vibration D. Pain E. Motor 57. A 25-year-old 25-year-old weight-lifter weight-lifter had an MRI examination examination of his shoulder, shoulder, which shows a SLAP lesion with a cyst in the spinoglenoid notch. Which one of the following clinical signs/tests would be expected to be positive in this scenario?  A. Jobe B. Horn blower C. Lift-off  D. Belly push E. Speed 58. All of the following are possible sites for compression of the radial nerve except:  A. Fascial band at radial head B. Edge of extensor carpi radialis brevis C. Recurrent leash of Henry  D.  Arcade of Frohse E. Ligament of Struthers

 

13

59. Which one of the following statements is true about about Wartenberg’s Wartenberg’s syndrome? syndrome?  A. Pain along the ulnar side of forearm B. Para Paraesthesiae esthesiae along the dorso-radial side of the hand C.  Aggrava  Aggravated ted by forearm forearm supination D. Surgery is usually required E. ypically associated with weakness of wrist dorsiflexion

 

14

Chapter 1 SBA Questions

60. Which one of the following would be the most most suitable test to detect a difference difference between the grade of Perthes’ disease diseas e (Herring classification) in two different population groups belonging to the rural or urban areas?  A. Paired t -test -test B.  ANOV  ANOVA A C. Mann–Whitney U test D.  Wilcoxon signed rank rank test E. Kruskal–Wa Kruskal–Wallis llis test 61. Which one of the following would be the most most suitable approach to to stabilise a -type fracture of the acetabulum?  A. Kocher–Langenbeck  B. Ilioinguinal C. Modified Smith–Petersen D. Ilio-femoral approach E. Combined anterior and posterior approach 62. What is the mechanism of injury injury for a typical Weber C fracture as per the Lauge– Lauge– Hansen classification?  A. Supination–abduction B. Supination–extern Supination–external al rotation C. Pronation–abduction D. Pronation–ex Pronation–external ternal rotation E. Pronation–dorsiflexion 63. All of these are associated associated with a pes cavus cavus deformity except?  A. Foref Forefoot oot adduction B. Foref Forefoot oot supination C. Hindfoot varus

D. Plantar flexion of first metatarsal E. Clawing of the great toe

 

Chapter 2 SBA Answers 1. B A co comb mbin inat atio ion n of wh whit ite e bl bloo ood d ce cell ll sc scan an an and da technetium bone scan  There is no single pre-operative investigation investigation that can reliably diagnose a prosthetic joint infection. Diagnosis is reliant on a thorough history and clinical examination. This can be followed by plain radiographs, blood tests, aspiration of  the joint and the use of specialised imaging. In 2010, a working group from the American Academy of Orthopaedic Surgeons released a guideline and evidence report,1 which gave recommendations for the management of prosthetic joint infections. One of the recommendations was that “nuclear “nuclear imaging is an option in patients in whom diagnosis of peri-prosthetic joint infection has not been established and are not scheduled for re-operation.” re-operation.” A combination of white blood cell scan and a technetium bone scan is the most specific scanning method.

2. D 60% Callaghan et al2 performed a retrospective study into the long-term outcomes of  twenty-eight patients following hip arthrodesis. About 60% of the patients had pain in the ipsilateral knee, with an average time to onset of 23 years. A similar percentage had back pain. Pain in the contralateral hip occurred in approximately 25%.

3. C Re Recu curr rren ence ce is se seen en in 5% of ca case sess Subluxation of the head of radius from the annular ligament is a common elbow injury. It is most common amongst young children, prior to the age of six. It I t results from traction on the hand with the elbow extended and the forearm pronated. pronated. The annular ligament either tears or slips over the radial head. After reduction, the elbow joint capsule remains stretched for a few months. Studies have reported recurrence rates up to 5%.

4. B Lo Low w st stra rain in an and d hi high gh oxy xyge gen n te tens nsio ion n  The manner man ner in which mechani mechanical cal factors fac tors influence in fluence fractu fracture re healing heali ng is explain explained ed by Perrin’s Perrin’s strain theory. Strain is the deformation of a material when a given force is applied. It is the change in length in comparison to original length after af ter a given load has been applied. It is expressed as a percentage. Granulation Granulation tissue

 

16

Chapter 2 SBA Answers

has a strain tolerance of 100%, whereas bone has a normal strain tolerance of 2%. Bony bridging between the distal dis tal and proximal callus can only occur when local strain is less than the forming woven bone can tolerate. tolerate. A low amount of strain is therefore ideal. Hard callus will not form if movement is too great. Experimental studies have elucidated that an environment with a high oxygen tension is beneficial for osteogenic progenitor cell differentiation.3

5. A Sup Supra raph phys ysiol iologi ogicc glu gluco coco cortic rticoid oid lev levels els do not cau cause se secondary hyperparathyroidism Glucocorticoid excess can lead to osteoporosis. Glucocorticoids decrease intestinal calcium absorption and decrease renal tubular calcium reabsorption.  This in i n turn raises urinar urinaryy calcium calci um levels. Dimin Diminished ished calcium absorpt absorption ion and an d increase of its excretion can lead to secondary hyperparathyroidism.4

6. D Wat ater er is imb imbibe ibed d by by carti cartilag lage e and and has a pass passiv ive e ro role le in resisting compr compression ession Water shifts in and out of cartilage to allow deformation of cartilage in response to stress. Water Water makes up 65–80% of the wet weight of hyaline cartilage. In osteoarthritis this can rise ris e to 90% but in very severe osteoarthritis the percentage of water can be reduced.5

7. B Fr From om the firs firstt we web b spa space ce to the hoo hook k of the ham hamat ate e Kaplan’s cardinal line is defined as the transverse line drawn from the apex of  Kaplan’s the thumb-index web space towards the hook of the hamate, parallel with the proximal palmar crease. crease. It is just proximal to the superficial palmar arterial arch and is an important consideration during hand surgery.6

8. B Aβ (A beta)7 Table 2.1 Fibre

Function



Alpha motor neurones, muscle spindle primary endings, Golgi tendon organs, touch



Touch, vi vib bration sense se,, musc sclle spindle secondary endings



Touch, pr pressure, ga gamma-motor ne neurones



Pain, cr crude touch, pr pressure, te temperature

 

SBA Answers

9

E Tetracycli etracyclinene-labelled labelled bone biopsy

Osteoporosis is a state of decreased bone mass with normal bone mineralisation. In osteomalacia, bone mass may be variable but mineralisation is decreased. When undertaking blood tests, serum calcium and phosphate are likely to be normal in osteoporosis and may be low or normal in osteomalacia. However However,, osteomalacia as a result of hypophosphatasia may cause elevated calcium and phosphate. Urinary calcium may be high or normal in osteoporosis and low or normal in osteomalacia (high in hypophoshatasia). For these reasons the blood and urine tests will not differentiate di fferentiate well. well. In terms of bone biopsy, tetracycline labelling allows the rate of calcium uptake and turnover to be estimated (mineralisation) and therefore in osteomalacia tetracycline labels are abnormal and they are normal in osteoporosis.5

10. D Axillary nerve (C5, C6) •









Suprascapular nerve (C5, C6) supplies supraspinatus and infraspinatus. Lower subscapular nerve (C5, C6) supplies subscapularis and teres major Upper subscapular nerve (C5, C6) supplies subscapularis Axillary nerve (C5, C6) supplies deltoid and teres minor Medial supraclavicular nerve (C3, C4) is one of several nerves arising from the cervical plexus that supply the skin over the upper medial part of the chest.8

17

11. A Bruising in the anterior triangle of the neck  A pre-ganglionic plexus injury may be associated with bruising in the posterior triangle of the neck rather than the anterior, due to the anatomical location of the pre-ganglionic portion of the brachial plexus.

12. A Male gender Ianotti et al9 found that female gender was more likely to be associated with a poor outcome following a latissimus dorsi transfer. It is postulated that men have an increased muscle mass and therefore improved improved pre-operative shoulder function and strength.

13. B The acute acute angle between the the lateral lateral wall of the ilium and the bony acetabular roof   The alpha alpha angle is used most common commonly ly as a measurem measurement ent of acet acetabula abularr concavi concavity, ty, and it is calculated as the angle between the lateral wall of the ilium and the roofline. A normal alpha angle is 60º or greater greater.. This measurement is vital to understanding the morphology of the immature i mmature acetabulum in developmental dysplasia.

 

18

Chapter 2 SBA Answers

14. D Valgus of 10

°

During development, the tibio-femoral alignment in children changes during early years. At birth it is 10  to 15  of varus, which remodels to neutral at about 24 months of age and then becomes 10° of valgus by the age of three years. Over the next four years it gradually goes to normal alignment of 7°.10 °

°

15. A Collagen type I  Tendons are composed of groups  Tendons groups of collagen bundles (fascicles) separated separated by endotenon and surrounded s urrounded by epitenon. Tendons Tendons are primarily composed of water,, but of the dry mass type I collagen makes up 65–80% of the tendon. Type III water collagen makes up less than 5%. Proteoglycans also make up less than 5% of the tendon’s dry mass. •







 Type I collagen: skin, tendon, vascular,  Type vascular, ligature, organs, organs, bone (main component of bone)  Type  Typ e II collagen: cartilage (main component of cartilage) cartilage)  Type  Typ e III collagen: reticulate reticulate (main component of reticular reticular fibres)  Type  Typ e IV collagen: forms bases bases of cell basement membrane

16. A Pain relief on two two occasions occasions after a facet facet joint joint injection is an indication for facet joint ablation If facet joint injection has provided successful pain relief the diagnosis is verified and therefore facet joint ablation is indicated. i ndicated. However, However, the exact number of lumbar facet joint injections required prior to facet joint ablation is disputed.11

17. E > 80% Kerr et al12 reported a favourable outcome in 84% of patients after coccygectomy for refractory coccygodynia.

18. D There is hind-foot varus deformity  Tarsal tunnel syndrome is a compressive  Tarsal compressive neuropathy caused by compression of the tibial nerve as it traverses the tarsal tunnel. This This is the space between the calcaneus and talus medially, the abductor halluces inferiorly and the flexor retinaculum. Causes of compression compression may be intrinsic, such as a ganglion cyst; or extrinsic, such as valgus deformity of the hindfoot. Symptoms are variable but the main symptom is of vague medial foot pain with occasional burning. There is often an association with intermittent parasthesiae related related to prolonged prolonged standing or

 

SBA Answers

walking. Symptoms may be made worse by eversion and dorsiflexion of the foot. On examination a Tinel’ Tinel’ss sign may be evident and wasting of the intrinsic muscles of the foot may be seen. However, However, the compression test is the most sensitive and specific clinical sign. To To elicit this sign, digital pressure is applied over the tarsal tunnel whilst the foot is held in plantar flexion and inversion. Reproduction of the symptoms is a positive response.

19. A Mean AOF AOFAS AS (Americ (American an Orthopaedic Foo Foott and Ankle Society) Ankle–Hindfoot Ankle –Hindfoot Scale score is higher for patients with ankle arthrodesis A 2007 meta-analysis of 49 primary studies confirmed that the intermediate outcome of total ankle arthroplasty appears to be similar to that of ankle arthrodesis. The mean AOFAS AOFAS Ankle-Hindfoot Ankle-Hi ndfoot Scale score was higher after arthroplasty than after af ter arthrodesis. For arthroplasty, the implant survival rate was 77% at 10 years. Revision rates were 7% after arthroplasty and 9% after arthrodesis.

19

 The main reasons for revision revision were subsidence and/or loosening after arthroplasty and nonunion after arthrodesis. Below-the-knee amputation rates were 1% and 5%, respectively respectively..

20. B Po Polar lar bearin bearing g is more condu conducive cive to fluid film lubric lubrication ation With equatorial bearing fluid lubrication is prevented. Although polar bearing is more conducive to fluid film lubrication, mid-polar bearing is ideal as it allows fluid in and out to lubricate the joint.13,14

21. A Abou Aboutt 15% 15% of all bone tumo tumours urs are prim primary ary spine spine tumours Bone tumours can be benign or malignant. They can also be classified as primary (arising from the bone) or secondary (metastases from different sites). Secondary bone tumours make up the th e vast majority of all bone tumours. Therefore Therefore 15% is a significantly greater percentage than the expected figure for primary spinal tumours.15

22. D Klippel– Klippel–T Trénauna rénaunay–W y–Weber eber syndrome Klippel–Trénaunay–W Klippel–Tr énaunay–Weber eber syndrome is characterised by a triad of port-wine stain, varicose veins, and bony and soft-tissue s oft-tissue hypertrophy involving an extremity. It therefore would not lead to leg length shortening. shor tening.

 

20

Chapter 2 SBA Answers

23. A 0.4%  There is a reported rate of of 0.4% risk of malignant transformation. Malignant Malignant degeneration of fibrous dysplasia complicates less than 1% of all cases, presenting clinically as pain and swelling. Radiographic findings include cortical destruction and associated soft-tissue masses. The most common malignancies include osteosarcoma, fibrosarcoma, and malignant fibrous histiocytoma. Transformation Transformation to chondrosarcoma has been reported, sometimes erroneously on the basis of the incidental finding of cartilaginous nodules in a specimen. The true number of cases of malignant degeneration is likely to be overestimated given previous irradiation 16

of involved bone in many cases.

24. C A Stener lesion will be found, found, if present, proximal to

the adductor aponeurosi aponeurosiss During surgical repair it is important to identify branches of the superficial radial nerve in the proximal aspect of the wound. One should aim to identify the extensor pollicis longus. A Stener lesion,17 which has been reported in up to 80% of cases is found proximal to the adductor aponeurosis. The ulnar collateral ligament nearly always separates from the base of first phalanx of the thumb rather than the metacarpal.

25. C Fun Function ction of the first interosseus muscles A key pinch grip involves opposing the pulp of the distal phalanx of the thumb to the radial border of the proximal phalanx of the index finger.

26. D During distraction osteogenesis Distraction osteogenesis mimics intramembranous ossification where recruitment and differentiation of primitive cells create a new bony framework.

27. A Kniest syndrome Kniest syndrome (spondyloepiphyseal dysplasia) is condition that involves a disproportionate dwarf. Patients have a short trunk and limbs with large joints. It is inherited in an autosomal dominant pattern and is caused by mutations in the s yndrome involves all zones of the growth plate as there is a COL2A1 gene. Kniest syndrome defective collage II molecule assembly.18

 

SBA Answers

28. B Tran ransfor sforming ming gro growth wth facto factor-β r-β19  Transforming growth factor-β  Transforming factor-β (TGF-β) has a proven role role in regulating cartilage and bone formation in fracture callus. In contrast, only some of the more than 20 known bone morphogenic proteins (which all belong to the TGF-β superfamily) induce bone formation; they do this by promoting differentiation of mesenchymal cells into chondrocytes and osteoblasts. Insulin-like growth factor promotes proliferation and differentiation of osteoprogenitor cells. Platelet derived growth factor is a mitogen for both mesenchymal cells and osteoblasts. Endothelial derived growth factor has a role in angiogenesis in fracture healing.

21

29. C Pseudoh Pseudohypop ypopara arathy thyroid roidism ism In this case there is a PTH receptor abnormality and PTH is not able to exert its effect on target cells. This leads to a reduction in the active form of vitamin D. The The low PTH and low active vitamin D levels lead to the low calcium. In hypoparathyroidism hypoparathyr oidism the PTH level would be low as well.18

30. B Use of a thinn thinner er distal distal fem femoral oral augm augmenta entation tion wed wedge ge Using a thinner distal femoral augmentation wedge will increase the extension gap without affecting the flexion gap.20

31. E Reat Reattachi taching ng the com complex plex will lead to painf painful ul restriction of rotation  The Buford complex is i s an anatomic anatomical al variant vari ant of the anterosuper an terosuperior ior labrum. labru m. Williams et al21 noted that it was present in 1.5% of shoulders arthroscoped and consisted of a “cord-like “cord-like”” middle glenohumeral ligament that originated directly from the superior labrum at the base of the biceps tendon and crossed the subscapularis tendon to insert on to the humerus. There was absent anterosuperior labral tissue present between this attachment and the midglenoid notch. Anatomical variant could be confused with labral detachment. However,, if reattached to the glenoid, severe painful restriction of rotation and However elevation would occur.

32.. A Isom 32 Isomet etri ricc  The following are are different types of muscle contraction: contraction: •



isometric–muscle contraction with constant length (e.g. pushing a fixed object) isokinetic–muscle contraction with constant speed

 

22

Chapter 2 SBA Answers









plyometric–initial rapid lengthening followed by contraction of muscle (e.g.  jumping up and down) isotonic–muscle contraction with constant tension concentric–muscle shortens during contraction (e.g. biceps curl) eccentric–muscle lengthens during contraction

33. E Flexor digitorum profundus laceration distal to

lumbrical origin may give a false positive test

An FDP laceration distal to lumbrical origin would lead to a lumbrical plus finger finger,, in which flexion at the MCP MCPJJ would lead to PIPJ PIPJ extension. Therefore a lumbrical plus finger would not give a false positive Bunnell–Littler test.

34. A The deformity deformity is usually due due to an abnormality abnormality in the the lumbrical or flexor digitorum superficialis insertion inser tion  There is very rar  There rarely ely any com compro promise mise in functio function; n; ther therefo efore re ope operat rative ive inte intervent rvention ion is usually not indicated. Infantile type camptodactyly can respond to stretching. If multiple digits are involved they can be associated with other syndromes, although this is rare.

35. A It results results from from a defect defect in the motor motor unit Arthrogryposis is a rare congenital disorder that is characterised by nonprogressive joint contractures. Causes can be extrinsic (severe oligohydramnios or other feto/maternal reasons for reduced fetal movement) or intrinsic (CNS, PNS, connective tissue disorders). Other organ abnormalities can be noted as well.

  Table 2.2 Grad Gr ade e

Desc scri rip ptio ion n

A

Good evide Good evidence nce (L (Leve evell I stud studies ies wit with h cons consist istent ent find finding ing)) for for or again against st reco recommen mmendin ding g intervention.

B

Fair evi eviden dence ce (L (Leve evell II or III III stud studies ies wit with h consis consisten tentt findin findings) gs) for for or or agains againstt reco recommen mmendin ding g intervention.

C

Poor-q Po or-qual uality ity evi eviden dence ce (L (Leve evell IV IV or or V) for for or or agai against nst rec recomme ommendi nding ng inte interven rventio tion. n.

D

There is There is insuffi insufficie cient nt or or confli conflicti cting ng evide evidence nce not all allowi owing ng a reco recommen mmendat dation ion for or agai against nst intervention.

See also levels of evidence http://www.aaos.org/research/evidence/levelstables.pdf 

 

SBA Answers

36. E An aspir aspirate ate whit white e blood blood cell cou count nt of > 50 50 000/m 000/mm3 m3 with 75% polymorphonuclear leucocytes is diagnostic of  sepsis in 60% to 70% of patients Male gender is a risk factor fac tor for septic arthritis. The knee and the hip are affected

23

in around one third of cases each, with the lower extremity accounting for the majority of cases.22–24 An aspirate, WBC count > 50 000/mm3 with 75% polymorphonuclear leucocytes is diagnostic of sepsis in 30–50%.

37. A Summarise treatmen treatmentt efficacies (risk ratio) across trials Forest plots are graphical representations of meta-analysis. They show the effect estimate and weight of each individual study as well as the overall effect estimate of combined studies.  The effect measures that are commonly commonly used include odds ratio and risk ratio amongst others. A box and a horizontal line represent each study. The The mid-point of the box represents point effectstudy. estimate of each study.of The The of the box the weight the given to each The width/length thearea horizontal linerepresents represents the 95% confidence interval for the effect estimate of each study. A diamond represents the overall effect estimate. The width of the diamond represents the confidence interval for the overall effect estimate.  There is also a vertical line, known as the line of no effect. effect.  The Forest Forest plot plot also provide providess the summary data data entered entered for each study. study. In addition, it provides the weight for each study, the effect measure, method and the model used to perform the meta-analysis, the confidence intervals used, the effect estimate from each study, the overall effect estimate and the statistical significance of the analysis. Information is also provided about the heterogeneity of the analysis.

38. B A systematic review of Level I studies investi investigating gating the results of treatment with inconsistent results would constitute a fair evidence study (Grade B) AAOS Grades of Recommendation for Summaries or Reviews of Orthopaedic Surgical Studies: http://www.aaos.or http://www.aaos.org/research/evidence/g g/research/evidence/gradesofrec.asp radesofrec.asp

39. D Hya Hyalur luroni onicc aci acid d

 There is evidence eviden ce that all of these the se agents have some s ome effect on disease di sease progression: •

Glucosamine (Cochraine review),

 

24

Chapter 2 SBA Answers





Chondroitin (various studies, Cochrane review in progress), Diacetylrhein (Cochrane review),



Doxycycline (Cochrane review “The symptomatic benefit of doxycycline d oxycycline is minimal to non-existent. The small benefit in terms of joint space narrowing is of questionable clinical relevance and outweighed by safety problems. Doxycycline should not be recommended for the treatment of osteoarthritis of  the knee or hip”),



Hyaluronic acid (Cochrane on viscosupplementation does not comment on disease progression, butreview concludes that HA is beneficial for pain and function. Some papers quote a positive effect on disease progression).

40. C It reduc reduces es the the need for tran transfusi sfusion on after join jointt replacement  Tranexamic acid significantly reduces the need for  Tranexamic for allogeneic blood transfusion transfusion 25,26 after joint replacement surgery.  Meta-analysis of existing RCTs does not support an increased risk of thromboembolic events. However, However, the overall effect on mortality, DVT and PE remains uncertain.

41. B 10° to 20° 20° of of dorsifle dorsiflexion xion is ideal ideal to prese preserve rve grip grip strength Maximum power grip is achieved at around 35° of extension, but this position would interfere with activities of daily living. In patients with rheumatoid arthritis, 5° to 10° ulnar deviation may be desirable to counteract the “Z” deformity. The rheumatoid wrist should be fused in slight flexionbecause the rheumatoid patient will often have difficulty with MCPJJ extension, and therefore MCP therefore,, the wrist should be fused in slight flexion to accommodate the extensor tenodesis effect at the MCP MCPJ. J.

42. A Abd Abduct uctor or pol pollic licis is lon longus gus In the surgical s urgical treatment of de Quervain’s Quervain’s stenosing tenosynovitis the first extensor compartment is decompressed. Specifically the extensor pollicis brevis and abductor pollicis longus tendons, either of which may be lying in a separate sub-sheath.

43. E Decr Decreased eased stre strength ngth of supi supinati nation on Klonz et al27 showed that following brachialis tenodesis supination strength was only 42% to 56% of the uninjured arm, compared with 91% supination strength following anatomic repair. repair. Flexion strength was 97% following anatomic repair and 96% following brachialis tenodesis.

 

SBA Answers

25

44. E Mu Mumf mfor ord d pr proc ocedu edure re  The Mumford procedure procedure is done to treat treat acromioclavicular acromioclavicular joint disorders. The distal end of the clavicle bone is resected to relieve the symptoms. This procedure procedure may be done arthroscopically ar throscopically or open.

45. C Tib Tibial ial tunne tunnell placeme placement nt 10 mm to to 11 mm anter anterior ior to to posterior cruciate ligament insertion Hutchinson and Bae28 showed that the anterior border of PCL was consistently 10.9 mm posterior to central sagittal insertion inser tion point of the ACL in a cadaveric study.

46. E It has no dire direct ct bacte bactericid ricidal al effe effect ct An 85% remission rate has been reported with surgery, antibiotics and HBOT together by Morrey et al.29 Hyperbaric oxygen therapy is bacteriostatic and bactericidal as it increases oxygen free radicals.

47. D Over Overall all long long-term -term pati patient ent sati satisfactio sfaction n after implantation of silicone prostheses is below 50% Möller et al30 showed that there was a silicone spacer fracture in 20 out of 120 implants in a prospective RCT. Silicone prosthesis have a higher rate of fracture but not all require revision surgery. Unconstrained prosthesis don’t have predictable results in rheumatoid patients with loss of soft tissues and collateral ligaments.31

48. A Define Defined d as an angul angulatio ation n of mor more e than than 10° 10° in in the the coronal or the sagittal plane  Tibial malunion is i s defined as a s an angulation more than 5° in the coronal or the 32 sagittal plane.

49. B Dynam Dynamic ic hind hind foo foott deformi deformity ty is note noted d in Stag Stage e II 33

Johnson and Strom et al  described tibialis posterior dysfunction in 4 stages. A dynamic hindfoot valgus deformity, attenuation of the spring ligament and progressive flattening of the longitudinal arch characterise stage II.

 

26

Chapter 2 SBA Answers

50. E Genetic predisposition34  The aetiology of disc degeneration is complex complex and multifactorial, involving involving a degenerative cascade of events in which mechanical and genetic factors combine in an age-related process. Evidence from a Japanese case–control study suggests that genetic susceptibility is the major factor.

51. D Reducing shank diameter increases the risk of fatigue failure •











Pitch is the distance between threads Lead is the distance dis tance advanced by a single revolution Screw working length is the length of bone traversed by the screw Inner diameter (core/root diameter) is the diameter of the metal core of the screw discounting the thread depth; it affects the strength of the screw (i.e. its risk of fatigue failure) Outer diameter is the total diameter including thread depth Pull-out strength is maximised by having a large difference between core and outer diameters (i.e. deeper threads) and having a fine pitch (i.e. more threads)

52. E Is a good good source of clotting factors Autotransfusion is not a good source of clotting factors (as they become depleted) and it is commonly employed when cement is used.

53. A Medial stem pivot With a lack of superomedial and inferolateral cement support the prosthesis pivots around the medial stem eventually causing cement fracture in Gruen zones 2 and 6.

54. D X-linked dominant  The inheritance patterns of common orthopaedic conditions conditions are commonly tested. Generally structural conditions (e.g. achondroplasia) are inherited in an autosomal dominant manner and enzyme deficiencies (e.g. Gauchers disease) are inherited in an autosomal recessive manner. The common X-linked recessive conditions are muscular dystrophy (both Becker’s Becker’s and Duchenne’s) Duchenne’s) and haemophilia. X-linked dominant conditions are rare; however, however, both Hypophosphataemic H ypophosphataemic rickets and LeriWeill dyschondrosteosis are inherited in this manner mann er..

 

SBA Answers

55. A Proteoglycan Gadolinium allows accurate assessment of the amount of proteoglycan in the articular cartilage.35

56. D Pain Recovery of deep cutaneous sensibility (pain caused by deep pressure) is the first sign of nerve recovery.

57. E Speed Speeds test is undertaken with the elbow extended, forearm supinated and humerus elevated to 60° – positive for long head of biceps pathology and SLAP tears.

58. E Ligament of Struthers  The ligam l igament ent of o f Struthe St ruthers rs runs ru ns betwe between en the t he humeru h umeruss and an d the th e medial me dial epicondyle. Its proximal attachment to the humerus may be from a supracondylar process or from the humerus itself. It is a possible site of compression of the median nerve.

59. B Paraest Paraesthesia hesia along along the dorso-radial side of of the hand Wartenberg’s syndrome is caused by compression of the superficial sensory branch of the radial nerve between the tendons of brachioradialis and extensor carpi radialis longus with forearm pronation. Symptoms include pain, numbness and paraesthesia over the dorsoradial aspect of the hand. Provocative tests include forceful forearm pronation for 60 seconds and a Tinel sign over the nerve.

60. E Krusk Kruskal–W al–Wallis allis test Here we are comparing two different groups that are not paired or matched. The The Herring classification36system is an ordinal measurement (i.e. Grade A, B, or C, with increasing severity).

27

 

28

Chapter 2 SBA Answers

61. A Kocher Kocher–La –Lange ngenbe nbeck  ck   This patt pattern ern com combine biness a tran transver sverse se compo component nent with a stem stem which which exit exitss either either thr through ough the obturator ring or – in unusual cases – at various levels through the ischium. Factors which must be considered when making Factors maki ng the approach choice are: Level of anterior and posterior column fractures, i.e. transtectal, juxtatectal, or infratectal Relative column displacement Presence and configuration of posterior wall involvement Associated marginal impaction •







 The majority of fracture patterns patterns have predominant predominant posterior column displacement displacement and therefore can be operated through a Kocher-Langenbeck approach. In cases where there is significant displacement of both columns the decision must be made whether to use a sequential ilioinguinal/Kocher-Langenbeck, or an extended iliofemoral approach. An alternative to the extended iliofemoral approach for complex complex patterns in older patients would be the Kocher-Langenbeck with digastric trochanteric osteotomy (trochanteric flip extension). This approach can be used with wi th or without true surgical dislocation. However, However, this alternative should not be considered in any way less demanding than the extended ex tended iliofemoral.37

62. D Pr Pronat onation–ext ion–external ernal rot rotatio ation n Weber C fractures occur with the foot in pronation when an external rotation force is applied. The first injury is a failure of the medial side, either with a deltoid ligament rupture or a medial malleolar fracture. The talus moves anteriorly as it rotates externally. The talus causes the fibula to rotate externally around its longitudinal axis, sequentially tearing the anterior and an d interosseous ligaments. Finally the fibula fractures proximal to the syndesmosis.

63. B Fo Foref refoot oot supin supinatio ation n A pes cavus deformity is associated with hindfoot varus, forefoot adduction, plantar flexion of the first metatarsal, forefoot pronation pronation and clawing of the toes.

 

References

References 1.

Della Valle C, Parvizi J, J, Bauer TW, et al. American Academy of Orthopaedic Surgeons clinical clinical practice guideline on: the diagnosis of periprosthetic joint infections of the hip and knee. J Bone Joint Surg [Am] 2011;93-A:1355-7. 2. Callaghan JJ, Brand RA, Pedersen DR. Hip arthrodesis: a long-term follow-up.J follow-up.J Bone Joint Surg [Am] 1985;67-A:1328-35. 3. D’Ippolito G, Diabira S, Howard GA, Roos BA, Schiller PC. Low Low oxygen oxygen tension inhibits osteogenic differentiation and enhances stemness of human MIAMI cells. Bone 2006;39:513-22. 4. Sobhani A, Moradi Moradi F, F, Pasbakhsh P, P, et al. Effects of glucocorticoid on bone metabolism markers and bone mineral density in rats. J Dent 2005;2:64-9. 5. Miller MD. MD. Review of Orthopaedics. Fifth ed. Philadelphia: Saunders Elsevier, 2008. 6. Green DP, DP, Hotchkiss RN, Pederson Pederson WC. Green’ Green’ss operative hand surgery. Sixth ed. New York: Churchill Livingstone, 2005. 7. FitzGerald MJT, Gruener G, Mtui E. Clinical neuroanatomy neuroanatomy and neuroscience. Philadelphia: Saunders. 8. Snell RS. Clinical anatomy. anatomy. Seventh ed. Philadeplphia: Lippincott Williams & Wilkins, 2003. 9. Iannotti JP, JP, Hennigan S, Herzog R, et al. Latissimus dorsi tendon transfer for irreparable posterosuperior rotator cuff tears: factors affecting outcome. J Bone Joint Surg [Am] 2006;88-A:342-8. 10. Salenius P, P, Vankka E. The development development of the tibiofemoral angle in children. J Bone Joint Surg [Am] 1975;57-A:259-61. 11. Cohen SP, SP, Williams KA, Kurihara C, et al. Multicenter, Multicenter, randomized, comparative cost-effectiveness study comparing 0, 1, and 2 diagnostic medial branch (facet joint nerve) block treatment paradigms before lumbar facet radiofrequency denervation. Anesthesiology 2010;113:395-405. 12. Kerr EE, Benson D, Schrot Schrot RJ. Coccygectomy for chronic refractory coccygodynia: clinical case series and literature review. J Neurosurg Spine 2011;14:654-63. 12a. Haddad SL, Coetzee JC, Estok R, Fahrbach K, Banel D, Nalysnyk L. Intermediate and long-term outcomes of total ankle arthroplasty and ankle arthrodesis. A systematic review of the literature. J Bone Joint Surg Am 2007;89:1899 -905. 13. Miller MD. Review of orthopaedics. Fifth ed. Philadelphia: Saunders Elsevier, 2008. 2008. 14. Canale ST, ST, Beaty JH, eds. Campbell’s Campbell’s Operative Orthopaedics. 11th ed. Philadelphia: Philadelphia: Mosby Elsevier; 2007. 15. Patchell RA, Tibbs Tibbs PA, Regine WF WF,, et al. Direct decompressive surgical resection in the treatment of spinal cord compression caused by metastatic cancer: a randomised trial. Lancet 2005;366:643-8. 16. Schwartz DT, DT, alpert M. The malignant transformation of fibrous dysplasia. Am J Med Sci 1964;247:1-20. 17. Stener B. Displacement of the ruptured ulnar collateral collateral ligament of the metacarpo-phalangeal joint of the thumb. J Bone Joint Surg [Br] 1962;44-B:869-79. 18. Miller MD. Review of orthopaedics. Fifth ed. Philadelphia: Saunders Elsevier, 2008. 2008. 19. Joyce ME, Jingushi S, Bolander ME. Transforming Transforming growth factor-beta in the regulation regulation of fracture repair. Orthop Clin North Am 1990;21:199-209. 20. Mihalko WM, Krackow KA. Flexion and extension gap balancing in revision total knee arthroplasty. Clin Orthop Relat Res 2006;446:121-6 21. Williams MM, Snyder SJ, Buford D Jr. The Buford complex: the “c “cord-like” ord-like” middle glenohumeral ligament and absent anterosuperior labrum complex: a normal anatomic capsulolabral variant. Arthroscopy 1994;10:241-7. 22. Goergens ED, McEvoy A, Watson M, Barrett IR. Acute osteomyelitis osteomyelitis and septic arthritis in children. J Paediatr Child Health 2005;41:59-62.

29

23. McCarthy Dormans JP, recent JP, Kozin SH, Pizzutillo PD.Instr Musculo Musculoskeletal skeletal infections in children: basic treatmentJJ, principles and advancements. Course Lect 2005;54:515-28. 24. Kang SN, Sanghera T, T, Mangwani J, Paterson JM, Ramachandran M. The management of septic arthritis in children: systematic review of the English language literature. J Bone Joint Surg [Br] 2009;91-B:1127-33. 25. Sukeik M, Alshryda S, Haddad FS, Mason JM. Systematic review review and meta-analysis of the use of tranexamic acid in total hip replacement. J Bone Joint Surg [Br] 2011;93-B:39-46. 26. Alshryda S, Sarda P, P, Sukeik M, et al. Tranexamic Tranexamic acid in total knee replacement: a systematic review and meta-analysis. J Bone Joint Surg [Br] 2011;93-B:1577-85.

 

30

Chapter 2 SBA Answers

27. Klonz A, Loitz D, Wöhler P, P, Reilmann H. Rupture of the distal biceps brachii tendon: isokinetic power analysis and complications after anatomic reinsertion compared with fixation to the brachialis muscle. J Shoulder Elbow Surg 2003;12:607-11. 28. Hutchinson MR, Bae TS. Reproducibility of anatomic tibial landmarks for anterior cruciate cruciate ligament 29. 30.

31. 32. 33. 34. 35.

reconstructions. Am J Sports Med 2001;29:777-80. Morrey BF, BF, Dunn JM, Heimbach RD, Davis J. Hyperbaric oxygen and chronic osteomyelitis. Clin Orthop Orthop Relat Res 1979;144:121-7. Möller K, Sollerman C, Geijer M, Kopylov P, P, Tägil Tägil M. Avanta versus Swanson silicone implants in the MCP joint: a prospective, randomized comparison of 30 patients followed for 2 years. J Hand Surg Br 2005;30:8-13. Fischgrund JS. Orthopaedic Knowledge Update: No. 9. Rosemont: American Academy Academy of Orthopaedic Surgeons, 2008: 366. Fischgrund JS. Orthopaedic Knowledge Update: No. 9. Rosemont: American Academy Academy of Orthopaedic Surgeons, 2008: 479. Johnson KA, Strom DE. Tibialis posterior tendon dysfunction. Clin Orthop Relat Res 1989;23:196-206. Matsui H, Kanamori M, Ishihara H, et al. Familial Familial predisposition for lumbar degenerative disc disease: a case-controlled study. Spine 1998;23:1029-34. Gray ML, Burstein D, Kim YJ, YJ, Maroudas A. 2007 Elizabeth Winston Lanier Award Winner. Winner. Magnetic

resonance imaging of cartilage glycosaminoglycan: basic principles, imaging technique, and clinical applications. J Orthop Res 2008;26:281-91. 36. No authors listed. BMJ. Study design and choosing a statistical test. http://www.bmj.com/about-bmj/ resources-readers/publications/statistics-square-one/13-study-design-and-choosing-statisti (date last accessed 7 March 2013). 37. Mayo K, Oransky M, Rommens P, P, Sancineto C. AO Foundation. Acetabulum: Acetabulum: T-type T-type fractures with Kocher-Langenbeck. http://tinyurl.com/bvkgvm4 (date last accessed 7 March 2013).

 

Chapter 3 Viva Questions: Adult pathology 32-year-old woman presents with with a history of pain and occasional clicking in 64. A 32-year-old her right hip. She gives a history of a skiing injury to her right groin 7 months earlier.. Tis is her radiograph (Fig. 3.1). earlier  A. Describe the abnormality on the radiograph in this patient with hip pain. B. Describe the various angles that can be measured to document hip morphology. C.  What further investigations investigations should be performed to investigate investigate the hip pain? D.  What are are the surgical options for management? management? E.  What are are the classification classification systems for more more severe cases? cases?

Figure 3.1

 

32

Chapter 3 Viva Questions

65. A 58-year-old 58-year-old keen runner runner presents with a history of hip pain especially after running. Tis is the radiograph obtained in the clinic (Fig. 3.2).  A. Describe the abnormality on the radiograph radiograph.. B. Describe the likely presentation and mechanism of injury of this patient. C.  What further investigations investigations should be performed? D.  What are are the options for management? management? E. Describe the blood supply to the femoral head.

Figure 3.2

66. A 28-year-old 28-year-old man presents presents with pain in his chest and lower back. Tis is his

radiograph (Fig. 3.3).  A. Describe the abnormality on the radiograph radiograph.. What is your diagnosis? B. How can we assess the severity of this condition? C.  What determines the the prognosis? D.  What other investigation/s investigation/s would you you like to request request and why? E. Describe a classification system for this pathology. F.  What are are the options for treatment? treatment?

 

Adult pathology

Figure 3.3

67. A.  Describe the abnormalities on this radiograph radiograph (Fig. 3.4) 3.4) taken one year after revision hip arthroplasty. B. Describe the stages of the pathological process seen around the right hip. C.  What is the common common classification classification used for this disease process? process?

33

D.  What can be done to reduce the incidence of this this process? E.  What are are the consequences of the the other abnormality? abnormality?

Figure 3.4

 

34

Chapter 3 Viva Questions

68. A 68-year 68-year-old woman presents with a history of pain in the region of her thumb. Tis is her-old radiograph (Fig. 3.5).with  A. Describe the radiograph. What is the diagnosis? B.  What are are the causes of this condition? C.  What is the treatment treatment for this condition? D.  What ar aree the possible complications complications of surgical surgical intervention in patients with this condition?

Figure 3.5

69. A 42-year-old 42-year-old female presents with a history of lower back and right leg pain. Tis is her radiograph (Fig. 3.6).  A. Describe the abnormalities seen in the radiograph. B.  What are are the associations of this this condition? C.  Wha  Whatt is the app approx roxima imate te incid incidenc encee of this abno abnorma rmality lity in the the gene general ral pop popula ulation tion?? D.  What are are the main surgical surgical implications of this condition? E. Describe the embryology of the spine.

Figure 3.6

 

Adult pathology

70. A 44-year-old 44-year-old microlight pilot pilot sustained this injury (Fig. 3.7) in a crash landing. landing.  A. Describe the radiograph radiograph.. B. Tere is an 8 cm clean laceration overlying the injury with a pale foot and no pulses. Describe your treatment strategy for this patient. C.  What is the common common classification classification of talar fractures fractures and what what is the likely likely risk of nonunion in this case? D.  What is Hawkin’ Hawkin’ss sign and what what is the pathogenesis pathogenesis of this phenomenon? E. Discuss the blood supply to the talus.

 

35

Figure 3.7

 

36

Chapter 3 Viva Questions

71. A 72-year-old 72-year-old man presents with a long-standing long-standing history of knee pain, which is affecting his daily life and recreational activities. his is his radiograph (Fig. 3.8).  A. Describe the radiograph radiograph.. B.  What other investigations investigations are are appropriate? appropriate? C. Te patient has a significant fixed flexion deformity and fixed valgus. Assuming surgical intervention is appropriate discuss what form of knee replacement  would be appropriate? appropriate? D. Describe a method of lateral ligament complex release release and balancing the knee for valgus knees? E.  What is the importance importance of the possible medial joint line opening seen in the radiograph?

Figure 3.8

 

Adult pathology

72. A fit and well 65-year-old 65-year-old fell down stairs five years after a successful total hip replacement. He presents with pain in his right hip and an inability to bear  weight. Tis is his radiograph (Fig. 3.9).  A. Describe the radiograph radiograph.. B. Describe a classification system for this injury that helps to guide management. C.  Where does this this injury fit with the classification classification system you you have described? described? D. How would you treat this injury? E.  What is the expected expected outcome following following treatment? treatment?

 

37

Figure 3.9

 

38

Chapter 3 Viva Questions

73. A 75-year-old 75-year-old woman presents with acute pain in the right hip after attempting attempting to tie her shoelaces. sho elaces. She had a total hip replacement 12 months previously. Tis is her radiograph (Fig. 3.10).  A. Describe the radiograph radiograph.. B.  What factors factors influence the risk of dislocation dislocation following a total total hip replacement? C.  What are are the options for management management of this this patient? D.  What is the prognosis prognosis if the patient patient underwent a revision for for this problem? E.  What are are the options for treatment treatment for recurrent recurrent dislocation dislocation following abductor abductor insufficiency?

Figure 3.10

 

Adult pathology

74. A 54-year-old woman who underwent a mastectomy for breast cancer 12 months ago followed by adjuvant chemotherapy presents with a two week history of increasing mid-thoracic back pain. Over the last two days her legs have felt  weak and and she has has struggled struggled to walk. walk. Tere Tere is no history history of urinary urinary or faecal incontinence. She is otherwise fit and well. Tis is the MRI of her spine (Fig. 3.11).  A. Describe the image.

39

B.  What is the likely likely diagnosis and how how would you manage manage this patient patient in the  A&E department? C.  What are are the options for definitive definitive management? management? D.  What factors factors govern the decision decision making process? process?

Figure 3.11

 

40

Chapter 3 Viva Questions

75. A 42-year-old 42-year-old woman presents with with a history of anterior knee pain for the past three years. She has difficulty climbing up and down the stairs and also finds it fairly difficult to drive for long distances because of the pain. Tese are her radiographs (Figs 3.12a to c).

 A. Describe the radiographs radiographs.. B.  What is the diagnosis diagnosis and what is the aetiology? C.  What are are the options for management management in this situation? D.  What factors factors govern the decision decision making process? process? E. If you decided to go down the arthroplasty route, route, what prosthesis would you use and why? F. How are patellofemoral joint replacements performing on the National Joint Registry in the UK?

a

b

c

Figure 3.12a to c

 

Chapter 4

Viva Answers: Adult pathology 64A. Describe the abnormality on the radiograph radiograph in this patient with with hip pain.  The radiograph shows shows evidence of mild dysplasia with a reduced lateral lateral centre-edge angle of Wiberg and an abnormal femoral head extrusion index. Asphericity of the right femoral head/neck junction consistent with cam type t ype femoroacetabular femoroacetab ular impingement is also noted.

64B. Describe the various angles that can be measured to document hip morphology. A number of angles can be measured around the hip to help establish pathology pathology.. 1. Acetabular inclination (Tönnis (Tönnis angle) – This is the angle angle between two lines: a. a line from the most inferior point of of the acetabular sourcil to the lateral margin of the acetabular sourcil b. a horizontal line running through the most inferior inferior part of the sourcil. This latter line is parallel to the transverse pelvic axes that can be determined by a line connecting the base of both acetabular teardrops. teardrops. A normal  Tönnis  T önnis angle is between 0 and 10°. A decreased Tönnis angle can lead to pincer form of FAI and an increased Tönnis Tönnis angle can indicate structural instability. 2. Sharp’ Sharp’ss angle – This acetabular angle angle is creating by by measuring the intersection between the following two lines: a. a horizontal line from from the inferior aspect of of one teardrop teardrop to the other and b. a line from the inferior aspect of the teardrop teardrop to the superolateral superolateral margin of the acetabulum A normal angle is between 33° and 38°. Higher angles, especially those above 47°, suggest dysplasia. 3. Lateral centre-edge centre-edge angle of Wiberg Wiberg – This assess the superolateral superolateral coverage coverage of the femoral head. It is obtained by calculating the angle between two lines: a. a vertical line through the centre of of the femoral head (perpendicular to the transverse pelvic axis), and b. a line from the sourcil. centre of the femoral femoral head20° to the supero superolateral lateral margin marg of the acetabular Values Values of less than indicate dysplasia and in values above 40° may notify pincer FAI.

 

42

Chapter 4 Viva Answers

4. Anterior centre centre edge angle of Lequesne Lequesne – This is measured on the false profile view. It is the angle between two lines: a. a vertical line through the centre of of the femoral head and b. a line from the centre of the femoral femoral head to the most anterior point point of the acetabulum. It is a measure of anterior coverage of the femoral head. An angle less than 20 is suggestive of instability.

64C. What furt further her investigations should be perfor performed med to investigate the hip pain? A cross-table lateral and an MRI of the right hip should be performed to investigate the cause of the hip pain. The MRI scan can detect chondral/labral pathology and extracapsular abnormalities that may be causative.

64D. What are the surgical options for management? Surgical options include a diagnostic hip injection with EUA to confirm the intraarticular origin of pain if in doubt, arthroscopic hip surgery (manage chondrolabral pathology and excision of a cam impingement lesion). Another option is open hip surgery +/+ /- dislocation of the head/periarticular osteotomy. osteotomy.

64E. What are the classificat classification ion systems for more severe cases?  The Crowe classification classification of hip dysplasia is based on the extent of proximal migration of the femoral headi 1. 2. 3. 4.

less than 50% subluxation 50 to 75% 75 to 100% greater than 100% subluxation

65A. Describe the abnormality abnormality on the radiogr radiograph. aph.  There is a transcervical fracture of the right right neck of femur. femur. This This is minimally displaced on the AP view and is on the tension side of the neck.

65B. Describe the likely likely presentation presentation and and mechanism mechanism of of injury of this patient.  This is is likely likely to to be a stress stress fractur fracture. e. The radiog radiograph raph repr represents esents a female female pelvis pelvis and at at the age of 58 the patient is very likely to post-menopausal. As the protective effects of oestrogen on bone (promotes bone formation) are diminished dimi nished post-menopause, patients may develop osteoporosis and are more susceptible to fractures. In this case, the stress fracture has likely resulted from repetitive microtrauma from running. The The presentation of stress fracture is normally insidious and slow due to the microtrauma that occurs. These These aresevere. normally very active and will repetitive only seek medical attention when the painpatients becomes

65C. What further investigatio investigations ns should should be performed? performed? Blood tests should be performed including calcium, phosphate and alkaline phosphatase to rule out causes other than osteoporosis. Urinary calcium should

 

Adult pathology

also be measured. A DEXA (dual energy X-ray absorptiometry) scan should also be undertaken – however this would likely follow the initial management. Additionally pre-emptive pre-operative bloods tests should be undertaken including FBC, U&E and Group and Save.

65D. What are the options for management? Options for management would depend on the full history and patient comorbidities and beliefs. As the patient is a keen runner it is likely that the patient would like to return to that level of function, this would be very unlikely without operative intervention. Operative options include closed reduction and internal fixation, open reduction and internal fixation and arthroplasty options. As the patient is 58 years old, bone preservation is preferred to arthroplasty and thus fixation would be the operation of choice. Additionally, as the fracture appears minimally displaced on the AP view, fixation is likely to be possible with a closed reduction, leading to the least disruption of blood supply to the femoral head. Fixation options would include cannulated screws or 2-hole dynamic hip screw with a derotation screw.  The major concern with closed reduction reduction and internal fixation is avascular necrosis to the femoral head

65E. Describe the blood supply to the femoral head.  The blood supply to the femoral femoral head is as follows: follows: Extracapsular arterial ring at the base of the femoral neck, formed posteriorly by large branch of medial femoral circumflex artery formed anteriorly by smaller branches of lateral femoral circumflex artery superior and inferior gluteal arteries have minor contributions







Ascending cervical branches: these give rise to retinacular arteries gives rise to subsynovial intra articular ring



Artery of ligamentum teres: derived from obturator or medial circumflex femoral artery forms the medial epiphyseal vessels only small amount of the femoral head is nourished this artery







Epiphyseal blood supply: arises primarily from lateral epiphyseal vessels that enter head posterosuperiorly





vessels from medial epiphyseal artery entering through ligamentum teres Metaphyseal blood supply: arises from extracapsular arterial ring arise from branches of ascending cervical arteries and subsynovial intra-articular ring





43

 

44

Chapter 4 Viva Answers

66A. Describe the abnormality on the radiograph. radiograph. What is your diagnosis?  The radiograph radiograph shows an abnormal abnormal right-sided right-sided lateral curvature of the thoracic thoracic spine. The diagnosis in this 28-year-old is of adult scoliosis. This is most likely to be due to pre-existent idiopathic adolescent scoliosis, but this also could be secondary to a neuromuscular disease, a tumour or degenerative change.

66B. How can we assess the severity of this condition? condition?  The severity can be assessed by performing a full history and examination to elucidate the clinical effect of the scoliosis. The severity of the curve can be radiologically assessed by measuring the Cobb angle. This is derived by drawing intersecting perpendicular lines from the superior surface of the superior end vertebra of the curve and from the inferior surface of the inferior end vertebra of the curve. Vertebral rotation can also be measured to give an indication i ndication of severity.

66C. What determines the prognosis?  The severity of the curve would determine prognosis. prognosis. Curve progression progression is unlikely if the Cobb angle is less than 30°. Progression is highest for right sided thoracic curves > 50° (1 mm/year). The underlying causative reason behind the scoliosis would naturally affect prognosis. A simple idiopathic i diopathic adolescent scoliosis at this stage at the age of 28 would have a much better prognosis than a neuromuscular or a tumour-related curve.

66D. What other investigation/s would you you like to request and why? why? Other investigations that we be appropriate would include posteroanterior posteroanterior and lateral radiographs of the whole spine with the patient standing. Lateral bending films are also of benefit if planning further treatment. An MRI would be essential to rule out a syrinx and also if there were any features features to suggest a malignancy or any neurological findings on examination.

66E. Describe a classificat classification ion system for this pathology.  The classification of King et al1 is used to describe thoracic curves as follows; •









King I – lumbar curve larger than thoracic curve, which is more flexible on bending films. King II – thoracic curve is larger than lumbar curve, less flexible and the thoracic rib hump is larger than the lumbar rotational prominence. King III – a thoracic scoliosis where the lumbar curve does not cross the midline. King IV – single long thoracic curve King V – double structural thoracic curve.

66F.. What are the options for treatment? 66F

 The options for treatment treatment are non-operative and operative. operative. In this case, if no suspicious features were discovered on history and examination, simple observation and repeat interval radiography of this < 30° curve would be

 

Adult pathology

indicated. Analgesia and physiotherapy would also be of benefit. Other options for younger patients with more severe curves would include orthotic bracing or casting and possibly operative intervention. This may include an anterior/ posterior approach with instrumented (Harrington, CD and Luque) fusion.

67A. Describe the abnormalitie abnormalitiess on the radiogr radiograph, aph, taken one year year after revision hip arthroplasty.  There is heterotopic heterotopic ossification (HO) around the right hip joint. I would would classify this as Brooker Grade III.2 There is also significant osteoarthritis of the left hip and there is also an element of leg length discrepancy.

67B. Describe the stages of the pathological pathological process process seen around the right hip. HO is, by definition, the formation of bone within soft tissue. The transformation of primitive cells of mesenchymal origin, present in the connective tissue septa within muscle, into osteogenic cells is thought to be the pathogenesis. Chalmers et al3 proposed three conditions needed for HO: osteogenic precursor cells, inducing agents, and a permissive environment.  The heter heterotop otopic ic bone bone may begi begin n some distanc distance e from from normal normal bone, bone, later later movin moving g toward it. Studies have also shown that muscle injury alone will not cause the ectopic ossification, concomitant bone damage also being required.4 Other contributing factors include hypercalcemia, tissue hypoxia, changes in sympathetic nerve activity, prolonged immobilisation and imbalance of PTH and calcitonin. Early in the course of HO, oedema with exudative cellular infiltrate is present, followed by fibroblastic proliferation proliferation and osteoid formation. The development of HO is extra-articular and bone forms in the connective tissue between the muscle planes and not within the muscle itself. i tself. The new bone can be continuous with the skeleton but generally does not involve the periosteum. Mature HO shows cancellous bone and mature lamellar bone, vessels, and bone marrow marrow..

67C. What is the common classificat classification ion used for this disease process?  The Brooker classification classification2 is used and it is based on an anteroposterior radiograph. •





Class I: represents islands of bone in soft tissues around the hip. Class II: includes bone spurs in pelvis or proximal end of femur leaving at least 1 cm between the opposing bone surfaces. Class III: represents bone spurs that extend from pelvis or the proximal end of femur, which reduce the space between the opposing bone surfaces to less than 1 cm.

45



Class IV: indicates radiologic ankylosis of the hip.

67D. What can be done to reduce the incidence of this process? Non-steroidal anti-inflammatory drugs (NSAIDs) have been shown to reduce the incidence of HO.5 Pre-operative radiation has also been shown to prevent HO. Pakos et al 6 demonstrated the efficacy of combined radiotherapy

 

46

Chapter 4 Viva Answers

and indomethacin in preventing heterotopic ossification after total hip arthroplasty. Prophylactic Prophyl actic measures against HO after hip and knee replacement should be 7

administered before the fifth postoperative day, optimally within 24 to 48 hours.   Meticulous clearance of bone debris and avoidance of muscle damage have also been shown to prevent HO.8

67E. What are the consequences of the other abnormalit abnormality? y?  The other abnormality on the radiograph radiograph is leg length discrepancy. discrepancy. This can lead lead to a limp, pain in the other joints, potentially sciatic nerve injury and more likely legal consequences.

68A. Describe the radiogr radiograph. aph. What is the diagnosis? diagnosis?  The radiograph of the wrist shows evidence evidence of decreased joint space and subchondral sclerosis of the joints between the scaphoid, the trapezium and the trapezoid. The diagnosis is arthritis of the scapho-trapezio-trapezoid (STT) joint.

68B. What are the causes of this condition?  The causes of STT arthritis are age-related osteoarthritis, post-traumatic arthritis (including STT arthritis post-scaphoid ORIF), abnormal trapezio-trapezoidal inclination, capito-trapezial ligament laxity and rotatory subluxation.

68C. What is the treatment for for this condition? Non-operative treatment options include rest, splinting, hand therapy, oral antiinflammatory medication, and intra-articular steroid injection. Surgical options for ongoing symptoms will depend on the extent of symptoms, arthritis and an d the age and functional demands of the patient. The options include excision arthroplasty, interposition arthroplasty (using tendon or pyrocarbon implant) and arthrodesis. The latter is preferred in the young active patient as it is more likely to result in a stable thumb for high power manual work. Arthrodesis provides a stable column across the carpus and is a definitive operation to deal withpain at the level of the STT joint. It is contra-indicated with radio-scaphoid OA and has a variously quoted non-union rate of 4% to 29%. 9  Furthermore,, there is the risk of altering wrist kinematics permanently resulting Furthermore in secondary arthrosis of other joints such as the radiocarpal joint as well as 1st carpometacarpal joint osteoarthritis both of which have been reported in the

literature. Kirk-Watson however found no further OA at five years. A key to a literature. successful fusion is maintaining the relationship between individual carpal bones and correcting any secondary deformities. Some of the tips to a successful fusion include:10 •





Articular surface must be thoroughly denuded of cartilage Original external dimensions of the STT joint must be maintained Radial styloidectomy must be carried out to prevent impingement

 

Adult pathology

Excision of the distal end of the scaphoid is also a useful procedure in the management of symptomatic STT ST T OA for the following reasons:11 •







Relatively simple Allows flexion and extension of the scaphoid in radial/ulnar deviation therefore th erefore does not lead to impingement Can be combined with soft-tissue interposition Can lead to early mobilisation

However, excision of the distal However, di stal pole of the scaphoid can increase carpal instability and any pre-existing DISI (dorsal intercalated ligament instability) may be exaggerated.

68D. What are the possible complications of surgical intervention in patients with this condition? Surgical intervention may lead to a number of complications including ongoing pain, neurovascular injury, infection, thumb instability, deformity, pseudoarthrosis, carpal translation and progressive adjacent arthritis.

69A. Describe the abnormalities abnormalities seen seen in the radiograph. radiograph.  This is an example of lumbosacral transitional transitional vertebra (LSTV); (LSTV ); there is an abnormal joint between right lateral process and ileum/sacrum i leum/sacrum with evidence of sclerosis of this joint.

69B. What are the association associationss of this condition? Lumbosacral transitional vertebrae are associated with cervical ribs, altered nerve ner ve root functioning and facet joint arthrosis. These patients are at higher risk of disc degeneration and back pain.12

69C. What is the approximate incidence of this abnormality in the general population? A systematic review of studies from 200712 has shown a total prevalence of 7.5% sacralisation in the general population. There There is a prevalence of 5.5% for lumbarisation. The total for both is under 13%.

47

69D. What are the main surgical implications of this condition?  Tr  Treatment eatment of this condition is largely conservative conservative. . The Theanaesthetic majority ofinjection patients is are asymptomatic. If symptoms occur occur, , a steroid and local the first line treatment and can provide pain relief.  There is little data regarding regarding surgical interventions and surgery surgery may be indicated in radiculopathy, radiculopathy, degenerative disc disease higher up the spine and in certain chronic pain cases. Finally, failure to recognise this can lead to errors in lumbar numbering in surgery Finally, for other lumbar spinal problems.

 

48

Chapter 4 Viva Answers

69E. Describe the embryology of the spine. During the pre-cartilaginous stage sclerotomes appear as paired condensations around the notochord and neural tube in a four-week-old embryo. These sclerotomes consist of cells, which move cranially to form the intervertebral discs, and cells, which move caudally to form the primordial of the body of the vertebra. Where vertebral bodies surround the notochord it degenerates. Between the th e vertebrae the notochord expands to form the nucleus pulposus. Mesenchymal cells that surround the neural tube become the neural arch (primordium of the vertebral arch).  There are two primary ossification centres centres in the vertebrae – ventral and dorsal. dorsal. 13 Ossification becomes evident in the neural arches at eight weeks.

70A. Describe the radiogr radiograph. aph.  This is a lateral radiograph radiograph of the distal tibia/fibula and hindfoot/midfoot. It shows a talar fracture with extrusion of the talus anteriorly. The tibiotalar, tibiotalar, subtalar and talo-navicular joint are dislocated.

70B. There is an 8 cm clean laceration overlying the injury with a pale foot and no pulses. Describe your treatment strategy for this patient.  The mechanism of injury and the radiograph are consistent with an extremely extremely high energy injury and therefore I would manage this patient according to ATLS protocol, identifying life and then other limb-threatening limb -threatening injuries. If this is is deemed to be an isolated injury, the extruded talus and vascular compromise is a surgical would needsyndrome emergency in theatre. I would also assessemergency for signs ofand compartment andtreatment contact plastic surgical and vascular surgical teams to assess the patient and to be present in i n theatre. The fracture would be treated as any open fracture in the emergency department with intravenous antibiotics, tetanus cover cover,, irrigation if any gross contamination, pictorial documentation and temporary splintage. In theatre, a thorough debridement and washout of the laceration would be

performed by joint orthopaedic and plastic surgical teams and an open reduction of the extruded fragment performed. A vascular surgeon would be present to assess restoration of pulses post reduction and consideration of an on-table angiogram would be made. A negative pressure vacuum dressing would be applied to the open wound. Consideration could be made at the time for definitive fixation and soft-tissue coverage, but given the high energy injury and associated swelling, I feel a spanning external ex ternal fixator would be the treatment of choice. This would be combined with foot fasciotomies if indicated. At a later stage, further CT imaging of the injury would be performed to plan definitive fixation. Of note, if the extruded fragment was significantly contaminated and had no soft-tissue attachments, then consideration could be made to removal of the fragment and the use of a cement spacer, with a view to a later pantalar fusion. This is because of the high infection rates associated with retained contaminated, denuded extruded fragments.

 

Adult pathology

70C. What is the common common classification of talar fractures and what is the likely risk of nonunion in this case?

 Talarr frac  Tala fractur tures es can be cla classifi ssified ed ana anatom tomica ically lly int into o head head,, neck neck,, body body,, late lateral ral and posterior process fractures. The most common fracture is one of the talar neck, for which the Hawkins classification is prognostic. Delayed union rates of up to 15% have been noted but non-union is rare. The more common problem problem is that of avascular necrosis (AVN) (AVN) and this varies according to the Hawkins classification as follows. •







Hawkins I – undisplaced neck fracture – AVN risk 0 to 13% Hawkins II – displaced with subtalar subluxation/dislocation AVN 20 to 50% Hawkins III – displaced with subtalar and tibiotalar dislocation AVN 50 to 100% Hawkins IV – displaced with subtalar/tibiotalar/talonavicular dislocation 80 to 100%

70D. What is Hawkin’ Hawkin’ss sign and what is the pathogenesis of this phenomenon? Hawkin’s sign is the appearance of reduced subchondral bone density in the talar Hawkin’s dome six to eight weeks following injury. This This subchondral resorption indicates that vascularity to the talus is sufficient to allow normal resorption associated with disuse osteopaenia.

70E. Discuss the blood supply to the talus.  Talar  T alar blood supply is from various various sources: 1. Artery of the tarsal canal (dominant supply to body) – branch of posterior tibial artery 2. Deltoid branch of posterior tibial artery (may be only remaining supply supply and must be preserved during fixation) 3. Anterior tibial artery – supplies head and and neck  neck 

49

4. Perforating peroneal arteries via the artery of the tarsal sinus (head and neck) neck)

71A. Describe the radiogr radiograph aph  This is an AP radiograph of the right knee. It shows a valgus deformity deformity with loss of the lateral compartment joint space s pace and lateral compartment osteophytes, consistent with lateral compartment osteoarthritis of the knee.

71B. What other investigations are appropriate?

Other appropriate investigations would include a lateral radiograph of the knee k nee (AP and extension lateral weight-bearing radiographs), radiographs), flexion lateral (90° to 100°), Merchant’s Merchant’s view, and an AP radiograph of the pelvis if there were any concerns regarding hip osteoarthritis. Full length standing radiographs of the lower limbs including the hip and ankle joints may be appropriate if deformity is present, in order to determine the mechanical axis of the lower limbs. If considering an operation then routine blood group and save and an ECG would be required as a minimum.

 

50

Chapter 4 Viva Answers

71C. The patient has a significant fixed flexion deformity and fixed fixed valgus. Assuming surgical intervention is appropriate discuss what form of knee replacement would be appropriate? If the patient has a significant s ignificant fixed flexion deformity and fixed valgus this will require soft-tissue releases in order to correct the deformity. The type of knee implant required depends on the extent of the soft-tissue s oft-tissue release and the competence of the ligaments following the release. In the valgus knee the competence of the medial collateral ligament (MCL) is a factor that should be assessed pre-operatively, as well as whether or not the FFD and valgus are correctable. If the MCL and the lateral collateral ligament (LCL) are competent after the soft-tissue releases and bone cutsbe have made,oranposterior unconstrained TKR(PCL prosthesis may be used. This may PCLbeen retaining stabilised substituting). If the PCL is attenuated and/or if the joint line is significantly altered, a posterior stabilised design is recommended. If a PCL retaining implant is planned, then a PCL substituting option should always be available at the time of surgery. If the MCL is attenuated then a more constrained prosthesis will be required. If the LCL has to be released then, again, a more constrained prosthesis will be required. The next level of constraint is i s a constrained non-hinged prosthesis. This This implant has a large central post that th at substitutes for the MCL or LCL function.  The use of a constrained non-hinged prosthesis prosthesis for complete MCL deficiency is controversial. If the MCL is completely deficient, then all of the valgus forces are controversial. placed upon the polyethylene post leading to post breakage.

Any form of constrained knee prosthesis should use a long medullary stem for both the femoral and tibial components in order to reduce the stress experienced at the bone-cement-prosthesis interface.

71D. Describe a method of lateral ligament complex release and balancing the knee for valgus knees.  This may or may not not be approached approached via the lateral lateral parapat parapatellar ellar approac approach. h. Lateral Lateral release should begin with removal of the lateral osteophytes off the femur and tibia and release of the lateral capsule off the tibia. If the knee is tight laterally in extension the iliotibial band can be released either sub-periosteally off Gerdy’s tubercle, or by Z-lengthening or “pie-crusting”. The next structure to be released is depopliteus tendon, which is released sub-periosteally off the lateral condyle of the femur. In a large > 15° valgus deformity these releases will usually be necessary. If the knee still remains tight laterally the next step would be to release the LCL sub-periosteally off the lateral condyle of the femur. At this stage a constrained knee prosthesis should be considered and one should be aware of the possibility of stretching the peroneal nerve when correcting large valgus deformities.

 

Adult pathology

71E. What is is the importance of the possible medial medial joint joint line opening seen in the radiograph?  The significance of the possible medial joint line opening opening is that this could represent deficiency of the MCL. In this situation one should have the option of a constrained prosthesis available when it comes to surgery.

72A. Describe the radiogr radiograph. aph.  This is an anteroposterior radiograph radiograph of the right hip. It shows a peri-prosthetic peri-prosthetic femoral fracture around a cemented total hip replacement (tapered stem). The acetabular component is cemented and appears to be well fixed with acceptable inclination. There is a degree of heterotopic ossification. The femoral fracture involves the cement mantle and there is subsidence of the stem within the cement mantle, indicating that the stem is loose.

72B. Describe a classification system for for this injury that helps helps to guide guide management. Vancouver classification of peri-prosthetic femur fractures.14,15 The Vancouver Vancouver classification assimilates three key factors: fracture location, stability of the implant, and the surrounding bone stock (Table 4.1). Anatomical fracture location groups divides these fractures into three categories: Type A occurring around the trochanteric region, Type Type B near or just distal to the femoral stem, and Type C well

51

below the femoral stem. Studies have found that the Vancouver Vancouver classification is reliable, reproducible, reproducible, and valid. One limitation of the system is that plain radiographs may not always provide provide enough information to distinguish between type B1, type B2, and type B3 fractures. If there is any question pertaining to implant stability, it should be assessed intra-operatively intra-operatively.. Therefore Therefore pre-operative planning is essential. Appropriate positioning (supine versus lateral), availability of equipment for

Table 4.1 Type

Subtype

Description

Treatment

A

AL

Lesser trochanter

Conservative (consider ORIF if large segment of medial cortex involved)

AG

Greater trochanter

Conservative with abduction precautions (consider ORIF if displaced > 2.5 cm)

B1

Well-fixed prosthesis

ORIF with or without cortical strut allograft

B2

Prosthesis loose

Revision THR with long-stem prosthesis

B3

Prosthesis loose with poor bone stock 

Revision THR and augmentation of bone stock with allograft or oncologic prothesis

Fracture well below tip of the prosthesis

ORIF

B

C

 

52

Chapter 4 Viva Answers

fracture fixation, and revision femoral components is critical in allowing the surgeon options if the implant is found to be more or less stable than was interpreted pre-operatively pre-operatively..

72C. Where does this injur injury y fit with the classification system you have described?  This is a Vancouver Vancouver type B2. The fracture involves involves the femoral stem and the cement mantle is fractured with resulting subsidence of the prosthesis. The bone stock appears good, with no obvious deficiency/lysis.

72D. How would you treat this injur injury? y?

According to the Vancouver classification, I would perform a revision total hip replacement using a long-stem prosthesis. Prior to the operation I would ascertain the implant type and sizes of all of the components. I would have all the equipment and prostheses available to enable me to deal with any intra-operative eventuality. I would approach the hip through the old incision, curving this posteriorly at its proximal end to perform a posterior approach. I would open the fracture site and expose the cement mantle and stem using

an extended trochanteric osteotomy if necessary. I would remove as much cement as possible with an osteotome. Distally this could be done with a drill d rill and sequential reaming or by using an OSCAR cement removal system or osteotomes. Iuse would reconstitute the femoral canal with cerclage cables. I would then plan to a long stem modular, uncemented, tapered, fluted revision prosthesis, aiming for a good distal press-fit after af ter reaming the femoral canal. If this cannot be achieved, further options would be either a long cemented femoral stem or a distally locking stem. I would assess the acetabular component for wear and stability after removal of the femoral stem. If loose, or if there is evidence of polyethylene wear I would remove the acetabular component and cement. I would then assess the remaining bone stock and decide upon either a further cemented or uncemented acetabular component.

72E. What is the expected outcome following treatment?

Springer et al16 reported a survival of 90% at five years and 79.2% at ten years for Vancouver Vancouv er type B peri-prosthetic femoral fractures with revision or removal of the femoral implant for any reason as the end point. Those treated treated with uncemented, extensively porous-coated implants having the best results. Corten et al17  reported no revisions at 46 months follow-up after treatment of 31 B2 fractures with a cemented long-stem. Similarly, Fink et al18 reported all excellent results in 22 B2 and B3 fractures treated with a modular, uncemented, uncemented, tapered, fluted revision prosthesis. A review of 1049 peri-prosthetic fractures of the femur from the Swedish National Hip Arthroplasty Register found that the risk of failure of treatment was reduced for Vancouver Vancouver type B2 injuries if revision of the implant

 

Adult pathology

was undertaken or revision and open reduction and internal fixation were performed.19

73A. Describe the radiogr radiograph. aph.  There is a dislocation of the hip in a cemented cemented total hip replacement. replacement. There There is also an avulsion of the greater trochanter trochanter possibly due to non-union and broken and loose cerclage wires.

73B. What factors influence the risk of dislocation following a total hip replacement? Dislocation occurs in around 3.1% of primary total hip replacements and in 10% to 15% of revision total hip replacements. If occurring in the first year it is likely to be due to the surgical technique. Factors Factors associated with an increased risk can be divided into implant factors, patient factors fac tors and surgical factors. •

Patient factors include patients with neuromuscular problems, females,

53





alcoholism, THR for trauma, soft-tissue laxity, DDH, osteonecrosis, takedown of arthrodesis, age > 70 years, and trochanteric migrations. Implant factors include a reduced offset, smaller head size, wear of the polyethylene,, an inappropriate head neck ratio and loosening. polyethylene Surgical factors include malpositioning of the implant, soft-tissue, bony or cement impingement, a less experienced surgeon s urgeon and following a revision  THR.

Dislocations occurring after five years are classified as late dislocations. Von Knoch et al20 found that risk factors for this included component malposition, a younger patient age at index arthroplasty ar throplasty,, substantial trauma, wear of more than 2 mm and onset of cognitive or neurological decline. The late dislocation rate from the Mayo clinic was 0.8%.

73C. What are the options for management of this patient?  The patient would require require a reduction in the first instance. Following Following reduction definitive management could be conservative or surgical s urgical depending on the patient. Surgical management would include a revision of the total hip arthroplasty and I would consider using a larger head size and also a constraint if necessary to prevent further dislocations. The trochanter would need to be reattached with a trochanteric claw or a cobra plate.

73D. What is the prognosis if the patient underwent a revision for this problem?  There is a high rate of greater greater trochanter non-union in this case. Hamadouche et 21 al  in a study of 72 greater trochanter trochanter avulsions were able to achieve 51 osseous unions following claw plate fixation (70%).

 

54

Chapter 4 Viva Answers

73E. What are the options for treatment for for recurrent dislocation following followin g abductor insufficiency? Conversion to hemiarthroplasty with a larger femoral head which is indicated for soft-tissue deficiency or dysfunction. This is contraindicated if acetabular bone stock is compromised. Conversion to a constrained acetabular component. Indicated Ind icated for recurrent instability with a well-positioned acetabular component due to soft-tissue deficiency or dysfunction. Resection arthroplasty (Girdlestone procedure) may be undertaken when all options have been exhausted and there is a significant amount of bone loss and soft-tissue deficiency including for psychiatric patients who are dislocating for secondary gain.

74A. Describe the image.  This is a T2-weighted T2-weighted sagittal MRI of the thoracic spine. There There are multiple lesions lesions in the vertebral bodies with vertebral collapse of T4 and encroachment of the cord at T6 representing representing spinal cord compression. There is also an increase in the thoracic kyphosis.

74B. What is in thethe likely and how would you manage this patient A&Ediagnosis department?  The likely diagnosis is metastatic vertebral lesions lesions from breast cancer, cancer, now causing spinal cord compression. I would take a full history and examination, especially neurological examination. Analgesia would be offered and unless contraindicated a loading dose of at least 16 mg of dexamethasone would be given as soon as possible after assessment. This would be followed by a short course of 16 mg dexamethasone daily while treatment is being planned. I would also consider immobilisation of the spine if unstable and an urgent referral to the oncologists and the spinal surgeons. s urgeons.

74C. What are the options for for definitive management?

Management is guided by NICE guidelines. Conservative management will include radiotherapy and this may be suitable for those requiring definitive treatment or who are unsuitable for surgery. If surgery is appropriate in patients with (metastatic spinal cord compression) MSCC, attempt to achieve both spinal cord decompression and durable spinal column stability. stabilit y. Consider vertebral body reinforcement with cement for patients with MSCC and vertebral body involvement who are suitable s uitable for instrumented decompression but are expected to survive sur vive for less than one year year.. Consider vertebral body reconstruction with anterior bone graft for patients with w ith MSCC and vertebral body involvement who are suitable for instrumented decompression, are expected to survive for one year or longer and who are fit to undergo a more prolonged procedure.

 

Adult pathology

Table 4.2 Surgery more likely

Radiotherapy more likely

Prognosis

> 3 months

< 3 months

 Tumour burden

Low

High

Spine stability

Unstable spine

Stable

Diagnosis

New primary

Established disease

Levels of compression

One

Several

Performance Status

Good

Poor

55

 Time since onset of neurology

< 48 hours

> 48 hours

74D. What factors govern the decision making process? As mentioned above a number of factors influence decision-making including fitness for surgery and life expectancy. Surgical factors may include degree of kyphosis, surrounding bone quality and associated neurology. neurology. Vertebroplasty Vertebroplasty or kyphoplasty can be considered for vertebral metastases with no evidence of MSCC or spinal instability. i nstability.22

75A. Describe the radiographs.  These are anteroposterior anteroposterior (AP), lateral lateral and skyline radiographs of both knees.  The AP views show an osteophyte of of the left lateral femoral femoral condyle, with well preserved joint spaces in the medial and lateral compartments. There There are osteophytes of the tibial spines in both knees. In both knees the patella appears to be sitting more laterally than normal. On the lateral views there are signs consistent with osteoarthritis of the patellofemoral patellofemor al joints (PF ( PFJ), J), with osteophytes of the patellae and femoral condyles at the articular margins.  The skyline views show loss of the normal joint spaces, with lateral lateral patellofemoral patellofemoral  joint space narrowing, osteophytes, cysts and sclerosis. sclerosis. The The patellae are sitting laterally in the trochlear grooves and there is concave erosion of the lateral facets.

75B. What is the diagnosis and what is the aetiology?  The diagnosis is patellofemoral patellofemoral osteoarthritis. Aetiology Factorss influencing the magnitude and distribution of PF Factor PFJJ pressure have important implications. The PFJ PFJ reaction force can reach more than three times body weight during stair ascent and descent and seven to eight times body weight during squatting activities. PFJ malalignment leads to increased contact pressure on an individual facet PFJ (lateral tilt of the patella leads to increased contact pressure on the lateral facet).

 

56

Chapter 4 Viva Answers

Factorss affecting alignment include: Factor •





Passive structures - osseous anatomy and soft-tissue constraints. Passive Active structures - medial (vastus medialis obliquus (VMO)) and lateral quadriceps (vastus lateralis (VL)). Overall lower limb alignment.

Osseous abnormalities include a shallow or dysplastic femoral trochlea or patella

alta. Soft-tissue abnormalities include an incompetent medial patello-femoral ligament (MPFL) or a tight lateral retinaculum, particularly the two distal expansions of the iliotibial band. Younger patients with PF PFJJ pain have demonstrated a delayed onset ons et of VMO activity relative to the VL during a monosynaptic reflex (patellar tap), during isokinetic knee extension ex tension and during a stair-stepping task. People with generalised knee OA do not n ot exhibit a delayed onset of VMO relative to VL. VL. Lower limb alignment may affect patellar tracking by altering the relative position of the femoral trochlea and changing the tension in soft-tissues. Experimentally induced femoral internal rotation (anteversion) or tibial external rotation have been associated with increased lateral patellar tilt and rotation and increased lateral PFJ pressure. Valgus malalignment is associated with isolated moderate to severe PFJ OA, and is associated with an increased risk of disease progression. Experimental increases in the Q-angle shifts the PFJ PFJ contact area laterally and further increases pressures within the lateral facet. Increases in the lateral compartment contact pressures correlate with the pattern of PF PFJJ osteoarthritis (OA), in which the lateral compartment is most commonly affected. Knees with a more laterally positioned patella and greater patella tilt laterally demonstrate a higher prevalence of PFJ OA. 23 Work by the same authors in Caucasian and African-American people shows that patella subluxation is 24

associated with knee pain severity and risk of disease progression. Significantly greater lateral tilt of the patella is seen in patients with isolated PFJ PFJ OA compared with those with concurrent tibiofemoral tibiofemoral joint (TF ( TFJ) J) OA. Macro/Microtrauma 28% of patients with isolated PFJ PFJ OA reported a history histor y of patellar dislocation/ subluxation, compared with none of the patients with combined PFJ/TF PFJ/TFJJ 25 OA,  suggesting that a history of patella dislocation/subluxation may be a predisposing factor for PFJ OA. Obesity has been associated with PF PFJJ OA.26  There is no definite evidence of gender preponderance.27,28

 

Adult pathology

75C. What are the options for management in this situation? Management options are broadly non-operative or operative: Non-operative

57

 These should always be recommended recommended as the first line of treatment: •











Activity modification Weight loss Physiotherapy; core stability, posture, VMO strengthening, taping Analgesia/anti-inflammatory medications Intra-articular injections – corticosteroid or hyaluronic acid Other preparations, including glucosamine, chondroitin and diacetylrhein

Operative May consist of one or a combination of the following: Arthroscopic debridement/washout/chondroplasty/microfr debridement/washout/chondroplasty/microfracture/ acture/ osteochondral allograft Partial lateral facetectomy Extensor mechanism realignment procedures (Fulkerson/Maquet (Fulkerson/Maquet procedures)/ lateral release Patellofemoral arthroplasty (PFA)  Total  T otal knee arthroplasty (TKA)











75D. What factors govern the decision making process? Factorss governing the management include: Factor Patient factors – general: Age and activity levels Patient demands and expectations BMI Co-morbidities Occupation











Patient factors – disease-specific: Degree of cartilage loss/bone erosion Patellar alignment – tilt/height/soft-tissue stabilisers and quadriceps function/ tibial tubercle trochlea distance (TTTD) Limb alignment Presence of trochlear dysplasia  Tibiofemoral joint disease











Surgeon factors: Experience with various patellofemoral procedures



75E. If you you decided to go down the arthrop arthroplasty lasty route, what prosthesis would you use and why? In this case the patient is relatively young and female. We We have no information on the general patient factors listed in the previous answer.

 

58

Chapter 4 Viva Answers

Arthroplasty options are TKA or PF PFA: A:

Total knee arthroplasty Because of its relationship with patellofemoral instability, total knee arthroplasty in patients with isolated patellofemoral osteoarthritis is a technically more demanding procedure. A systematic review of treatments for isolated patellofemoral patellofemor al osteoarthritis in 2010 found low quality, weak evidence for the use of TKA.29 Furthermore, there is little information in the literature on patellar resurfacing in TKA for this indication. TKA with patellar resurfacing gives satisfactory five to seven year results. However, However, anterior knee pain can be problematic after TKA. Patellofemoral arthroplasty Patellofemoral  The same systematic review reported reported that the clinical results are related related to prosthetic design, surgical technique, patient selection and indication, ind ication, and length of follow-up, and have shown good to excellent three- to 17-year results in two-thirds to all patients.29 Progression of femorotibial osteoarthritis, malposition of the prosthesis, and Progression wear or loosening may result in failure of the patellofemoral arthroplasty. It was also reported that patellofemor patellofemoral al arthroplasty does not have a negative effect on the outcome of later total knee arthroplasty. Poor prognostic markers include evidence of tibiofemoral osteoarthritis prior to surgery, BMI > 30, Prior meniscectomy,, patella alta or baja, ligamentous instability.30 meniscectomy I would perform a patellofemor patellofemoral al arthroplasty in this patient (Avon implant).  TKA with patellar resurfacing results in predictable and durable durable good results. However,, for osteoarthritis involving only one compartment, it may be too However aggressive, especially in this relatively young patient. The reported outcome after patellofemoral arthroplasty in selected patients is good to excellent in the literature.. Total literature Total knee replacement can be performed per formed later if painful femorotibial osteoarthritis develops.

75F.. How are patellofemoral joint replacements performing on the 75F National Joint Registry in the UK? In the 9th annual report of the national joint registry (NJR), 2012 31 the median age of patients undergoing patellofemoral arthroplasty is lower than for other knee arthroplasty ar throplasty operations. The The eight-year revision rate was 14.7% for patellofemoral patellofemor al knee replacements, considerably higher than TKA. They had the highest three-year, five-year and eight-year revision rates. In comparison, at eight years the all-cemented TKA revision rate was 2.82%. Pain and asceptic loosening were the most common reasons for revision. The Avon patellofemoral implant has the best record – 10.4% revision rate at seven years.

 

References

59

References 1.

King HA, Moe JH, Bradford DS,Winter RB. RB. The selection of fusion levels in thoracic idiopathic idiopathic scoliosis. J Bone Joint Surg [Am] 1983;65:1302-13. 2. Brooker AF AF, Bowerman JW, Robinson RA, Riley LH Jr. Ectopic ossification ossification following following total hip replacement: incidence and a method of classification. J Bone Joint Surg [Am] 1973;55-A:1629-32. 3. Chalmers J, Gray DH, Rush J. Observations on the induction of bone in soft tissues. J Bone Joint Surg [Br] 1975;57:36-45. 4. Garland DE. A clinical perspective on common forms of acquired heterotopic ossification. Clin Orthop Relat Res 1991;263:13-29. 5. Fransen M, Neal B, Cameron ID, et al. al. Determinants of heterotopic heterotopic ossification after total hip replacement surgery. Hip Int 2009;19:41-6. 6. Pakos EE, Tsekeris PG, Paschos NK, et al. The role of of radiation dose in a combined therapeutic protocol for the prevention of heterotopic ossification after total hip replacement. J BUON 2010;15:74-8. 7. Iorio R, Healy WL. Heterotopic ossification after hip and knee arthroplasty: risk factors, prevention, and treatment. J Am Acad Orthop Surg 2002;10:409-16. 8. Pavlou G, Salhab M, Murugesan L, et al. Risk factors for heterotopic ossification in in primary total hip arthroplasty. Hip Int 2012;22:50-5. 9. Rogers WD, Watson HK. Degenerative arthritis at the triscaphe joint. J Hand Surg Am 1990;15:232-5. 10. Watson HK, Hempton RF RF.. Limited wrist arthrodesis. I: the triscaphoid joint. J Hand Surg 1980;5:320-7. 11. Weiss KE, Rodner CM. Osteoarthritis of the wrist. J Hand Hand Surg Am 2007;32:725-46. 12. Bron JL, van Royen BJ, Wuisman PI. The clinical significance of lumbosacral transitional anomalies. Acta Orthop Belg 2007;73:687-95. 13. Moore KL, Persaud Persaud TVN, Torchia MG. Before Before we are born: essentials of embryology and birth defects. 7th ed. Philadelphia: Elsevier Health Sciences, 2007. 14. Duncan CP, CP, Masri BA. Fractures Fractures of the femur after hip replacement. Instr Course Lect 1995;44: 293-304. 15. Masri BA, Meek RM, Duncan CP. CP. Periprosthetic fractures evaluation and treatment. Clin Orthop Relat Res 2004;420:80-95. 16. Springer BD, Berry DJ, Lewallen DG. Treatment Treatment of periprosthetic periprosthetic femoral fractures following total total hip arthroplasty with femoral component revision. J Bone Joint Surg [Am] 2003;85-A:2156-62. 17. Corten K, Macdonald SJ, McCalden RW, RW, Bourne RB, Naudie DD. Results of cemented femoral revisions for periprosthetic femoral fractures in the elderly. J Arthroplasty 2012;27:220-5. 18. Fink B, B, Grossmann A, Singer J. Hip revision arthroplasty in periprosthetic fractures of vancouver type B2 and B3. J Orthop Trauma 2012;26:206-11. 19. Lindahl H, Malchau H, Odén A, Garellick G. Risk factors for failure after treatment of a periprosthetic fracture of the femur. J Bone Joint Surg [Br] 2006;88-B:26-30. 20. von Knoch M, Berry DJ, Harmsen WS, Morrey BF. BF. Late dislocation dislocation after total hip hip arthroplasty. J Bone Joint Surg [Am] 2002;84-A:1949-53. 21. Hamadouche M, Zniber B, Dumaine V, Kerboull M, Courpied JP. JP. Reattachment of the ununited greater trochanter following total hip arthroplasty: the use of a trochanteric claw plate. J Bone Joint Surg [Am] 2003;85-A:1330-7. 22. No authors listed. listed. NHS National Institution for for Health and Clinical Excellence. Metastatic spinal cord compression: diagnosis and management of adults at risk of and with metastatic spinal cord compression, CG75, 2008. http://www.nice.org http://w ww.nice.org.uk/CG75 .uk/CG75 (date last accessed 5 March 2012). 23. Niu J, Zhang Y, Nevitt M, et al. Patella malalignment is associated with prevalent patellofemoral patellofemoral osteoarthritis: the Beijing Osteoarthritis Study. Arthritis Rheum 2005;52:S456-7. 24. Hunter Zhang2005;52:S686. Y, Niu J, et al. Patella Patella malalignment and its consequences: consequences: the Health ABC Study. ArthritisD,Rheum 25. Iwano T, T, Kurosawa H, Tokuyama H, Hoshikawa Y. Y. Roentographic and clinical findings of patellofemoral arthritis. Clin Orthop Relat Res 1990;252:190-7. 26. Cicuttini FM, Baker JR, Spector TD. The association of obesity with osteoarthritis osteoarthritis of the hand and knee in women: a twin study. J Rheumatol 1996;23:1221-6.

 

60

Chapter 4 Viva Answers

27. McAlindon TE, Snow S, Cooper C, Dieppe PA. PA. Radiographic patterns of osteoarthritis osteoarthritis of the knee in the community: the importance of the patellofemoral joint. Ann Rheum Dis 1992;51:844-9. 28. Davies AP, AP, Vince Vince AS, Shepstone L, Donell ST, ST, Glasgow MM. The radiological prevalence prevalence of patellofemoral osteoarthritis. Clin Orthop Relat Res 2002;402:206-12. 29. van Jonbergen HP, Poolman RW, RW, van Kampen A. Isolated Isolated patellofemoral osteoarthritis. Acta Orthop 2010;81:199-205. 30. Tarassoli P, P, Punwar S, Khan W, Johnstone D. Patellofemoral Patellofemoral arthroplasty: a systematic review of the literature. Open Orthop J 2012;6:340-7. 31. No authors listed. listed. National Joint Registry for England and Wales. http://www.njrcentre.org.uk/ http://www.njrcentre.org.uk/ (date last accessed 7 March 2013).

 

Chapter 5 Viva Questions: Trauma 70-year-old man fell on his arm while working in his garden and presented to 76. A 70-year-old  A&E with the following injury (Figs 5.1a and b).  A. Describe the findings on the radiographs. radiographs. B.  What else would would you like to know know about the pre-injury pre-injury status of the patient? C.  What are are the principles of management management of such an injury? are the pitfalls of the the proposed modalities modalities of treatment? treatment? D.  What are E.  What ar are e the absolute indications indications for operative operative treatment treatment in such an injury?

a

b

Figure 5.1a and b

 

62

Chapter 5 Viva Questions

77. A 74-year-old 74-year-old female living alone in a warden-controlled home, tripped and fell at home injuring her right leg. She was unable to weight-bear and was brought into A&E. Tis is a radiograph of her pelvis (Fig. 5.2).  A. Describe the radiograph radiograph.. B.  What are are the classification classification systems that that are commonly commonly used to describe this fracture fractur e and which system allows for recommendat recommendation ion of treatment based on the type of fractur fracture? e? C.  What are are the principles of management management of such an injury? D. If you decided to proceed with operative intervention, what kind of implant  would you use and why? E.  What drug has has been recently implicated in the the treatment treatment of these fractures fractures and  what are are the classical classical radiological features features to suggest suggest its usage?

Figure 5.2

78. A 48-year-old 48-year-old right hand dominant dominant civil servant presented to the fracture fracture clinic following a fall from a horse on his right arm. Tis is the radiograph obtained in  A&E (Fig. 5.3) .  A. Describe the abnormality on the radiograph radiograph.. the classification system commonly used for this injury and how how would B.  What is the  you classify this this fracture? fracture?

C.  What are are the ligaments ligaments attached to the distal end of clavicle and how do they influence displacement at the fracture site? D.  Which type of fracture has the highest highest rate of nonunion? nonunion? E. How would you treat this injury?

 

Trauma

Figure 5.3

79. A 25-year-old 25-year-old man sustained this injury after a falling from his motorbike (Figs 5.4a and b).  A. Describe the abnormalities in the radiographs. radiographs. B. How would you initially manage this patient? C.  Wha  Whatt classific classification ation syste system m would would you you use use to descr describe ibe this this injury? injury? Please Please expla explain. in. D. How will you manage this injury definitively? E.  What ar are e the Ruedi–Allgower Ruedi–Allgower principles of operative fixation fixation for the management of these fractures?

b

 

63

a

Figure 5.4a and b

 

64

Chapter 5 Viva Questions

80. An 18-year-old 18-year-old male presents presents with an acutely swollen knee after an incident incident  whilst football training training during which he was tackled tackled from the side. Tese are the radiographs obtained in A&E (Figs 5.5a and b).  A. Describe the radiographs. What is your diagnosis? B. How would you investigate this patient further? C.  What is the classification of this injury? D.  What would your management management be? What What are the impediments impediments to successful successful closed reduction? E. If you had to treat this patient operatively, what would be the post-operative managementt and what would you specifically warn the patient about prior to managemen surgery?

a

b

Figure 5.5a and b

81. A 46-year-old 46-year-old patient was brought into A&E following a fall from a horse. Te patient is haemodynamically stable and this is the radiograph of the pelvis (Fig. 5.6).

 A. Describe the abnormalities on the radiograph. radiograph. B.  What is the possible mechanism of this injury and how would you classify  pelvic fractures in general? C.  Where would this injury fit in your classification? classification? D.  What is the risk of urological injury or impotence in patients patients with such injuries? E.  What are are the radiological radiological signs of rotational rotational and vertical instability instability in pelvic fractures? F. How would you manage this patient?

 

Trauma

Figure 5.6

82. A 26-year-old 26-year-old motorcyclist was involved in a road traffic accident at a speed of 50 mph and sustained this injury (Figs 5.7a and b).  A. How would you classify this injury? B. How do you classify soft-tissue injuries with closed fractures? C. How would you manage this injury, assuming that the initial ATLS guidelines have been followed and the patient is stable? D.  What decision making making tools are available for for predicting limb limb salvage after high-energy trauma to the lower extremity? Describe them in detail. E.  What is the evidence to support the use of these tools?

 

65

a

b

Figure 5.7a and b

 

66

Chapter 5 Viva Questions

83. A 43-year-old 43-year-old bricklayer presents presents to A&E with the following injury to his left shoulder (Fig. 5.8).  A. Describe the radiograph radiograph.. B.  What are are the deforming forces forces involved with a fracture fracture of this type? C. How would you define nonunion of the clavicle and what is the rate of nonunion with diaphyseal fractures fractures of the clavicle? D.  What are are the risk factors for for nonunion and which is the most predictive? predictive? E.  What is the rate of dissatisfaction dissatisfaction following these these fractures fractures and what are the possible problems with malunion?

Figure 5.8

84. A 27-year-old 27-year-old man fell from a bike onto his right shoulder and presented  with the following isolated injury to A&E. Tis is the radiograph radiograph obtained on admission (Fig. 5.9).  A. Describe the radiograph radiograph.. B.  What other investigation/s would you request? C.  What are are the other injuries are are associated with this injury? D.  What is the classification system system used for describing these these fractures? fractures? E.  What are are the indications for surgery for these fractures fractures and what what approach approach  would you use if you you decided to proceed proceed with operative intervention?

Figure 5.9

 

Trauma

85. A six-year-old six-year-old boy presents after after a fall on his right arm with the following radiographs (Figs 5.10a to c).  A. Describe the radiographs. What are the critical lines used to assess in these radiographs? B.  What is the commonly used classification classification of these injuries? injuries? C.  What are are Monteggia equivalent equivalent fractures? fractures? D.  What would your management management of this injury be and what is the most most common complication? E.  What are are the reasons for for inability to achieve anatomical reduction? reduction? F. How would you treat a chronic post-traumatic dislocation of the radial head in a child?

a

b

 

67

c

Figure 5.10a to c

 

68

Chapter 5 Viva Questions

86. A 42-year-old 42-year-old woman presented with a fall onto her dominant right hand with with a painful and stiff elbow. Tese are her radiographs (Figs 5.11a and b).  A. Describe the radiographs radiographs.. B.  What is the pattern of injury injury in these fractures? fractures? C.  Which gender is this injury most commonly seen in and why? D. How would you classify this injury? E. How would you treat it and what approach will you use? F.  What are are the complications? complications?

a

b

Figure 5.11a and b

 

Trauma

87. A 17-year-old 17-year-old footballer presented with a sudden onset of pain in his right groin  while trying to kick the ball. Tese are the radiographs radiographs obtained in A&E (Figs 5.12a and b).  A. Describe the radiographs radiographs.. B.  What is the diagnosis? C.  Wha  Whatt is the mech mechanis anism m of inj injury ury and and how does it reflec reflectt upon upon the type of frac fractur ture? e? D.  What is the common age group group of presentation presentation for these injuries? injuries? E.  What is the prognosis and how how soon can the patient patient expect to return return to sport?

69

a

Figure 5.12a and b

b

 

Chapter 6 Viva Answers: Trauma 76A. Describe the findings on the radiographs.  There is a comminuted fracture fracture of the midshaft of the right humerus. There

is medial translation of the distal fragment and the bone appears osteolytic, particularly at the fracture site. si te. There There are also significant osteoarthritic changes at the glenohumeral joint.

76B. What else would would you you like to know about the pre-injury status of the patient? I would be concerned about a pathological fracture, possibly due to secondary metastases from a tumour, and thus would like to know about any constitutional symptoms (weight loss, fever etc) as well as any specific symptoms relating to any possible malignancy if such a diagnosis had not been made previously. I would also like to know about shoulder pain and function prior to the injury in view of significant glenohumeral OA.

76C. What are the principles of management management of such an injury? Obtaining length, alignment and rotational alignment, and relative stability of the fracture with concomitant treatment of the shoulder pathology would be the main principles behind the management of this injury. A long stem hemi-arthroplasty or a shoulder replacement should be considered.

76D. What are the pitfalls of the proposed modalities of treatment? Pitfalls of 1. Non Non-ope -opera rativ tive e treat treatmen mentt – non-u non-unio nion n 2. Intra Intramedull medullary ary nailing nailing – obtainin obtaining g access access to perform perform the the nailing nailing with with high risk  risk  of iatrogenic causing a proximal humeral fracture 3. Plat Plate e fixation fixation – shoul shoulder der patholo pathology gy will will be left untre untreated ated

76E. What are the absolute indications for operative treatment in such an injury? Open fracture and vascular injury are absolute indications. Nerve injury may be a relative indication for surgical management.1

 

72

Chapter 6 Viva Answers

77A. Describe the radiogr radiograph. aph.  There is a 4-part subtrochanteric fracture fracture of the right femur. femur. There There is medial displacement of the shaft and lesser trochanter and the split through the greater trochanter and involvement of the piriformis fossa extending into the lateral wall down to the fracture site. This is a Seinsheimer type V fracture.

77B. What are the classificat classification ion systems that are commonly used to describe this fracture and which system allows for

recommendation recomm endation of treatment based on the type of fracture? Seinsheimer, Fielding’s, AO and Russell-Taylor are classification systems. Seinsheimer2 1. undisp undisplaced laced fract fracture ure with less than 2 cm displac displacement ement 2. 2 par partt fr frac actu turre 2a. Transverse fracture 2b. Spiral Spiral configuration with the lesser trochanter attached to proximal fragment 2c. Spiral configuration with the lesser trochanter attached to distal fragment 3. 3 par partt fr frac actu turre 3a. Three-part spiral configuration with the lesser trochanter a part of the third fragment 3b. Three-part spiral configuration with the third part a butterfly fragment 4. Comm Comminuted inuted fract fracture ure with four or more more frag fragments ments 5. Subtr Subtrochan ochanteric teric-inte -intertroch rtrochanter anteric ic config configurat uration ion Fielding’s3 1. fractu fracture re is at the level level of the the lesser lesser troc trochante hanterr 2. fractu fracture re is 2.5 2.5 to 5 cm belo below w the lesser troc trochante hanterr 3. fractu fracture re is 5 to 7.5 7.5 cm cm below below the lesser lesser troc trochante hanterr Russell-Taylor4 (based on piriformis fossa involvement) 1. fra fractur cture e does does not extend extend into into pirif piriform ormis is fossa fossa 1a. comminution and fracture lines extend from below lesser trochanter to femoral isthmus 1b. fracture lines and comminution involve area of lesser trochanter to isthmus 2. fractu fracture re extends extends proximall proximallyy into greater greater trochan trochanter ter and involve involvess piriformis piriformis fossa 2a. no significant comminution or fracture of lesser trochanter 2b. significant comminution of medial femoral cortex and loss of continuity of lesser trochanter Of these classifications the Russell-Taylor Russell-Taylor system is most useful us eful as treatment can be tailored specifically for different fracture configurations. If the piriformis fossa is intact then an intramedullary nail can normally be utilised and if the piriformis fossa is fractured then a fixed angle hip screw device may be preferred.

77C. What are are the principles of management management of such an injury? Initial management of such an injury in the emergency department would include a full ATLS work up including resuscitation and analgesia. The fracture

 

Trauma

is best treated with operative intervention, given anaesthetic suitability, to ease nursing care, enable a return to mobility and to reduce future medical complications. Informed consent would then be obtained. Pre-operative optimisation of the patient status would include orthogeriatric and anaesthetic input. Regional anaesthesia (spinal) could be used in the event of significant cardio-respiratory co-morbidity. Post-operative management would centre on early mobilisation, nutrition, fluid balance and DVT prophlyaxis.

73

Surgical principles would be to obtain indirect reduction and restoration of length, alignment and rotation and stabilisation s tabilisation using principles of relative stability.

77D. If you decided to proceed with operative intervention, what kind of implant would you use and why?  The Russell-T Russell-Taylor classification would recommend recommend a fixed angle device such as a DCS given that the piriformis fossa is involved and the entry point for a nail is comminuted. The key issue for this fracture is to attempt to restore the anatomy prior to fixation and therefore one should have a low threshold to perform per form an open reduction with cabling/wiring of the subtrochanteric element. Modern cepahlomedullary long intramedullary nails with distal locking can also be used and would be my implant of choice.

77E. What drug has been recently implicated in the treatment of these fractures and what are the classical radiological features to suggest its usage? Bisphosphonates have been recently associated with subtrochanteric fractures.  This is typically a transverse fracture in an area area of thickened cortices, following following low energy trauma. Before the fracture a conical ridge at the lateral aspect of the femur in the subtrochanteric area can be seen.5

78A. Describe the abnormality abnormality on the radiogr radiograph. aph.  The radiograph shows shows a vertically displaced, comminuted fracture fracture of the lateral third of the clavicle.

78B. What is the classificat classification ion system commonly used for this injury and how would you classify this fracture?  The Rockwood and Green classification6 of lateral clavicle fractures: •







 Type I – fracture fracture lateral to the coracoclavicluar coracoclavicluar ligaments  Type II – fracture medial to the coracoclavicluar coracoclavicluar ligaments ligaments  Type III – injury extends into the ACJ  Type IV – Paediatric periosteal sleeve sleeve injury

 Type V – comminuted fracture  The radiograph shows shows a Type Type V  V fracture. •

 

74

Chapter 6 Viva Answers

78C. What are the ligaments attached to the distal end of clavicle and how do they influence displacement at the fracture site?  The ligaments attached to the distal end of the clavicle clavicle are the acromioclavicular acromioclavicular

and coracoclavicular ligaments. The acromioclavicular ligament provides anterior/posterior stability. It has superior, inferior, anterior, anterior, and posterior components, with the superior being the strongest. This This is why the distal fragment in a lateral clavicle fracture is held in place adjacent to the acromion.  The coracoclavicular coracoclavicular ligaments (trapezoid (trapezoid and conoid) provide vertical stability. stability.  The trapezoid ligament inserts 3 cm from end of clavicle clavicle and the conoid ligament inserts 4.5 cm from end of clavicle in the posterior border. border. The conoid ligament is strongest. Rupture of these ligaments leads to significant vertical displacement of the clavicle fracture as seen in the radiograph.

78D. Which type of fracture has the highest rate of nonunion? Robinson et al7 found that displacement of the fracture and increased age of patient were associated with a higher risk of non-union in lateral clavicle fractures. These These fractures in themselves have an increased rate of non-union, when compared to middle or medial third fractures.

78E. How would you treat this injur injury? y? A fall from a horse should be considered a high-energy injury and I would therefore manage the patient according to ATLS protocol. Assuming this is an isolated injury, I would perform a history and a thorough examination with special reference to neurovascular neurovascular status also ensuring that this was a closed injury. Using Robinson’s Robinson’s prognostic index I would explain to the patient that there was a 30% chance of nonunion at six months and that treatment could be operative or non-operative. Non-operative treatment would eliminate surgical risk. Indeed Robinson8 advocated non-operative management in middle-aged individuals with displaced di splaced lateral clavicle fractures: Nonoperative treatment treatment of most displaced lateral fractures of the clavicle in middle-aged and elderly patients achieved a good mediumterm functional result. Symptoms that were severe enough to warrant a delayed reconstructive procedure procedure developed in only 14% of the patients. Asymptomatic nonunion does not appear to adversely affect the functional outcome in the medium term.8 I would therefore advocate non-operative management. However, However, operative intervention may lead to a quicker improvement in function and I would discuss this with the patient. Surgical options could include a hook plate or reconstruction of the coracocla coracoclavicular vicular ligaments.

79A. Describe the abnormaliti abnormalities es in the radiographs.

 There is a comminuted distal tibial fracture fracture with intra-articular extension (pilon fracture). Moderately displaced transverse distal diaphyseal fibular fracture. Both fractures show varus angulation.

 

Trauma

75

79B. How would you initially manage this patient? Initial management would involve an advanced trauma life support (ATLS) approach to rule out any life threatening injuries. Following this, the limb would be assessed and neurovascular status documented. If this was an open injury, intravenous antibiotics would be administered, tetanus prophylaxis given, a photograph would be taken and a preliminary lavage and betadine dressing would be applied.  This injury injur y represents a significant si gnificant soft-tissue soft-t issue insult ins ult and this must be considered consi dered prior to any definitive fixation. The injury would be splinted temporarily in and above knee back slab and temporary external fixation (+/- debridement +/- plastic surgical consultation) would then be undertaken in the operating theatre. A CT scan would be ideal pre-operatively pre -operatively to plan definitive fixation.

79C. What classification classification system would would you use to describe describe this injury? Please explain. Ruedi–Allgower classification of pilon fractures:  Type 1 - Mild to moderate moderate displacement and no comminution comminution without major disruption of ankle joint  Type 2 - Moderate Moderate displacement and no comminution without significant significant dislocation of ankle joint  Type 3 - Explosion Explosion fracture with severe comminution comminution and displacement







79D. How will you manage this injur injury y definitively? Definitive management would involve a CT scan to accurately assess the degree of intra-articular involvement and plan the surgical approach to the distal tibia.  The preferred option to restore arti articular cular congruity con gruity would be open reduction red uction and internal fixation of the distal tibia once the soft tissue insult has been minimised. An anterolateral or anteromedial approach approach could be utilised depending on exact fracture pattern on the CT scan. Options for fixation would be ORIF, ORIF, External fixation (bridging or non-bridging), and combination of internal and external fixation or percutaneous plating.

79E. What are the Ruedi–Allgower principles of operative fixation for the management of these fractures?  The best functional results in the past series were observed in patients treated treated according to the following four sequential principles:9  1. Reconstruction of the correct length of of the fibula 2. Anatomical reconstruction of the articular surface of the tibia 3. Insertion of a cancellous autograft to to fill gaps left by impaction and comminution

 

76

Chapter 6 Viva Answers

4. Stable Stable internal internal fixation fixation of the the fragments fragments by by a plate plate placed placed on the medial medial aspect of the tibia.

80A. Describe the radiographs. What is your diagnosis?  The radiographs demonstrate demonstrate a lipohaemarthrosis, a Segond fracture and an avulsion fracture of the tibial spine. Originally described by the French surgeon Paul Segond in 1879 after a series of cadaveric experiments, the Segond fracture describes a cortical avulsion of the tibial insertion inser tion of the middle third of the lateral capsular ligament. It is commonly associated with injury to the anterior cruciate ligament.

80B. How would you investigate this patient furt further? her? I would perform a full history and examination and request an MRI scan of the knee, to confirm an ACL injury and to identify any other injuries within the knee k nee to the menisci, collateral ligaments and to the PCL.

80C. What is the the classification classification of this this injury? injury?  The Myers and McKeever10 classification for fractures of the intercondylar eminence is as follows: •







 Type I: Undisplaced fracture  Type II: Partially displaced fracture with anterior anterior elevation of the eminence;  Type III A: Completely displaced displaced fracture with no contact between the eminence and proximal end of tibia  Type III B: The eminence eminence is rotated as well as displaced displaced

80D. What would your management be? What are the impediments to a successful closed reduction?  The radiographs show a Type II injury. injury. This This could be managed with a trial of closed reduction under anaesthetic. A block to full extension may be caused by interposition of the anterior horns of either the medial or the lateral meniscus. If full extension is not possible, reduction is not complete. This would necessitate an arthroscopic assisted/open reduction and internal fixation of the tibial spine.

80E. If you had to treat this patient operatively, what would would be the post-operative management and what would you specifically warn the patient about prior to surgery? Post-operatively, Post-operative ly, I would immobilise the knee in extension for six s ix to eight weeks. At stage,two a knee k nee brace would be employed, gradually increasingas flexion by that 30° every t wo weeks. Closed chain exercises would be advocated per a standard ACL rehabilitation protocol protocol.. I would warn the patient about the usual risks ris ks of surgery, but more specifically, the risk of ACL laxity upon fracture healing and also arthofibrosis ar thofibrosis with resultant stiffness of the knee.

 

Trauma

81A. Describe the abnormalities abnormalities on the the radiograph radiograph..  There is a significant diastasis of the symphysis pubis and significant widening of the right sacro-iliac (SI) joint.

81B. What is the possible mechanism of this injury and how would you you classify pelvic fractures in general?  This is likely to be an antero-posterior antero-posterior compression injury. This injury can be 11 classified by the Tile  or Young and Burgess12 classification systems.  The Tile Tile classification includes Type A (stable), (stable), Type Type B (rotationally unstable but vertically stable) and Type C (rotationally and vertically unstable).  The Young and Burgess system is based on mechanism of injury, Type A (lateral (lateral compression), Type Type B (AP compression) and Type C (vertical shear). Within the AP compression (APC) type there are three further subdivisions: APC-I •





Slight widening of pubic symphysis/anterior SI joint Intact anterior SI, sacrotubero s acrotuberous us and sacrospinous ligaments Intact posterior SI ligaments

APC II Widened anterior SI joint; disrupted anterior SI, sacrotuberous and sacrospinous ligaments Intact posterior SI ligaments





APC III Complete SI joint disruption with lateral displacement Disrupted anterior SI, sacrotuberous and sacrospinous ligament







Disrupted posterior SI ligaments

81C. Where would this injury injury fit in in your your classification? classification?  This is therefore a Type B, B, APC II injury. More specifically this is an open book pelvis injury. As the posterior SI joint is intact it acts as a hinge from which the displaced anterior SI joint and the right hemipelvis can externally ex ternally rotate.

81D. What is the risk of urological injury or impotence in patients with such injuries? A recent study (Pavelka et al) showed that primary injury to the urological tract was recorded in 13.5% of patients with pelvic fractures. Injury to the urethra was found in 7.5% and urinary bladder trauma in 6%. Injury to the urological tract was associated with a Type Type A pelvic ring fracture in in 5%, type B in 34% and type C in 61% of the patients. Out of the patients with urethral trauma, 35 % receive therapy for urethral stenosis

77

and 30% reported urinary incontinence. 30% have problems related to impotence.13

 

78

Chapter 6 Viva Answers

81E. What are the radiological signs of rotational and vertical instability in pelvic fractures? Radiological signs of rotational instability include pubic symphyseal diastasis of more than 2.5 cm; fracture of the ischial spine and avulsion fracture of lateral sacrum. Radiological signs of vertical instability include a sacral fracture with a gap; avulsion fracture of the tip of the L5 transverse process; vertical displacement of the sacroiliac joint of more than one cm.

81F.. How would you manage this patient? 81F I would adopt the ATLS approach, initially assessing airway with wi th cervical spine control, breathing breathing and then circulation. There There is likely to be massive blood loss from this injury so rapid fluid replacement including type O negative blood will be key. A full primary survey would be undertaken with a chest radiograph, lateral cervical spine and AP radiograph of the pelvis. A pelvic binder would be applied on suspicion of a pelvic injury. A detailed trauma CT would ideally be obtained, which may obviate the need for a chest and lateral C-spine radiograph.  The genitourinary examination would be undertaken and retrograde retrograde gastrograffin cystogram would be undertaken to assess urethral integrity prior to catheter placement. A pelvic external fixator could be considered in this case if the appropriate expertise was available. Also, transfer to a Level 1 trauma centre should be considered for angiographic embolisation if necessary. After initial resuscitation and temporary stabilisation (either pelvic binder or external-fixator) definitive management would involve stabilisation of anterior symphyseal (open reduction and internal fixation with a symphyseal plate) and posterior sacroiliac diastases (sacroiliac screws).

82A. How would you classify this injury?  The radiographs shows a displaced, displaced, comminuted, segmental segmental and intra-articular fracture of the tibia with an associated fibula fracture. Using the Schatzker classification this is a type VI injury and according to AO this would be a 41-C3 multifragmentary complete articular fracture.

 

Trauma

82B. How do do you you classify soft-tissue injuries injuries with closed fractures? fractures?   Table 6.1 Grade 0

Minimal soft-tissue damage Indirect injury to limb (torsion) Simple fracture pattern

Grade 1

Superficial abrasion or contusion Mild fracture pattern

Grade 2

Deep abrasion Skin or muscle contusion Severe fracture pattern Direct trauma to limb

Grade 3

Extensive skin contusion or crush injury Severe damage to underlying muscle Compartment syndrome Subcutaneous avulsion

82C. How would would you manage this injury, assuming that the initial ATLS ATLS guidelines have been followed and the patient is stable? Assuming that this is an isolated injury in a haemodynamically h aemodynamically stable patient and is closed, initial management would also involve assessment of compartment syndrome with compartment pressure monitoring. I would inform the patient that this is a limb-threatening limb -threatening injury and limb salvage may not eventually be possible. Consultation with consultant vascular/plastic surgeons would aid in this decision making process. However, However, assuming that the injury was closed, isolated and vascularity was intact to a sensate s ensate foot in a physiologically stable patient, I would discuss the options with the patient. One option is to immediately have an above knee amputation, which would lead to a quicker return to function but with the inherent problems of an above- knee k nee amputation. The other option would involve temporary spanning external fixator +/- fasciotomies and then definitive treatment with a fine wire external fixator aiming to achieve a mechanically aligned, united and non-infected tibia. This would be a long process potentially taking months/years of surgical intervention and compliance by the patient.

82D. What decision making tools are available for predictin predicting g limb

79

salvage after high-energy trauma to the lower extremity? Describe them in detail. The mangled extremity severity score (Johansen score (Johansen et al14)  The mangled extremity severity score is probably probably the most well known limb salvage scoring system and is as follows:

 

80

Chapter 6 Viva Answers

Skeletal/soft-tissue injury Low energy (stab; simple fracture; pistol gunshot wound): 1 Medium energy (open or multiple fractures, dislocation): 2 High energy (high speed MVA or rifle GSW): 3 Very high energy (high speed trauma + gross contamination): 4 Limb ischaemia Pulse reduced or absent but perfusion normal: 1* Pulseless; paresthesias, diminished capillary refill: 2 Cool, paralysed, insensate, numb: 3* Shock Systolic BP always > 90 mm Hg: 0 Hypotensive transiently: 1 Persistent hypotension: 2 Age (years) < 30: 0 30 to 50: 1 > 50: 2 It has been stated that a MESS score of greater than 7 equates to a poor limb viability prognosis. Limb Salvage Index (LSI) (Russell et al15) Limb trauma associated with vascular injury. Absolute indications for amputation included a score of 6 or more. Retrospective analysis of 70 limbs. 26 had pulse deficits requiring revascularisation. Seven components related to injury: •













Arterial Nerve Bone Skin Muscle Deep venous injury Warm ischaemia time

Predictive Salvage Index (PSI) (Howe et al16) Combined orthopaedic and vascular injuries. Intent to help prevent the attempted salvage of a doomed or useless limb. Retrospective, 21 limbs, studied

to determine which variables influenced salvage or loss. A limb-salvage score was developed that weighted: •







Level of the vascular injury Degree of osseous injury Degree of muscle injury Warm ischemia time

 

Trauma

82E. What is the evidence to support support the use of these tools? tools? Table 6.2 MESS

LSI

PSI

Bosse (n = 556)

0.46 (sensitivity)

0.46 (sensitivity)

0.46 (sensitivity)

(all limbs, subgroups analysed in original paper)

0.91 (specificity)

0.97 (specificity)

0.87 (specificity)

Bosse et al17 in a prospective study of 556 limbs found that all the above lowerextremity injury-severity scoring systems have limited usefulness and cannot be used as the sole criterion by which amputation decisions are made. Overall, the lower-extremity injury-severity scores lack sensitivity, although in some cases they were very specific. The high specificity of the scores confirms that low scores could be used to predict limb-salvage potential. The converse, however, was not true. The low sensitivity of the indices failed to support the validity of the scores as predictors of amputation. The LSI perormed better than the MESS or PSI scores overall, especially when only Grade IIIb open tibial fractures were considered.

83A. Describe the radiogr radiograph. aph.  The radiograph shows shows a comminuted fracture of the mid-shaft of the clavicle with complete displacement and shortening.

83B. What are the deforming forces involved with a fracture of this type? For a midshaft fracture:  The medial segment is stabilised by the sternoclavicular sternoclavicular ligaments. Sternohyoid Sternohyoid and subclavius muscles also insert medially.  The clavicular head of the sternocleidomastoid sternocleidomastoid muscle pulls the medial segment

81

superiorly.  The lateral segment segment is pulled inferiorly and medially by the clavicular clavicular head of pectoralis major.  The shoulder girdle and the weight weight of the arm exert an inferiorly deforming force on the lateral segment through the acromioclavicular joint and the coracoclavicular coracocla vicular ligaments. Fibres of the trapezius also insert into the posterior border of the lateral third of the clavicle, providing a stabilising force against inferior displacement.

 

82

Chapter 6 Viva Answers

83C. How would would you define nonunion of the clavicle and what is the rate of nonunion with diaphyseal fractures of the clavicle? Non-union of the clavicle is defined as a lack of radiological union by four months. Clinically, Clinically, this can be assessed by the presence of pain and movement at the fracture site upon stressing. Radiologically, Radiologically, it can be assessed on either radiographs or a CT scan. Radiographs will show a lack of bridging callus, but these may be difficult to interpret due to the difficulty in obtaining orthogonal or thogonal views. Usually the clavicle is assessed using an AP and 15° to 20° elevated AP view. An apical oblique view may also be useful. Clavicle fractures can also be assessed using an axillary view for the more lateral fractures. A CT scan will give a more reliable diagnosis of non-union.  The incidence has been reported to be between between 0.13% and 15% for midshaft fractures. In the paper by Robinson et al 18 studying 868 clavicle fractures, the reported incidence of non-union in diaphyseal clavicle fractures was 4.5% at 24 weeks post-injury.

83D. What are the risk factors for nonunion and which is the most predictive? Robinson et al18 found that following a diaphyseal clavicle fracture, advancing age, female gender, displacement of the fracture, and the presence of comminution significantly increased the risk of non-union. On multivariate analysis, all of these factors remained independently predictive of non-union, and, in their final model, the risk of non-union was increased by lack of cortical apposition. In Robinson’s Robinson’s study a complete fracture displacement was the most predictive of non-union. Other factors that have been found to relate to non-union include shortening > 20 mm, displacement > 20 mm, increasing severity of trauma and re-fracture.

Other general risk factors for non-union such as co-morbidities, smoking and medications should also be taken into account.19

83E. What is the rate of dissatisfa dissatisfaction ction following these fractures and what are the possible problems with malunion? Hill et al20 found that malunion with shortening of 2 cm or more in a middle third fracture led to a poor symptomatic and functional result. 31% reported unsatisfactory results, including pain and cosmesis.  There was also a recognised recognised rate of thoracic outlet outlet syndrome. In a RCT by the Canadian Orthopaedic Trauma Trauma Society there was a significantly higher rate of dissatisfaction with the appearance of the shoulder in the non-operative group (47%). This included a “droopy” shoulder, a bump, pain and asymmetry. 21 A recent meta-analysis reported symptomatic malunions in 17 of 200 (8.5%) nonoperatively treated patients after midshaft fractures of the clavicle.22

 

Trauma

In this patient, who works as a bricklayer bricklayer,, one must consider whether he carries heavy on the injured shoulder, as a bump from a malunited clavicle fracture may beloads problematic.

84A. Describe the radiogr radiograph. aph.  There is a displaced right scapular neck fracture, the articular surface of the glenoid does not appear to be involved in the fracture.

84B. What other investigation/s would you request? I would request true anteroposterior radiograph of the shoulder and axillary views. Additionally a CT scan to further fur ther evaluate the degree of displacement, rotation and intra-articular extension (glenoid fracture) should sh ould be undertaken.

84C. What are are the other injuries injuries are are associated associated with this injury? •









Rib fractures Haemo-/pneumothorax Flail chest/tension pneumothorax Brachial plexus injuries Vascular injuries

84D. What is the classification system used for describing these fractures?  The classification system used here is as follows follows – Ideberg:23  Type I – non-angulated, non-angulated, non-displaced  Type IIa – shortened/displaced > 1 cm •



83



 Type IIb – Angulated Angulated > 40°

84E. What are the indications for surgery for these fractures and what approach would you use if you decided to proceed with operative intervention? Indications for surgery include more than 1 cm displacement or 40° angulation (Type IIa and IIb fractures). I would use a posterior (Judet) ( Judet) approach. A curved skin incision starts star ts at the lateral prominence of acromion, courses medially along scapular spine and caudally to inferior angle of the scapula. Deltoid is detached from the scapular spine s pine and retracted laterally. laterally. The approach is deepened through the interval between infraspinatus in fraspinatus and teres minor.

85A. Describe the radiographs. What are the critical lines that are used to assess these radiographs?  There is a lateral dislocation of the radial head and a fracture of the proximal proximal ulna. The radiocapitellar radiocapitellar line is used to assess radial head dislocation. When a line is drawn through the proximal radial shaft and neck, and extended ex tended through the joint, it should pass through to the articulating capitellum. An abnormal

 

84

Chapter 6 Viva Answers

radiocapitellar line will demonstrate dislocation of the radial head. Additionally the anterior humeral line should also be assessed. This is a line drawn on a lateral view along the anterior surface of the humerus and it should pass through the middle third of the capitellum. This This is altered in displaced supracondylar humeral fractures.

85B. What is is the commonl commonly y used classification of these injuries? Monteggia fracture classification: •







Bado I – anterior dislocation of radial head with proximal ulna fracture Bado II – posterior dislocation of radial head with proximal ulna fracture Bado III – lateral dislocation of radial head with proximal ulna fracture Bado IV – anterior radial head dislocation and radial neck and proximal ulna fracture

85C. What are Monteggia equivalent fractures fractures?? 1. Isolated radial head dislocation 2. Fracture of proximal proximal ulna and radial radial neck 3. Both bones proximal third fractures fractures with the radial fracture fracture more more proximal proximal than the ulnar fracture

85D. What would would your management of this injury be and what is the most common complication? Initial management would involve accurate accurate history taking and an d assessment of neurovascular status. This injury would have an initial closed reduction attempted

but may require open reduction. Internal fixation of the proximal ulna would be carried out in an older child.

85E. What are the reasons for for inability to achieve anatomical reduction? Interposition of the annular ligament, inadequate flexion of the elbow and buttonholing of the radial head through the capsule. If there is plastic deformation of the ulna (without fracture) this may also impede reduction.

85F.. How would you treat a chronic post-traumat 85F post-traumatic ic dislocation of the radial head in a child? If the dislocation is symptomatic the radial head can be resected at skeletal maturity, prior to this may lead to angular deformity at the elbow and wrist. Within six months an open reduction and reconstruction of the annular ligament can sometimes be undertaken and also a transcapitellar pin insertion for unstable dislocations. For reconstruction many procedures have been described.  The Bell-Tawse Bell-Tawse procedure uses a central cent ral slip of triceps fascia whilst whils t the LloydRoberts procedure uses the lateral slip of triceps fascia attached distally. 24

 

Trauma

86A. Describe the radiogr radiographs. aphs. Anteroposterior and lateral radiographs radiographs of the elbow showing a fracture of the th e capitellum.

86B. What is the pattern of injury injury in these these fractures? fractures?  This is normally a coronal shear injury that occurs occurs following a fall onto outstretched hand or elbow.

86C. Which gender gender is this injury most common commonly ly seen in and why? why?  This injury is more common in females due to the greater greater carrying angle of the elbow, and also the ability to hyperextend the elbow allowing the radial head to elbow, forcefullyy shear off the capitellum. forcefull

86D. How would you classify this injur injury? y?  Type I •





Hahn-Steinthal fracture Fracture of capitellum in coronal plane Large osseous fragment

 The other main type is Kocher-Lorenz Kocher-Lorenz (Type II) which is a sleeve fracture of the

85

articular surface with little osseous bone. Broberg and Morrey (Type III) – comminuted fracture McKee modification also mentions Type IV – coronal shear including capitellum and trochlea.

86E. How would you treat it and what approach will you use?  This is a displaced fracture and would cause a block block to flexion, I would treat treat this injury operatively. I would employ a Kocher’ Kocher’ss lateral approach to the elbow elbow,, starting over the lateral supracondylar ridge, 5 cm proximal to the elbow joint and then continuing distally dis tally to the lateral surface of the proximal forearm posterior to the radial head. I would keep the forearm in pronation to avoid injury injur y to the posterior interosseous nerve during durin g the approach. The interval between ECU and anconeus is dissected and a capsulotomy is made. After reduction of the fracture I would use headless screws to fix the fracture and prevent impingement at the radiocapitellar joint.

86F.. What are the complications? 86F Nonunion – 1% to 11% with open reduction and internal fixation, heterotrophic ossification (4%) and avascular necrosis.

 

86

Chapter 6 Viva Answers

87A. Describe the radiographs.  The radiographs are are an AP and frog leg lateral lateral view of the right hip in a skeletally immature patient. They They show an avulsion of the anterior inferior iliac i liac spine apophysis.

87B. What is the diagnosis?  Traction injury resulting in the straight head of the rectus femoris muscle avulsing  Traction avulsing the anterior inferior iliac spine apophysis.

87C. What is the mechanism mechanism of injury injury and how how does it reflect upon the type of fracture? Forceful concentric or eccentric contraction of the straight head of the rectus Forceful femoris muscle avulses the anterior inferior iliac i liac spine apophysis. This can typically occur when kicking whilst playing football. A sudden, large force may be more likely to produce a complete avulsion, whereas chronic repetitive traction forces may produce an incomplete avulsion fracture or apophysitis without a history of acute trauma. A hyperflexion injury may lead to an avulsion of the reflected head of

rectus femoris and a hyperextension injury leads to this type of a fracture.

87D. What is the common age group of presentation for these injuries?  The  These se injurie inof juries s occur occur in before children child ren aged 11 to tfuses o 17 year years. They The y occur occurThe throug thr ough h seconda secondary ry centres ossification, the centre withs.the pelvis. secondary ossification centre at the anterior inferior iliac spine may be present at approximately 14 years of age, fusing around 16 years of age.

87E. What is the prognosis and how soon can the patient expect to return to sport? Further fracture displacement may be prevented because the rectus femoris reflected head will still be attached in its normal location. Most paediatric pelvic avulsion fractures will do well with non-operative non- operative treatment. Two case series reported that all patients returned to pre-injury activity levels.25,26 However  However,, some authors recommend open reduction and internal fixation in acute fractures displaced > 1 cm to 2 cm.27 I would advise this patient to rest and partially parti ally weight bear for at least two weeks, avoiding stretching of the rectus femoris muscle mus cle (avoid hip extension postures). I would advise that they should return to football in around three months, however,, this would ultimately be determined with serial clinical and radiological however follow-up.. I would advise them to miss the rest of the football season. follow-up

 

References

References 1. 2. 3. 4. 5. 6. 7.

8. 9.

Bishop J, Ring D. D. Management of radial radial nerve palsy associated with humeral shaft fracture: a decision analysis model. J Hand Surg Am 2009;34:991-6. Seinsheimer F. F. Subtrochanteric Subtrochanteric fractures of the femur. femur. J Bone Joint Surg [Am] 1978;60-A:300-6. Fielding JW, Magliato HJ. Subtrochanteric fractures. Surg Gynecol Obstet 1966;122:555-69. Nork S, Reilly M, Russell TA. Subtrochanteric fractures. In: Browner BD, Levine AM, Jupiter JB,  Trafton PG, eds. Skeletal Trauma. 1998;2:1891-7. Lenart BA, Neviaser AS, Lyman S, et al. Association Association of low-energy low-energy femoral fractures with with prolonged bisphosphonate use: a case control study. Osteoporos Int 2009;20:1353-62. Craig EV. EV. Fractures Fractures of the clavicle. In: Rockwood CA Jr, Jr, Green DP DP,, Bucholz RW, eds. Fractures Fractures in adults. Third ed. Philadelphia, etc: JB Lippincott Company, 1991:928-90. Robinson CM, Court-Brown CM, McQueen MM, Wakefield AE. Estimating the risk of nonunion following nonoperative treatment of a clavicular fracture. J Bone Joint Surg [Am] 2004;86-A: 1359-65. Robinson CM, Cairns DA. DA. Primary nonoperative nonoperative treatment of displaced lateral fractures of of the clavicle. J Bone Joint Surg [Am] 2004;86-A:778-82. Rüedi TP TP,, Allgöwer M. The operative operative treatment of intra-articular fractures of the lower end of the tibia. Clin Orthop Relat Res 1979;138:105-10.

87

10. Meyers MH, McKeever FM. Fracture Fracture of the Intercondylar Eminence of the Tibia. J Bone Joint Surg [Am] 1970;52-A:1677-84. 11. Tile M. Pelvic ring fractures: should they be fixed? J Bone Joint Surg [Br] 1988;70-B:1-12. 12. Burgess AR, Eastridge BJ, Young JW, JW, et al. Pelvic ring disruptions: effective classification system and treatment protocols. J Trauma 1990;30:848-56. 13. Pavelka T, T, Houcek P, P, Hora M, Hlavácová J, Linhart M. Urogenital trauma associated with pelvic ring fractures. Acta Chir Orthop Traumatol Cech 2010;77:18-23 (in Czech). 14. Johansen K, Daines M, Howey T, Helfet D, Hansen Hansen ST Jr. Objective criteria accurately predict predict amputation following lower extremity trauma. J Trauma Trauma 1990;30:568-73. 15. Russell WL, Sailors DM, Whittle TB, Fisher DF Jr, Burns RP. Limb salvage versus traumatic amputation: a decision based on a seven-part predictive index. Ann Surg 1991;213:473-81. 16. Howe HR Jr, Jr, Poole GV, GV, Hansen KJ, et al. Salvage of lower extremities following combined combined orthopedic and vascular trauma: a predictive salvage index. Am Surg 1987;53:205-8. 17. Bosse MJ, MacKenzie EJ, Kellam JF, JF, et al. A prospective evaluation of the clinical utility of lower extremity injury severity scores. J Bone Joint Surg [Am] 2001;83-A:3-14. 18. Robinson CM, Court-Brown CM, McQueen MM, Wakefield Wakefield AE. Estimating the risk of nonunion following nonoperative treatment of a clavicular fracture. J Bone Joint Surg [Am] 2004;86A:1359-65. 19. Gaston MS, Simpson AH. Inhibition of fracture healing. J Bone Joint Surg [Br] 2007;89-B:1553-60. 20. Hill JM, McGuire MH, Crosby Crosby LA. Closed treatment of displaced displaced middle-third fractures of the clavicle gives poor results. J Bone Joint Surg [Br] 1997;79-B:537-9. 21. No authors listed. Nonoperative treatment compared with plate fixation of displaced midshaft clavicular fractures: a multicenter, randomized clinical trial. J Bone Joint Surg [Am] 2007;89-A:1-10. 22. McKee RC, Whelan DB, DB, Schemitsch EH, McKee MD. MD. Operative versus nonoperative care of displaced midshaft clavicular fractures: a meta-analysis of randomized clinical trials. J Bone Joint Surg [Am] 2012;94-A:675-84. 23. Ideberg R. Fractures of of the scapula involving the glenoid fossa. In: Bateman JE, Welsh Welsh RP, RP, eds. Surgery of the shoulder. Philadelphia: Decker, 1984:63-6. 24. Canale ST, ST, Beaty JH. Campbell’s Campbell’s operative orthopaedics. orthopaedics. 11th ed. St. Louis: Mosby, Mosby, 2008. 25. Fernbach SK, Wilkinson RH. Avulsion Avulsion injuries of the pelvis and proximal femur. femur. AJR Am J Roentgenol 1981;137:581-4. 26. Metzmaker JN, Pappas AM. Avulsion fractures of the pelvis. Am J Sports Med 1985;13:349-58. 27. Lynch SA, Renström PA. PA. Groin injuries in sport: treatment strategies. Sports Med 1999;28:137-44.

 

Chapter 7 Viva Questions: Hands

88. A 32-year-old 32-year-old electrician presents presents to the clinic with a history of pain in his elbow, which is radiating down his forearm. He does not give any history of  trauma but complains of occasional numbness and tingling in his forearm.  A.  What is your differential diagnosis? diagnosis? B. How will you confirm your diagnosis clinically? C.  What are are the common sites of entrapment entrapment of the ulnar nerve along its course in the upper arm and forearm? D.  What investigation investigation would you request request if any and why? why? E. How would you treat him at this stage? F. If conservative management fails, what kind of surgical intervention would you perform for this condition? G.  What is the expected expected prognosis following following surgical intervention? intervention? 89. A 34-year-old 34-year-old man sustained sustained a hyperextension hyperextension injury injury to his little finger leading  to this injury (Fig. 7.1).  A.  What is your differential diagnosis? diagnosis? B. How would you confirm your diagnosis?

Figure 7.1

 

90

Chapter 7 Viva Questions

C.  Which finger is commonly commonly involved? D.  What is the classification classification system system associated with this problem? E.  What are are the treatment treatment options? F. How would you like to treat this patient? 90. A 40-year-old scaffolder presents with persistent pain in his wrist wrist following a

fall on his outstretched hand about six months ago. Tese are the radiographs (Figs 7.2a and b).  A.  What is your diagnosis? B.  What is the classification classification system system associated with this problem? C.  What stage/grade stage/grade would you assign to this radiograph? radiograph? D.  What is the natural natural history of this this condition? E.  What are are the options for treatment? treatment? F. How would you treat this patient?

a

b Figure 7.2a and b

 

Hands

91. A 42-year-old woman woman presents to you with a pain and swelling in the small joints of her hand. Tese are her radiographs (Figs 7.3a and b).

 

91

 A.  What is the diagnosis? diagnosis? B.  Which skin condition is associated associated with this problem? problem? C.  What are are the characteristic characteristic radiological radiological features? D.  What is the natural natural history of this this condition?

a

b

Figure 7.3a and b

92. A 14-year-old left-hand left-hand-dominant -dominant boy presents with a painless deformity of his hand. Tese are the clinical c linical photograph and radiograph (Figs 7.4a and b).  A. Describe the abnormalities seen in the photograph and the radiograph. B. Classify this abnormality. C. Discuss common inheritance patterns of this condition. D.  What are are the main indications indications for surgical treatment in this condition? E.  What are are the principles when considering surgical treatment treatment of this this particular case?

 

92

Chapter 7 Viva Questions

a

b

Figure 7.4a and b

93. A 50-year-old 50-year-old female presents presents with a history of a painful swelling in her finger. finger. It appears to burst on occasion. Tis is the clinical photograph and radiograph (Figs 7.5a and b).  A.  What are are the differential differential diagnoses? B.  What is the diagnosis? diagnosis? C.  What are are the causes of this condition? D. How would you treat this patient? E.  What are are the possible complications? complications?

a Figure 7.5a and b

 

b

Hands

94. A 32-year-old 32-year-old weightlifter presents with with this injury (Figs 7.6a and b).  A.  What is your diagnosis? B. How does this injury usually occur? C.  What are are the treatment treatment options? D.  What is the prognosis prognosis if the patient patient opted for surgical surgical intervention?

a

b

Figure 7.6a and b

95. A 32-year-old pianist pianist who is left-hand dominant dominant presents after a fall from stairs and injuring his left hand. Tis is his radiograph (Fig 7.7).  A. Describe the radiograph radiograph.. B.  What is your diagnosis? C. How is this injury assessed? D.  What are are the options for treatment? treatment? E. How will you manage this patient?

 

93

 

94

Chapter 7 Viva Questions

Figure 7.7

96. A 55-year-old 55-year-old I professional presents presents with painful superficial nodules over his palm over a period of a year (Fig. 7.8).  A.  What is your diagnosis? B.  What is the cause cause of pain in this condition? condition? C. How would you stage this condition? D.  What is the treatment treatment for for this condition? E.  What are are the possible complications complications of surgical surgical intervention?

Figure 7.8

 

Hands

97. Tis is the clinical photograph (Fig. 7.9) of a man who has had a previous previous procedure for an injury to one of the tendons in the hand.  A.  What procedure procedure has this patient had? had? B.  What deficit would the patient patient have prior to to surgery? are the causes of this pathology? C.  What are D.  Which other tendon transfers have have been described?

E.  What are are the principles of a tendon transfer?

Figure 7.9

 

95

 

96

Chapter 7 Viva Questions

98. Tis is the clinical photograph (Fig. 7.10) 7.10) of a 28-year-old 28-year-old who presented to the hand clinic with a long-standing history of a problem with his thumb.  A. Describe the clinical photograph. differential diagnosis? diagnosis? B.  What is the differential C.  What is the most most likely diagnosis?

D.  What is the pathophysiology pathophysiology of this condition? E.  What is the treatment treatment of this condition?

Figure 7.10

 

Hands

99. A 14-year-old 14-year-old boy presents with a hyperextension hyperextension injury to his index finger  while playing football. Clinically Clinically the finger appears slightly slightly swollen and tender with severely reduced range of movements particularly flexion. Several attempts at reduction under a ring block in A&E were unsuccessful. Here are the radiographs (Figs 7.11a to c).  A. Describe the radiographs radiographs.. B.  What is the diagnosis? diagnosis? C.  What ar are e the causes of irreducibility? D.  What is the treatment? treatment? E.  What is the prognosis? prognosis?

a Figure 7.11a to c

b

c

 

97

 

Chapter 8 Viva Answers: Hands 88A. What is your differential diagnosis? •

Local

–   Ulnar nerve entrapment entrapment in the cubital tunnel –  Lateral/medial epicondylitis –   Radial/median nerve entrapment entrapment at the elbow –  Elbow arthropathy (Pancoast lung tumour) tumour) –   Brachial plexus lesion (Pancoast –  Cervical radiculopathy • Systemic –  Peripheral neuropathy –   Mononeuritis multiplex –  Multiple sclerosis –   Leprosy –   Lyme disease

88B. How will you confirm your diagnosis clinicall clinically? y? History – do symptoms come on at night, does patient sleep with elbows bent (ulnar nerve compression). Is the pain aected by movement?

Examination evidence of claw hand, looking for evidence of ulnar nerve dysfunction (weakness of small muscles of hand, Froment’s Froment’s sign). Sensory decit in ulnar 1.5 ngers. Tapping Tapping over nerve in cubital tunnel may re-create symptoms. Also assess elbow range of movement and look for tenderness over epicondyles to rule out other pathology.

88C. What are the common sites of entrapment of the ulnar nerve along its course in the upper arm and forearm?

Cubital tunnel – the ulnar nerve ner ve becomes commonly compressed in the cubital tunnel which has the aponeurotic attachment of the two heads of exor carpi ulnaris (FCU) as a roof with the arcuate (Osborne’s) (Osborne’s) ligament and the oor of the tunnel is formed by the medial collateral ligament. Guyon’s canal – pisiform and hamate and roof is the volar carpal and pisihamate ligament - both ulnar nerve and artery travel through here.

 

100

Chapter 8 Viva Answers

88D. What investigation investigation would you you request if any and why? Investigation would depend on clinical history and examination. If the patient was experiencing mild discomfort and there were few clinical signs, simple measures such as avoiding aggravating positions of the elbow may help. Nonetheless it would be useful to establish a clear diagnosis and therefore I would obtain nerve conduction studies.

88E. How would you treat him at this stage? Avoid excessive elbow exion and any aggravating manoeuvres. Consider elbow splintage, particularly at night. Non-steroidal anti-inammatories unless otherwise contra-indicated. Further follow up in 3 months.

88F.. If conservati 88F conservative ve management fails, what kind of surgical intervention would you perform for this condition? Simple decompression of the ulnar nerve ner ve (cubital tunnel decompression): 1. The incision is 8 cm in length and is made over the medial aspect of the elbow.. An attempt to identify the medial antebrachial cutaneous nerve is elbow made (often crossing FCU). 2. The ulnar nerve is identied and tagged proximally proximally.. 3. The nerve is followed followed upward upward and released released at the intermuscular septum 4. The aponeurosis aponeurosis of the the cubital tunnel and FCU are are then incised. 5. Submuscular or subcutaneous subcutaneous transposition transposition of the ulnar nerve maybe considered prior to closure

88G. What is the expecte expected d prognosis following surgical intervention?

Keith & Wollstein1 showed that overall 90% of the patients reported improvement in function following cubital tunnel decompression. Complete recovery is less frequent than in carpal tunnel syndrome. Poor prognostic factors include: a. b. c. d.

Symptoms existing for for over one year year before surgery Severe Sever e existin existing g atro atrophy phy Alcoho Alc oholis lism m Absence of response response to electrical stimulation stimulation of the nerve

89A. What is your differential diagnosis?  The clinical picture shows the loss of the normal cascade of the ngers when the hand is in the resting position. Given the acute hyperextension injury, the likely diagnosis is of exor digitorum proundus (FDP) avulsion of the little nger nger.. Other dierentials include a trigger nger n ger.. Rupture of the exor tendons could be secondary to rheumatoid arthritis, ar thritis, cystic degeneration, post-fracture, or calcication of the triangular brocartilage complex (TFCC).

89B. How would you confirm your diagnosis?  The diagnosis would be conrmed with a full history to rule out antecedent pathology and a thorough clinical examination. This would include separate

 

Hands

testing of the FDP and exor digitorum supercialis (FDS) tendons. In FDP avulsion the avulsed fragment can be felt as a mass over the proximal phalanx or sometimes in the palm. Radiographs may help localise the position of the retracted tendon. MRI allows for accurate pre-operative assessment of tendon position and degree of retraction, thereby facilitating surgical planning and approach. Some centres advocate the use of ultrasound rather than MRI.

89C. Which finger is commonly involved?  The ring nger is most commonly aected aected (in 75% of patients). The ring ngertip ngertip is usually the most prominent or “longest” during grip in 90% of cases and that it absorbs more force than any other nger during pull-away testing. Secondly the exor digitorum profundus tendon, as demonstrated in cadaver specimens indicates a signicantly weaker insertion of the ring nger compared with the middle nger. These factors contribute to the susceptibility of the ring nger to the profundus avulsion injury.2

89D. What is the classificat classification ion system associated with this problem? Leddy and Packer3 have classied profundus avulsions. In type I, the tendon retracts to the palm, held up by the lumbrical origin with disruption of the entire vincular system. Type Type II avulsion is characterized by retraction of the tendon to the PIP level that spares the vinculum longum, presumably maintaining blood supply.. In type supply t ype III, the profundus tendon avulsion occurs in addition to a fracture

101

of the distal phalanx base.

89E. What are the treatment options?  Treatment  Trea tment can either either be operat operative ive or or non-opera non-operative tive.. Return Return to functio function n is signicantly better with surgical intervention. Options include re-attachment to the distal phalanx, tendon grafts, staged reconstruction, tenodesis and arthrodesis.

89F.. How would you like to treat this patient? 89F  The factors that inuence treatment treatment are: 1. the length of time between injury and treatment 2. tendon retraction 3. blood supply to the fragment 4. the presence of bony fragments. My preferred preferred treatment would depend on all of the above. An early diagnosed Type 1 injury would be amenable to delivery of the tendon from the palm and re-attachment to the distal phalanx with a pull-out wire. For three weeks the limb is immobilized in a dorsal splint with the wrist in exion, MCPJ MCPJ in 70° of exion and the th e IPJs IPJs in extension. The wire is then removed and hand therapist completes the rehabilitation. Late injuries can be treated by tendon grafting, tenodesis or arthrodesis. Type Type 2 and 3 injuries can be repaired at a later date because of preservation of circulation.

90A. What is your diagnosis?  This is a radiograph of a chronic chronic scapholunate (SL) injury with a dorsal intercalated segmental instability (DISI) deformity.  

102

Chapter 8 Viva Answers

90B. What is the classificat classification ion system associated with this problem?  These injuries are assessed and best treated based on which restraints restraints are damaged and the duration from the actual index event. Garcia-Elias et al4 set out the following assessment criteria in the assessment of SL injuries. • Is the dorsal scapholunate ligament partially or or completely torn? • If compl complete, ete, can the ligament be repaired repaired and what is the healing potential? potential? • What is the status status of the the secondary secondary scaphoi scaphoid d stabilizer stabilizerss (i.e. (i.e. is the radioscaphoidangle less than 45°)? • Is the carpa carpall malal malalignmen ignmentt reduci reducible? ble? • Are the cartila cartilaginou ginouss surfaces normal normal?? Based on these conditions SL injuries can be classied as per Table Table 8.1: Table 8.1 Stages

Pathology

S1: Pre-dynamic

SLIL partial tear

S2: Dynamic

Complete SLIL injury and repairable DSL

S3 : Static no DISI

Complete nonrepairable D and VSL, no DISI ST T ligaments intact

S4:: St S4 Stat atic ic wi with th red edu uci cibl ble e DI DISI SI

Com ompl plet ete e sc scap apho holu luna nate te lig igam amen entt in inju jury ry wi with th a non-repairable reducible rotary subluxation of the scaphoid

S5:: St S5 Stat atic ic wi with th irr rred educ ucib ible le DI DISI SI

Com ompl plet ete e sc scap apho holu luna nate te lig igam amen entt in inju jury ry wi with th irreducible malalignment but normal cartilage

S6: Static with irreducible DISI and degenerative change

Complete scapholunate ligament injury with irreducible irreducib le malalignment and cartilage degeneration

90C. What stage/grade would you assign to this radiograph?  This appears to be a static deformity. deformity. The reducibility can be checked on table table during surgery. It is likely that this is stage s tage 4 to 5 injury based on radiographs.

90D. What is the natural history of this condition? condition? If left untreated chronic scapho-lunate dissociation will lead to a scapholunate advanced collapse (SLAC) wrist. This describes arthritic changes that result due to abnormal loading. The rst joint to develop degenerative changes is the radioscaphoid, then capitolunate and then the scapho-trapezoid/trapezium joints.

90E. What are the options for treatment? See Table 8.2

 

Hands

Table 8.2 Treatment Stage

Pathology

Management

S4: Static with reducible DISI

Complete scapholunate ligament injury with a nonrepairable reducible rotary subluxation of the scaphoid

Ideal indication for the tendon weaves.

S5: Static with irreducible DISI

Complete scapholunate ligament injury with irreducible malalignment but normal cartilage

Better to accept the morbidity caused by a partial fusion than to attempt a ligament reconstruction that most likely will fail. Options: STT, scaphocapitate fusions.

3 LT LT tenodesis (Figs 8.1a and b) of Brunelli procedure63

103

a

b

Figure 8.1a and b

90F. How would you treat this patient? See Table 8.2

91A. What is the diagnosis?  The radiological abnormalities abnormalities and clinical nding would be consistent with a diagnosis of a seronegative spondyloarthopathy aecting the hand. In this case radiographic features features of psoriatic arthritis are present.

91B. Which skin condition is associated with this problem? Psoriasis.

91C. What are the characteri characteristic stic radiological features?  The class cl assic ic radiogra ra diographic phic feature featuress of psori psoriatic atic art arthrit hritis is that t hat can be seen se en in i n this th is case include joint space narrowing, peri-articular joint erosions, osteolysis, PIPJ/DIPJ PIP J/DIPJ ankylosis and the development of a “pencil-in-cu “pencil-in-cup” p” deformity of

 

104

Chapter 8 Viva Answers

marked lysis of the distal end of a phalanx with bony remodelling of the proximal end of the more distal phalanx. Other radiological signs include resorption of the distal phalanges i.e. Morningstar appearance, bony proliferation including shaft and periarticular peri-ostitis, spur formation and spondylitis. Radiological changes in psoriatic arthritis are often asymmetric and oligoarticular, oligoar ticular, most commonly involving involvin g the carpus, MCP MCP,, PIP, PIP, and DIP DI P  joints.  joint s.

91D. What is the natural history of this condition? condition? Psoriatic a pleomorphic that considered can aect any joint and has a variablearthritis course is and prognosis. It disease was initially to be less severe than rheumatoid arthritis. However However,, up to 20% of patients having a severe, debilitating form of degenerative arthritis. Patients with psoriatic arthritis

experience intermittent symptomatic ares, with variable lengths of intervening i ntervening remission. The natural history of the disease varies by subtype of clinical and radiologicalpresentation. Risk factors for severe, progressive progressive destructive disease include female gender gender,, polyarticular disease at presentation, younger age at symptom s ymptom onset, and acute onset of arthritis. The mortality rate of patients is higher than that of the general population. The disease leads to signicant functional disability and a reduced quality of life. At time of diagnosis almost a third of patients are bedridden or have limited their activities of daily living to self-care. Progression Progr ession of clinical damage is seen in the majority of patients, and only small percentages achieve complete, prolonged remission without therapeutic intervention.5

92A. Describe the the abnormalities abnormalities seen in the photograph photograph and the radiograph.  The photograph photograph and the radiograph show polydactyly of the thumb, thumb, with duplication of the proximal and distal phalanges and a broad based metacarpal head.

92B. Classify this abnormalit abnormality. y.  The Wassel Wassel 6 classication is used to determine the dierent types of duplication; • • • • • • •

Type I: Bid distal phalanx Type II: Duplicated distal phalanx Type III: Bid proximal phalanx Type IV: Duplicated proximal phalanx (most common) Type V: Bid metacarpal Type VI: Duplicated metacarpal Type VII: Triphalangia

 This case is classied as a Type Type IV, IV, which is the most common form (43%).7

 

Hands

92C. Discuss the common inheritanc inheritance e patterns patterns of this condition. condition. Isolated duplication is normally unilateral and sporadic, whereas a triphalangeal thumb usually arises form autosomal dominant inheritance. Type VII is also associated with several s everal syndromes such as Holt-Oram syndrome, Fanconi’ Fanconi’ss anemia, Blackfan-Diamond anemia, imperforate anus and cleft palate.

92D. What are the main indications for surgical treatment in this condition? Surgical intervention is almost always indicated for better function func tion and cosmetic

105

reasons. This This is normally performed from the age of 18 months to 5 years. The The type IV polydactyly patient should be treated very early, because if one waits too long, the supernumerary component displaces the normal component into marked radial or ulnar deviation, and growth continues in this direction.

92E. What are the principles when considering surgical treatment of this particular case? Surgical intervention in this Type IV duplication involves excision of the most hypoplastic (usually radial) thumb, narrowing of the MCPJ articular surface, ligament (especially radial collateral) reconstruction, intrinsic transfer and possibly centralisation of the extrinsic exor and extensor tendons.8

93A. What are the differential diagnoses?  The dierential diagnoses diagnoses here would include the following: following: • • • •

Mucous cyst Tumour Tumo ur (benign/malignant) Xanthoma Abscess

93B. What is the diagnosis?  The diagnosis is a mucous cyst with associated breakdown of the skin. There are degenerative changes in the DIPJ (dorsal osteophytes).

93C. What are the causes of this condition? Osteoarthritis of the DIP DIPJJ and trauma can cause this condition. There is mucoid degenration of the connective tissue.

93D. How would you treat this patient? I would take a detailed history and examination and ascertain the eect of the cyst on function of the nger and the patient’s symptoms. Conservative measures include massage and compression. Aspiration can be attempted but re-accumulation is likely.  The denitive management is excision of the cyst, which can result result in loss of skin

 

106

Chapter 8 Viva Answers

over the ulcerated area. In order to achieve skin coverage a ap can be rotated using the excess skin ski n over the PIP joint. There are several ways of doing this, one of which is demonstrated below in Fig. 8.2a.  The procedure procedure is performed using a digital block and a digital tourniquet for haemostasis. The ap is marked and with the area of skin ulceration that needs to be excised (Fig. 8.2b).

 The ap is the raised and rotated to cover the defect (Fig. (Fig. 8.2c). This is facilitated

b

a

c

Figure 8.2a to c

by a proximal release (arrow) of the ap, which allows rotation and coverage coverage.. Any osteophyte associated with the cyst must be removed to decrease chances of recurrence.

93E. What are the possible complications? Recurrence, nail deformities, risk of nerve/tendon/vessel damage (rare).9

94A. What is your diagnosis?

 The diagnosis is a right-sided distal biceps tendon rupture. rupture. In addition due to the obvious retraction of the biceps muscle belly it is likely that the bicipital aponeurosis is also not intact.

94B. How does this injury usually occur?  This occurs due to eccentric eccentric loading of the exed elbow elbow during activity such as weight lifting. Almost solely in middle-aged males, with steroid and tobacco use being risk factors.

94C. What are the treatment options?  Treatment options are non-operative but this leads to achievement of 50%  Treatment normal supination strength and 70% normal exion strength at one year. Surgical intervention is recommended for active individuals and can be

 

Hands

performed through a single/dual incision technique. Fixation techniques using suture anchors/interference screws and an endobutton have all been described.

94D. What is the prognosis if the patient opted for surgical

107

intervention? Peeters et al10 found that at 16 months following xation using an endobutton showed excellent functional outcomes with an almost full range of movement. 80% of exion strength and 91% of supination strength was regained. One out of 26 patients required removal of the endobutton and there were no other signicant complications. Other reported complications in other series are heterotopic ossication, radioulnar synostosis, infection and neurological injury.

95A. Describe the radiograph.  There is a comminuted intra-articular intra-articular fracture at the base of the middle phalanx aecting the PIP joint. The fracture aects > 50% of the articular surface and there th ere is subluxation of the joint.

95B. What is your diagnosis? Hastings type III unstable dorsal fracture subluxation of the PIP PIPJ. J. Dorsal dislocations of the PIP joint are often associated with palmar lip fractures and can be classied based on the degree of stability. When instability persists with greater than thirty degrees of exion at the PIP joint to maintain reduction, the cup shaped geometry of the volar lip of the middle phalanx will need to be reconstructed.  The amo amount unt of arti articul cular ar surfa surface ce inv involv olveme ement nt and the res result ultant ant cli clinic nicall ally y demonstrated instability can be used to group palmar lip PIP fracture dislocations into one of three categories: stable, tenuous, and unstable. Stable fractures involve < 30% of the joint surface and maintain a congruent reduction, throughout the joint excursion. Tenuous Tenuous fractures will show an involvement of 30 to 50% and are unstable injuries. Unstable injuries are those in which > 50% of the articular surface is involved. Also included in this group are fractures that remain unstable at 30° of exion.

95C. How is this injury assessed? A detailed history and examination is necessary. The history should include hand dominance, profession, hobbies/sports and to what level these are participated in (professional/amateur). (professional/amateur ). The examination should include the neurovascular status of the digit and record any laceration, bruising and areas of tenderness. The alignment (coronal, sagittal and rotational) and swelling of the digit should be noted. Under a digital nerve block the range of movement and subluxation can be assessed. One should also document the status of the tendons FDS, FDP and the central slip, although the latter is more commonly injured in i n volar PIPJ PIPJ dislocations. Radiologically the injury is assessed with an AP and true lateral radiograph of the digit. Look for the “V” sign sign dosally at the PIPJ on the lateral radiograph, which represents subluxation and an incongruent joint.

 

108

Chapter 8 Viva Answers

A CT scan may give useful information about the fracture conguration if one is considering an ORIF.

95D. What are the options for treatment?  The joint is subluxed and incongruent, incongruent, therefore therefore the aim of treatment is to reduce reduce the joint and hold the reduction to allow healing of the fracture in a favourable position. Earlyare movement is desirable in order to minimise stiness. The options of treatment broadly operative or non-operative.  Table 8.3 lists the various options with pros and cons of each. This particular  Table injury is unstable and aects more than 50% of the joint surface; therefore, extension block splinting is unlikely to be eective. The other treatment options listed here are all possible in this patient.

Table 8.3 Treatment Option

Pros

Cons

Extension splinting block

 Treatment of choice cho ice forvolar most < 40% articular surface base fractures.

Requires carefulnot assessment ofmost congruence and follow-up, eective in > 40% articular surface volar base fractures. With a greater amount of joint surface involved the blocking must begin at a higher angle. May have a residual xed exion contracture.

Extension block pinning

Selected volar base fractures that reduce spontaneously with PIP joint exion.

Pin tract infection.

ORIF

Can restore art rtiicular congruity and may permit early range of movement.

Stiness, infection. May not be an option if fracture is comminuted.

Dynamic external xator

All llo ows so some me mo mov vem eme ent nt..

Pin tr trac actt in inffec ecti tion on..

Volar plate arthroplasty

Specic strategy for subset of volar base fractures involving between 30–60% of the articular surface.

Numerous specic anatomic requirements must be met for procedure to be eective; surface is not hyaline cartilage.

Osteochondral reconstruction

Bone to bone healing, true hyaline cartilage restoration, recreates volar lip buttress eectively.

 Technically complex; graft resorption and  Technically collapse are possible.

 

Hands

109

95E. How will you manage this patient? It is important to inform the patient that this is a serious injury and no matter how it is treated, the joint will have signicant stiness and swelling. I would seek advice from a specialist hand surgeon, especially as the patient is a pianist. I would inform the patient that for this injury there is no single treatment option that is proven to give a superior outcome. Stable palmar lip fractures can be treated by early range of motion exercises with buddy taping and protected motion. PIP joint hyperextension should be avoided with the help of extension block splints.  Tenuous  Tenuo us PIP P IP fractu f racture re disloca di slocation tionss can ca n be treate treated d with wi th stati s taticc splinti spl inting ng but b ut this does not lend itself to early motion. Dorsal extension block splinting (McElfresh) maintains reduction by limiting extension at the PIP joint while allowing for early active exion. Extension block pinning can also be used to limit extension. In unstable fractures, treatment options include open reduction and internal xation, palmar plate arthroplasty, external xation and hemi hamate autograft. Open reduction is suitable for large minimally comminuted palmar lip fractures. Depressed fragments can be elevated and the buttressing eect of the volar lip can be restored. In the presence of extensive comminution a number of traction devices can be used to treat unstable fractures. However these systems in addition to being bulky may fail to restore the volar lip, resulting in recurrent dislocation and posttraumatic arthritis. Eaton in 1980 advanced the palmar plate into the defect after excising the comminuted fragment. Ishida advocated the use of an osteochondral graft to resurface the damaged PIP  joint in 1994. Hill Hastings in 1999, noting the similarity in shape of the middle phalanx base and the distal hamate, described the use of a hamate allograft in the treatment of PIP fracture dislocations. A volar approach is used and the joint is “shotgunned” “shotgunned” (Fig. (Fig. 8.3a). With the fracture prepared (Fig. (Fig. 8.3b) the defect is measured and a matching graft is harvested from the distal hamate (Figs 8.3a to c). This is then xed to the base of the middle phalanx with screws of adequate length (Fig. 8.3e).  

Further Reading Williams RM, Hastings H 2nd, Kiefhaber TR. PIP fracture/dislocation treatment technique: use of a hemi-hamate resurfacing arthroplasty. Tech Tech Hand Up Extrem Ex trem Surg 2002;6:185-92. Williams RM, Kiefhaber TR, Sommerkamp Sommerk amp TG, Stern PJ. Treatment Treatmentautograft. of unstable dorsal proximal interphalangeal fracture/dislocations using a hemi-hamate J Hand Surg Am 2003;28:856-65.

 

110

Chapter 8 Viva Answers

a

b

c

d

e Figure 8.3a to e

96A. What is your diagnosis? Dupuytren’s nodes.

96B. What is the cause of of pain in this this condition? condition? Pain is not always a feature of Dupuytren’s but may be present in the initial initi al presentation of nodules due to local pressure eects. Work undertaken by von Campe et al 11 showed that at dissection tiny nerve bres were found passing through excised nodules. Biopsies showed nerve ner ve bres embedded in the dense brous tissue in six of their cases.  The authors also found neuromas neuromas and the presence of intraneural or perineural brosis. The The authors concluded that the ndings may be a consequence of local nerve compression by the bromatosis. Fibrous tissue tends to be inelastic thus persistent compression of a nerve may be followed by decrease in axonal

 

Hands

transport and nerve perfusion resulting ultimately in a brosis of the nerve and the development of a pseudoneuroma. Enlarged Pacinian corpuscles are found in Dupytren Dupytren’’s disease. It I t was theorised that the increase in density and size of the corpuscles may be related to the release nerve growth factors. Such enlarged Pacinian corpuscles can be 12 presentof inlocal patients with no pain but can also cause pain. In the paper by von Campe et al11 the authors discuss the association of Dupuytrens disease with sprouting of substance P positive nerve bres which have been implicated in causing pain in Achilles tendinosis. This pathophysiological mechanism, it was felt, may explain pain in the early stages of Dupuytrens disease.

96C. How would you stage this condition? I would use the Woodru classication system:13 • • • • •

Stage 1 - early palmar disease with no contracture contracture (as in this case) Stage 2 - one nger involved involved with MCPJ MCPJ contracture contracture Stage 3 - one nger MCP MCPJJ + PIP PIPJJ Stage 4 - stage stage 3 + more more than one nger nger invol involved ved Stage 5 - nger in palm palm defo deformity rmity

96D. What is the treatment for this condition? condition? Patients with persistently painful nodules for greater than a year can be oered excision. However the evidence base for this is poor poor.. The authors (von Campe 11 et al)  acknowledge that their numbers were small and that their study did not include controls.

96E. What are the possible complications complications of surgical intervention? A 20-year review of surgical complications associated with fasciectomy fasciec tomy for Dupuytren’’s contracture showed that major complications occurred in 15.7% of Dupuytren cases, including digital nerve injury (3.4%), digital artery injury (2%), infection (2.4%), hematoma (2.1%), and complex regional pain syndrome s yndrome (5.5%), in addition to minor complications including painful are reactions in 9.9% of cases and wound healing complications in 22.9% of cases. Amputation may be required in cases of digital artery injury.14

97A. What procedure has this patient had? EIP (extensor indicis

proprius transfer) for rupture rupture of the extensor pollicis longus.

97B. What deficit would the patient have prior to surgery surgery??  The extenso extensorr pollicis pollicis longu longuss originate originatess from from the middle thir third d of poster posterior ior surface surface of of the ulna and the radioulnar interosseous membrane. It inserts into the dorsal base

111

of the thumb distal phalanx through the thumb extensor mechanism. Its function is to retropulse the thumb at the interphalangeal i nterphalangeal joint. In simple terms retropulsion is the term used for lifting the thumb o a table while keeping the hand at.

 

112

Chapter 8 Viva Answers

97C. What are the causes of this this pathology? pathology? Most ruptures of the extensor pollicis longus (EPL) occur at the level of Lister’s tubercle and are the consequence of mildly displaced distal radial fractures that have healed, leaving a rough surface on the oor of the third extensor compartment. A combination of tendon attrition and reduced blood supply to such a tight compartment may explain the late appearance of the rupture. Reports of EPL tendonitis secondary to an isolated fracture of Lister’ Lister ’s tubercle or due to a direct injury to the tendon without a distal radial fracture is much less common. Ferreres et al15 also have demonstrated wrist hyperextension injury as a possible cause of EPL rupture. Spontaneous rupture of the EPL has also been described in a patient with long-term systemic steroid usage.16

97D. Which other tendon transfers have been described? Mensch in 1925 was the rst to describe the use of extensor indicis for tendon transfer and this is the most commonly used tendon. Extensor carpi radialis longus (ECRL), extensor pollicis brevis (EPB) and an intercalated tendon graft have also been used. APL is unsuitable for reconstruction of EPL due to its small amplitude of movement, and it also has a dierent direction of pull. However, Chitnis and Evans17 used APL in their study of 20 patients and described satisfactory objective and subjective results.

97E. What are the principles of a tendon transfer? Principles when deciding on tendon transfers are: • • • • • • •

Match muscle strength Force For ce should be proportional to cross sectional area area Amplitude should be proportional to length of the muscle Work capacity = force × amplitude Motor strength will decrease one grade after transfer Appropriate tensioning Appropriate excursion (can adjust with pulley pulley or tenodesis eect)

Requirements also include patient compliance, no joint contractures, no active infection and grade 5/5 power (this will drop one grade).

98A. Describe the clinical photogr photograph. aph.

Clinical photograph of macrodystrophia lipomatosa (ML) which is a rare cause of congenital macrodactyly, associated with progressive proliferation proliferation of all mesenchymal elements, but primarily the broadipose tissue.18 Unusually it may involve the whole upper limb and abdominal wall involvement has been reported.19 Clinically the condition may present as early as the neonatal

 

Hands

period to late adulthood. There is a male preponderance and usually a single sin gle sclerotome appears to be involved. Commonly the lateral aspect of the upper limb along the median nerve distribution appears to be involved while the medial aspect of the lower limb (i.e. along the plantar nerve distribution) is can be aected.  The lower limb is more more often involved than the upper limb and the 2nd and 3rd digits are more commonly aected. Growth usually stops with the onset of puberty.

98B. What is the differential diagnosis?  There are certai ce rtain n conditions, condi tions, which can present pres ent with wi th localized loca lized limb li mb hypertrophy.. The dierential diagnosis of ML includes neurobromatosis hypertrophy type 1 (plexiform neurobroma), brolipomatous brolipomatous hamartoma (FLH), lymphangiomatosis, hemangiomatosis and Klippel– Klippel–Tr Trénaunay–W énaunay–Weber eber syndrome, Mafucci syndrome, Ollier disease and Proteus syndrome. It is important to note that in all these conditions there is a positive family history and these conditions are characterized by cutaneous or systemic manifestations. Fibrolipomatous hamartoma (FLH) is a condition in which digital overgrowth Fibrolipomatous occurs like ML. It usually presents as an isolated nerve lesion and associated with intramuscular fat deposition. But in ML, in addition to deposition of fat in nerve sheaths, subcutaneous and muscle compartment, there is periosteal involvement leading to the bony changes including exostosis, fatty invasion of the medullary cavity, hypertrophy hypertrophy and ankylosis.20,21

98C. What is the most likely diagnosis? Macrodactyly

98D. What is the pathophysiology of this condition? Several theories exist as to the cause of macrodatyly including abnormal nerve and vascular supply to the digits although none have been proven. Macrodactyly may be associated with neurobromatosis, tuberous sclerosis and Mauci syndrome.

98E. What is the treatmen treatmentt of this conditio condition? n?

113

 The usual presentation is with problems problems associated with cosmesis. However However patients may present with secondary osteoarthritis or compression of neurovascular structures resulting in entrapment syndromes. synd romes.22–24 Surgical usually takes the form of a debulking procedure and is dependent on the patient’ss symptoms, age, and extent and severity of the disease. A conservative patient’ conser vative approach is advocated as growth usually stops at puberty also the incidence of nerve injury following extensive debulking or lesion removal is approximate approximately ly 30% to 50%. A high recurrence rate of 33% to 60% has been reported. In a localised forms of the disease ray amputations have been described.

 

114

Chapter 8 Viva Answers

99A. Describe the radiographs.  The radiograp radiographs hs are AP latera laterall and oblique oblique views centred centred on on the index nger metacarpophalangeal joint (MCPJ). They show a complex dislocation of the MCPJ of the index nger and is usually associated with signicant trauma. These injuries are uncommon and early recognition is crucial in the management of this injury.  The index nger is commonly involved involved and dorsal osteochondral fractures are reported in 50% cases in some series. Radiological features may be subtle and three radiographic views are mandatory. The The AP view may show an ulnar drift of the proximal phalanx. The true lateral is of limited value due to overlap of the other ngers. The The third oblique view is most useful and at times may show interposition of a sesamoid, which is pathognomonic of a complex dislocation of the MCPJ MCPJ..

99B. What is the diagnosis?

Any dislocation associated with a fracture is termed complex. Thus this is a complex dorsal dislocation of the metacarpophalangeal joint of the left index nger.

99C. What are the causes of irreducibil irreducibility? ity? Causes of irreducibility may be interposition of any one of the following structures: • Metacarpal head “button-holed” between the exor tendon (ulnar side) and the lumbrical (radial side); most common cause. • Volar plate • Sesamoid bones • Supercial transverse metacarpal ligament • Natatory ligaments

99D. What is the treatment? Complex dorsal dislocations will almost always require open reduction. Attempts at closed reduction and excessive traction make the “noose” tighter and result in

excessive trauma to the aected digit. Closed reductions may be attempted in the thumb, open injuries or subluxed MCPJs which present with signicant deformity. Excessive traction runs the risk of converting a simple subluxation into a complex dislocation. The preferred treatment for complex dislocations is open reduction.  The MCPJ MCPJ may be approached through through a standard extensile volar approach. approach. In the index nger the radial digital nerve lies in close proximity to the to the rst lumbrical and the dislocated metacarpal head and great care must be exercised as injuries to this nerve are well documented. A key step in reducing the tension around the metacarpal head is to release the A1 pulley thereby allowing a degree of play of the exor tendons. The use of a freer elevator placed under the 1st Lumbrical can be used to replace the dorsally trapped volar plate. Simultaneously the exor tendons are retracted in a volar direction and an assistant applies distal traction holding the index nger nger..

 

References

A dorsal approach may also be used. It has the advantage of being technically simpler and does not involve encroach upon the digital nerve and allows for treating associated dorsal osteochondral fractures. However it does entail resection of the entrapped volar plate and can in theory cause MCP joint instability. However there is no published work to support this view.

99E. What is the prognos prognosis? is? Previous reports are limited to small case series focusing on surgical approaches. Barry et al25 presented four patients who regained nearly full function after open reduction. With a concentric reduction, it seems likely that patients will regain the majority of their pre-injury motion within four to six weeks. Similar good results were reported in seven cases treated with a volar approach, all regaining normal ROM, power and sensibility. sens ibility.26

References 1. 2. 3. 4. 5. 6. 7. 8.

Keith J, Wollstein R. A tailored approach approach to the surgical surgical treatment of cubital tunnel syndrome. syndrome. Ann Plast Surg. 2011;66:637-9. Bynum DK Jr, Gilbert JA. Avulsion of the exor digitorum profundus: anatomic and biomechanical biomechanical considerations. J Hand Surg Am 1988;13:222-7. Leddy JP JP,, Packer Packer JW. JW. Avulsion Avulsion of the profundus tendon insertion in athletes. J Hand Surg Am Am 1977;2:66-9. Garcia-Elias M, Lluch AL, Stanley JK. Three-ligament tenodesis for the treatment of scapholunate dissociation: indications and surgical technique. J Hand Surg Am 2006;31:125-34. Manuel J, J, Moran SL. The diagnosis and treatment treatment of scapholunate instability. instability. Hand Clin Clin 2010;26:129-44. Strauss EJ, Alfonso D, Baidwan G, Di Cesare PE. Orthopedic manifestations and management of of psoriatic arthritis. Am J Orthop (Belle Mead NJ) 2008;37:138-47. Wassel HD. The results results of surgery for polydactyly of the thumb: a review. review. Clin Orthop Relat Res 1969;64:175-93. Yen CH, Chan WL, Leung Leung HB, HB, Mak KH. Thumb polydactyly: polydactyly: clinical outcome after reconstruction. J Orthop Surg (Hong Kong) 2006;14:295-302.

115

9. Miller MD. Review of orthopaedics. Fifth ed. Philadelphia: Saunders Elsevier, 2008. 10. Peeters T, Ching-Soon NG, Jansen N, et al. Functional Functional outcome after repair of distal biceps tendon ruptures using the endobutton technique. J Shoulder Elbow Surg. 2009;18:283-7 11. von Campe A, Mende K, Omaren H, Meuli-Simmen C. Painful nodules and cords cords in dupuytren dupuytren disease. J Hand Surg Am 2012;37:1313-18. 12. Fletcher CD, Theaker JM. Digital pacinian neuroma: a distinctive hyperplastic lesion. Histopathology 1989;15:249-56. 13. Woodru MJ, Waldram MA. A clinical grading system system for Dupuytren’ Dupuytren’s contracture. J Hand Surg Br 1998;23:303-5. 14. Denkler K. Surgical complications complications associated with with fasciectomy for for Dupuytren’s disease: a 20-year 20-year review of the English literature. Eplasty 2010;10:e15. 15. Ferreres A, Llusá M, García-Elías M, Lluch A. A possible mechanism of direct injury to the EPL tendon at Lister’s tubercle during falls with the wrist fully extended. J Hand Surg Eur Vol 2008;33:149-51. 16. Anwar I, Owers KL, Eckersley R. Spontaneous rupture of the extensor pollicis longus tendon. J Hand Surg Br 2006;31:457-8. 17. Chitnis SL, Evans DM. Tendon transfer to restore extension of of the thumb using abductor pollicis longus. J Hand Surg Br 1993;18:234-8. 18. Krengel S, Fustes-Morales Fustes-Morales A, Carrasco D, D, et al. Report of eight cases and review of literature. Pediatr Dermat 2000;17:270–6.

 

116

Chapter 8 Viva Answers

19. Aydos SE, Fitoz S, Bökesoy I. Macrodystrophia lipomatosa of the feet and subcutaneous lipomas. Am J Med Genet A 2003;119:63-5. 20. Jain R, Sawhney S, Berry M. CT diagnosis diagnosis of macrodystrophia lipomatosa: lipomatosa: a case report. Acta Radiol 1992;33:554-5. 21. D’Costa H, Hunter Hunter JD, O’Sullivan G, et al. al. Magnetic resonance imaging in macromelia and macrodactyly. Br J Radiol 1996;69:502-7. 22. Oztürk A, Baktiroğlu L, Oztürk E, Yazgan Yazgan P. P. Macrodystrophia lipomatosa: a case report. Acta Orthop Traumatol Turc 2004;38:220-3 (in Turkish). 23. Watt AJ, Chung KC. Macrodystrophia lipomatosa: a reconstructive reconstructive approach to gigantism of the foot. J Foot Ankle Surg 2004;43:51-5. 24. Brodwater BK, Major NM, Goldner RD, Layeld LJ. Macrodystrophia lipomatosa lipomatosa with associated brolipomatous hamartoma of the median nerve. Pediatr Surg Int 2000;16:216-18. 25. Barry K, McGee H, Curtin J. Complex Complex dislocation of the metacarpophalangeal joint of the index nger: a comparison of the surgical approaches. J Hand Surg 1988;13:466-8. 26. Durakbasa O, Guneri B. The volar surgical approach in complex dorsal metacarpophalangeal dislocations. Injury 2009;40:657-9.

 

Chapter 9 Viva Questions: Children’s orthopaedics 100. Here is the radiograph of an infan infantt who p presented resented with a history of a fall from a chair and inability to move her right arm (Fig. 9.1).  A.  What is the diagnosis diagnosis and how would you manage the the case?

Figure 9.1

 

Chapter 9 Viva Questions

118

 

Here are a photograph and radiograph of a child who presented with an unstable right knee and foot deformities (Figs 9.2a and b). B.  What is the diagnosis diagnosis and how would you manage this this child?

a

b

Figure 9.2a and b

 

Children’s orthopaedics

101. Tis child h has as no complaints complaints but her parent parentss are disappointed disappointed about the shape of her left upper limb one year after an elbow injury (Figs 9.3a and b). Tey  want an explanation explanation and a remedy. remedy.  A. How would you manage the case?

 

119

a

b

Figure 9.3a and b

 

Chapter 9 Viva Questions

120

 

Here is a radiograph (Fig. 9.4) showing a recent fracture of the upper tibial metaphysis in a three-year-old boy. B.  Whose name is associated associated with the injury, how might might it cause a problem and how should it be managed?

Figure 9.4

102. Here ar are e clinical photographs photographs and a radiograph radiograph of a 14-year 14-year-old -old girl who complains of intermittent discomfort and loss of full extension and supination of the right elbow and forearm (Fig. 9.5).  A.  What is the diagnosis diagnosis of the elbow disorder? disorder? B.  What is the underlying underlying condition and how could you confirm this? C. How would you treat the elbow?

 

Children’s orthopaedics

 

121

Figure 9.5

Clinical photograph and radiograph of a five-year-old boy who walks on his left heel (Fig. 9.6). Te foot is painless but the foot deformity is progressive. D.  What are are the deformities? E.  What is the likely likely cause? F. How would you manage the condition?

Figure 9.6

 

122

Chapter 9 Viva Questions

103. Here is a pelvic radiograph of a six-ye six-year-old ar-old child with a d dislocated islocated right hip (Fig. 9.7).  A. How would you manage the condition?

Figure 9.7

 

Here is the chest radiograph of a nine-year nine-year-old -old child (Fig. 9.8). B.  What is the diagnosis diagnosis and how would you address the problem?

Figure 9.8

 

Children’s orthopaedics

104. Tis bab baby y is about about to undergo a an n operat operation ion (Fig. 9.9).  A.  What is the condition condition and the opera operation tion planned for it?

 

123

Figure 9.9

 

Clinical photograph (Fig. 9.10a) and radiograph (Fig. 9.10b) of an active, symptom-free boy with severe genu varum. B. How would you manage the case?

a

b

Figure 9.10a and b

 

124

Chapter 9 Viva Questions

105. A. What A. What is this condition (Fig. (Fig. 9.11), 9.11), how would you manage it and what is the

likely outcome?

Figure 9.11

B. Describe the foot deformities in the photographs (Figs (Figs 9.12a and 9.12a and b). How should they be treated?

a

b

Figure 9.12a and b

 

Children’s orthopaedics

 

125

106. Tis 11-year 11-year-old -old girl presented with a painless swelling of her right ring fin finger ger (Figs 9.13a and b). Te swelling was noted several months ago and it is slowly increasing in size. She is concerned about the appearance.  A.  What is the condition condition and how would yyou ou manage th thee case?

a

b Figure 9.13a and b

 

Here is the radiograph of a baby who has spontaneously developed an increasingly swollen left forearm (Fig. 9.14). Te infant is generally well but is irritable when the arm is moved. B.  What is the diagnosis diagnosis and how would you manage the the condition?

Figure 9.14

 

Chapter 9 Viva Questions

126

107. Here are a clinical photograph and later lateral al radiograph of the spine of a six year-old girl who is of short stature (F (Figs. igs. 9.15a and b)  A.  What is the likely likely diagnosis and wh what at are the manifestations and aetiology of the condition?

a

 

b

Figure 9.15a and b

Tis 12-year 12-year-old -old boy is about to undergo an operation because of progressive  valguss of the  valgu the lef leftt elbow elbow an and d early early ssympt ymptoms oms of ulnar ulnar neu neuritis ritis (Figs. (Figs. 9.16a 9.16a an and d b). B.  What is the condition condition and what op operation eration would you you undertake?

a Figure 9.16a and b

b

 

Children’s orthopaedics

108. A five-year five-year-old -old who has never walked but wants to do so (Fig. 9 9.17). .17). He has fixed flexion deformities of his wrists and knees and severe, stiff talipes equinovarus.  A.  What is the diagnosis? diagnosis? B. How would you manage the case?

Figure 9.17

 

Te parents of this six-week six-week-old -old baby have just noticed a swelling in the right side of his neck (Fig. 9.18). C.  What is the likely likely diagnosis and ho how w would you man manage age the case?

 

127

Figure 9.18

 

Chapter 9 Viva Questions

128

109. Clinical photograp photograph h and radiograph of a 12-year-old 12-year-old girl with a thor thoracic acic kyphosis (Figs 9.19a and b).  A. Describe the radiological features features.. B.  What is the likely likely diagnosis?

a

 

b

Figure 9.19a and b

Here are the clinical photographs of a four-yea four-year-old r-old boy (Figs 9.20a and b). He is of normal intelligence but can only walk with difficulty. C.  What is the condition condition in general terms? D.  What is the diagnosis? diagnosis? E. How would you treat his feet?

a

b

Figure 9.20a and b

 

Children’s orthopaedics

110. Figures 9 9.21a .21a and b show tthe he initial and post-operat post-operative ive radiographs of a ten year-old  year-ol d boy who injured his knee in a road traffic accident. F Figure igure 9.21c is a current radiograph, two years later.  A. Describe the original injury, i njury, the current radiological features and how you  would manage the case.

a

b

c

Figure 9.21a to c

 

Tis 14-year14-year-old old boy, previously normal, presented with a painless prominence of his right scapula following a canoeing holiday (Fig. 9.22). B.  What is the condition, condition, what ar aree the differential diagnoses and, of th these, ese,  which is the most likely in this case?

 

129

Figure 9.22

 

Chapter 9 Viva Questions

130

111. Here is a clinical phot photograph ograph and radiograph of a child who wants his feet corrected (Figs 9.23a and b).  A. How would you manage the case?

a

b

 

Figure 9.23a and b

Here is the clinical photograph of an 11-year 11-year-old -old boy (Fig. 9.24). B.  What is the condition? condition? C.  What are the clinical fea features? tures? D.  What are the principles of ma management? nagement?

 

Children’s orthopaedics

Figure 9.24

 

131

 

Chapter 10 Viva Answers: Children’ss or Children’ orthopaed thopaedics ics 100A. What is the diagnosis and how would you manage the case?  The radiograph shows shows an undisplaced spiral fracture of the humerus. humerus. This would would be unexpected from the history and raises the suspicion of non-accidental injury.  The child should be referred referred to the paediatricians and, if further investigations are indicated, these should include a skeletal survey and a bone scan to look for injuries at different times elsewhere. In this case the chest X-ray showed healing fractures of the ribs (Fig. (Fig. 10.1a) and the bone scan (Fig. 10.1b) showed evidence of activity in several areas, compatible with earlier trauma.

a

b

Figure 10.1a and b

 

134

Chapter 10 Viva Answers

100B. What is the diagnosis and how would you manage this child?  The diagnosis is Larsen’s syndrome, the orthopaedic features features of which are hypermobility with multiple joint dislocations, usually present at birth, and extra bones in the wrist and tarsus. The diagnosis can be confirmed by the double ossification centre of the os calcis, as seen in this case (Fig. 10.2). Function is usually good and most surgeons would not try to reconstruct the  joints, as operative treatment treatment is likely to fail.  The child may therefore therefore be helped by knee and foot orthoses orthoses He should also be monitored through growth for spinal deformity, which can be severe with neurological problems.

Figure 10.2

101A. How would you you manage the case?  The diagno diagnosis sis is cubitu cubituss varus varus follow following ing a supra supracondy condylar lar fractur fracture. e. A full explanation is necessary. Regardless of the initial treatment, there is a significant number of malunions after this injury. It is also important to explain that, although the deformity would be permanent, there is usually no functional disability and correction of the deformity by valgus supracondylar osteotomy is for appearance. As with all cosmetic operations, the patient has to want the appearance improved and their expectations may not be realised because it may not be possible to correct the deformity completely and there will be a scar scar.. Therefore, Therefore, do not offer supracondylar osteotomy unless all these points are understood and the child herself finds the deformity unsightly.

101B. Whose name name is associated associated with the injury, injury, how how might it cause a problem and how should it be managed?  The radiograph radi ograph shows sh ows a Cozen’s fracture of o f the proximal tibial t ibial metaphysi m etaphysis. s.  This fracture fra cture can c an lead to a valgus va lgus angular an gular deformity, de formity, even if the fractu fracture re

 

Children's orthopaedics

appears to be undisplaced. I would manage this patient by taking a full history and performing a thorough clinical examination. I would warn the parents about the risk of later deformity. This may result from asymmetrical growth stimulation of the proximal tibial physis following the injury. in jury. It would be best managed in a long-leg straight cast with varus moulding. Regular radiographs should be taken for the first three weeks and the th e cast wedged appropriately. If a suitable reduction was not possible and the fracture became further displaced, open reduction and removal of periosteum or even the th e pes anserinus from the fracture site, followed by fixation may be necessary to allow for suitable reduction and stabilisation.

102A. What is the the diagnosis of the elbow disorder?  The radiograph of the elbow elbow demonstrates a posterior dislocation dislocation of the radial 1 head.  The radial head looks convex. This probably represents represents a congenital dislocation of the radial head and is often of ten associated with a hypoplastic capitellum. Additionally there may be both shortening and increased bowing of the abnormal radius.

135

102B. What is the the underlying condition condition and how how could you you confirm this? Nail–patella syndrome. Examination and radiographs of the knees are likely to show absence or hypoplasia of the patellae and a pelvic radiograph may show iliac horns. The inheritance is autosomal dominant so siblings and parents may be affected.

102C. How would you treat the elbow?  The symptoms are not bad enough enough for operative treatment treatment but if they became so the radial head could be excised.

102D. What are the deformities? Calcaneo-cavus and tibial bowing.

102E. What is the likely cause? Osteomyelitis with destruction of the lower tibial growth plate.

102F.. How would you manage the condition? 102F  The foot should be made plantigrade plantigrade by correction correction through the ankle and lower fibula. The tibial deformity may be left to mature and remodel but may need correction depending on progress and resources.

103A. How would you you manage the condition?  The radiograph shows shows a right-sided high hip dislocation. There There is delayed ossification of the femoral head and an increased acetabular index. These

 

136

Chapter 10 Viva Answers

findings are consistent with developmental dysplasia of the hip. I would take a thorough history from the parents and the child and perform a full clinical examination. I would enquire about treatment to date, current symptoms and co-morbidities. I would fully explain the condition and the future prognosis.  The child has a myelomeningocele. myelomeningocele. The spinal abnormality, abnormality, ventriculoperitoneal shunt and bowel stasis are obvious. There There is little acetabular dysplasia, indicating the dislocation is i s related to muscle weakness. The child is a non-walker and the position of the hip with regard to sitting and propped standing is good. The child should be managed non-operatively.

103B. What is the diagnosis and how would you address the problem?  The diagnosis is a Sprengel deformity on the right. Management depends on cosmetic and functional disability. In this case the condition is not severe and there is no associated Klippel–Fe Klippel–Feilil anomaly. If the appearance is a problem, it

could be addressed by excision of the upper angle of the scapula with division of any vertebral connection. In more severe cases a vertical scapular osteotomy can be helpful. It is debatable whether more extensive procedures are overall better as scarring can be a major cosmetic disability and function may not be significantly improved. improved.2,3

104A. What is the condition condition and the operation operation planned planned for it?  The condit co ndition ion is const constric riction tion band synd syndrome rome of o f the th e right ri ght lower l ower leg. l eg. The dista d istall limb is significantly oedematous and a small toe is seen. It is possible that there has been auto-amputation of the other toes. Surgical intervention is based on limb salvage with release of the constriction bands and Z-lengthening procedures or amputation of the limb, if it is felt that that salvage is not feasible.

104B. How would you manage the case?  The photograph photograph and the radiograph show bilateral bilateral genu varum. It is important to know the age of the child, as varus is physiological before before the age of two and is definitelyflaring pathological after three. It is i s likely to bedocument physiological as there is symmetric of the femora and tibiae. I would height, weight and percentiles for age. Short stature may point to pathological causes of genu varum, including rickets and skeletal s keletal dysplasias. Other pathological causes include Blount’s Blount’s disease, trauma, infection, polio and spina bifida. Examination includes full assessment of the back, pelvis, hips, knees kn ees and feet. In addition rotational profile of the lower limb would be measured to document any internal tibial torsion. The intercondylar distance is also important to measure to determine progression. Management of this case would again depend on the age. In a two-year-old child, I would ask to see the child in a year’s time with repeat radiographs. Above the age of three, I would investigate for pathological causes and then consider intervention if the deformity was persistent or progressive.

 

Children's orthopaedics

 This could inclu include de bracin b racing, g, guided gui ded par partial tial growth arres arrestt at an older o lder age, or or osteotomy.

105A. What is this condition, how would you manage it and what is the likely outcome?  There is a calcaneovalgus calcaneovalgus deformity due to congenital postero-medial postero-medial bowing of the tibia and an d fibula. The condition should be treated by gentle stretching and simple splintage during infancy. infanc y.  The bowing will correct correct spontaneously duringofchildhood but there betomild be to moderate residual shortening of the order 1–2 cm, which may will need be addressed towards the end of growth.

137

105B. Describe the foot deformities in the photographs. How should they be treated? In the part a photograph, the feet rest in positions of forefoot varus and hindfoot equinovarus. In the part b image, the hindfoot on the baby’s left is correctible, indicating residual forefoot varus, but not on the right, which indicates resistant congenital talipes equinovarus.  The left foot does not require require surgery. The hindfoot will respond to simple stretching. The metatarsus varus is likely to resolve spontaneously over the next few years. If it does not it can be helped by a period of casting or release of abductor hallucis. Major surgery on the asymptomatic mid-foot is rarely justified  The right foot requires requires operative release, release, either a simple tendo achillis lengthening or a more extensive release. Thereafter, Thereafter, any residual forefoot varus should be addressed as described above.

106A. What is the condition and how would you manage the case?  The condition is a benign subungual exostosis. exostosis. It is causing minor distortion of the nail bed but there is no angular deformity and the growth plate is intact.  The exostosis will grow grow in proportion to the rest of the phalnx. Because of the risk of damage to the growth plate with subsequent angulation it would be wise to postpone removal of the exostosis for several years until the physisis is closed or closing. At that time the nail bed could be carefully raised intact to allow excision of the exostosis and replacement of the nail bed in order to prevent distortion of the nail.

106B. What is the the diagnosis and how would would you manage manage the condition? condition?  The condition is Caffey’s Caffey’s disease (Infantile cortical hyperostosis). There are no laboratory tests to confirm the diagnosis which is usually us ually reached by exclusion of other possible conditions, such as osteomyelitis, neoplasm, scurvy, hypervitaminosis A and child abuse.

 

138

Chapter 10 Viva Answers

 The age at presentation is typical and, and, although the swelling may increase increase alarmingly,, the condition is self-limiting and will usually resolve spontaneously alarmingly over 6 to 9 months and leave no long-term sequelae.

107A. What is the likely diagnosis and what are the manifestations and aetiology of the conditio condition? n?  The likely diagnosis is Morquio-Brailsford Morquio-Brailsford disease (mucopolysaccharidosis Type IVA), IV A), in which deficiency of lyzosomal enzymes enz ymes required for the degradation

of polysaccharides leads to accumulation of keratan sulphate and chondroitin 6 sulphate.  The clinical and radiological manifestations manifest ations include includ e dwarfism, dwarfis m, multiple dysostoses, lumbar lordosis, hip dysplasia, odontoid hypoplasia (Fig. 9.15b),  joint laxity, large elbows and an d knees (genu ( genu valgum is common) and flat feet.  This 12-y 12-yearear-old old boy is abou aboutt to und undergo ergo an oper operati ation on beca because use of progressive progressiv e valgus of the left elbow and early symptoms of ulnar neuritis (Figs 10.3a and b).

a

b

Figure 10.3 a and b

107B. What is the condition and what operation would you undertake?  The diagnosis is nonunion of a minimally displaced fracture of the lateral lateral condyle of the humerus sustained sustai ned seven years earlier. Flynn4 outlined 3 criteria for surgical treatment of an established nonunion: 1. A large large metaphyseal metaphyseal fragment fragment 2. Displacement of less than 1 cm cm from the joint surface 3. An open, viable viable lateral condylar physis physis It was thought that reconstruction ofbe therestored elbow was because it ensue. was unlikely that joint congruency could andinadvisable major stiffness would  The operation undertaken was a supracondylar supracondylar closing-wedge osteotomy osteotomy with transposition of the ulnar nerve.

 

Children's orthopaedics

One year later the boy was symptom-free with full movements and the osteotomy had united (Figs 10.4a to c).

139

b

a

c

Figure 10.4a to c

108A. What is the the diagnosis? diagnosis?  The diagnosis is arthrogryposis multiplex congenita (AMC), causing severe severe weakness and stiffness.

108B. How would you manage the case?  The child was managed as follows: follows: •







Full functional assessment with a physiotherapist Clear goals of treatment. The aim was to achieve independent walking with orthoses and crutches. This required correction of his feet and knees. His wrists and hips were not a block to this. Indeed, with AMC, flexion is often the best position for wrists and hands to work together. Skilled anaesthesia to cope with positioning and access to veins and airway Radical surgery. This boy required soft-tissue releases and talectomies for his 5





feet and soft tissue releases,  including posterior capsulotomies along with lower femoral extension osteotomies, for his knees Rehabilitation with a physiotherapist Prolonged splintage In this case the goals of treatment were achieved (Figs (Figs 10.5a and b)

 

140

Chapter 10 Viva Answers

a

b

Figure 10.5a and b

108C. What is the the likely diagnosis diagnosis and how would you you manage the case?  The diagnosis is a benign sternomastoid tumour. tumour. The The aetiology is likely to be traumatic. The condition is right-sided in 75% of cases and a higher proportion of babies were breech deliveries, as in this case. One in seven cases progress to a muscular torticollis. The remainder resolve spontaneously.. This was explained to the parents, no treatment was given and spontaneously the condition resolved spontaneously.6

109A. Describe the radiological features.  There is general osteopaenia. osteopaenia. The The upper and lower vertebrae vertebrae are H-shaped, due to step-like depression of the end-plates, with sparing of their peripheral portions.  There is vertebral collapse collapse due to avascular necrosis. necrosis.

109B. What is the likely diagnosis? In a patient of Afro-Caribbean descent it is likely this represents sickle cell disease. Bone infarcts secondary to sickling of haemoglobin cause the characteristic pattern of spinal collapse known as fish-tail vertebrae.

109C. What is the condition in general terms?  The general appearances appearances are of a syndromic talipes equinovarus. equinovarus.

 

Children's orthopaedics

141

109D. What is the diagnosis?  The diagnosis is Freeman-Sheldon Freeman-Sheldon syndrome (distal arthrogryposis type 2A, craniocarpotarsal dysplasia, “whistling face syndrome”).  The child shows the typical facial features features of microstomia, naso-labial folds, folds, long philtrum, a chin dimple di mple (typically H-shaped) and low-set ears. There is also drooping of the lower eyelids.

109E. How would you treat his feet?  The feet are likely likely to resist treatment and even even with extensive soft-tissue and bony surgery, correction is difficult and long-term splintage essential. There There are special anaesthetic problems with airway and venous access and a risk of malignant hyperpyrexia.

110A. Describe the original injury, the current radiological features and how you would manage the case. Figure 9.21a and b show Salter-Harris Type Type II injury of the lower femur, which, with its location and mechanism, has a high risk of growth plate arrest. Figure 9.21c shows a posterior growth plate arrest and corresponding angular deformity. This is likely to progress and, along with the angulation, leave a limb length discrepancy in the order of 5 cm to 6 cm. MRI would be useful in defining the extent of the growth plate tether. However, an attempt to resect any tether would be technically difficult and the results unreliable  The best option would be to to ablate the remaining growth growth plate and correct correct the angulation and shortening using a circular frame. If such facilities were not available or the child was unsuitable for the method, the angulation could be corrected by supracondylar osteotomy and epiphysiodesis of the contralateral knee.

110B. What is the condition, what are the differential diagnoses and, of these, which is the most likely in this case?  The diagnosis is winged scapula.  The aetiology may be in bone (e.g. (e.g. congenital malformation malformation or subscapular exostosis), joint (e.g. post-inflammatory or traumatic fusion), nerve ner ve (neuropraxia of the long thoracic nerve) or muscle. In this case, although the history suggested a nerve injury, the diagnosis proved to be facio-scapular-humeral (F-S-H) dystrophy. With hindsight, there was already early winging of the left scapula.

 

142

Chapter 10 Viva Answers

111A. How would you you manage the case?  There must be a full history and examination to exclude exclude other conditions, sometimes hereditary, associated with polydactyly polydactyly.. Confirm that child and family want the extra digit removed. In some cases or societies the condition is accepted and even considered lucky. In this case the most normal of the lesser toes in each foot is the th e outermost, although all the toes are abnormal to some degree. It is important to explain pre-operatively that the toes cannot be restored to normal.7 At operation, a wedge resection of the fifth toe and its metatarsal component was undertaken. This allowed the outermost metatarsal to lie easily against the fourth, to which it was sutured. Three weeks in a bandage helped to maintain the position and allow healing.

111B. What is the condition?  The diagnosis is a syndromic type of arthrogryposis, namely multiple pterygium pterygium (Escobar) syndrome.

111C. What are the clinical features? As with arthrogryposis ar throgryposis generally, generally, the children are of normal intelligence and there is amyoplasia, so they are weak (note the wheelchair). Particular features of Escobar syndrome may include: 1. Multiple webs, webs, crossing crossing not only major joints but also involving involving the neck, eyelids and perineum, where there may be genital abnormalities and cryptorchism. 2. Joint contractures contractures in hands and feet, feet, where major foot foot deformities are possible. 3. Decreased lung capacity. 4. Cleft palate. 5. Fac Facial ial features features such as ptosis, ptosis, sloping eyelids, low-set low-set ears, long filtrum and retrognathia. 6. Scoliosis.

111D. What are the principles of management?  The management of the deformities is dominated by by weakness and anaesthetic considerations. There must be a full functional assessment and clearly defined goals of treatment. In the upper limbs early corrective splintage can be useful but surgery is only indicated occasionally as it would not improve function. In the lower limbs, operative correction of the deformities should be considered in order to allow walking or at least standing for transfers. The operations can be complex and require good anaesthetic support and prolonged splintage postoperatively.

 

Children's orthopaedics

References 1. 2. 3. 4. 5. 6. 7.

Almquist EE, Gordon LH, LH, Blue AI. Congenital Congenital dislocation of the head of the radius. J Bone Joint Surg [Am] 1969;51-A:1118-27. Miller MD. Review of orthopaedics. Fifth ed. Philadelphia: Saunders Elsevier, 2008. Canale ST, Beaty JH, eds. Campbell’s Operative Orthopaedics. 11th ed. Philadelphia: Mosby Elsevier; Flynn JC.2007. Nonunion of slightly slightly displaced fractures of the lateral lateral humeral condyle in children: an update. J Pediatr Orthop 1989;9:691-6. Lourenco AF AF,, Dias LS, Zoellick Zoellick DM, Sodre H. Treatment of residual residual adduction deformity in clubfoot: the double osteotomy. J Pediatr Orthop 2001;21:713-18. McDonald D. Sternomastoid tumour and muscular torticollis. torticollis. J Bone Joint Surg [Br] 1969; 50-B:432-43. McCarthy GJ, Lindaman L, L, Stefan M. Pedal Pedal polydactyly: an overview with with case report. J Foot Ankle Surg 1995;34:577-82.

143

 

Chapter 11 Viva Questions: Basic science 112. A.  A.  What is your thromboprophyla thromboprophylaxis xis regime for a 72-year-old 72-year-old man who is scheduled for a total hip replacemen replacement? t? B.  What is your thromboprop thromboprophylaxis hylaxis regime for a 72-year-old 72-year-old man who is scheduled for a total knee replacemen replacement? t? C.  What is the evidence evidence for previous two answers D. Is there a role for routine chemical thromboprophylaxis thromboprophylaxis in patients with ankle fractures? If yes, what is your regime for the same? E.  What is the mechanism mechanism of action of aspirin, low-molecular-weight low-molecular-weight heparin and warfarin? F. Do you know of any oral anticoagulant that can be used as a thromboprophylactic thrombopr ophylactic agent in joint replacemen replacementt surgery? What is the mechanism of action of this agent? G.  What are the contraindications contraindications to the use of chemical thromboproph thromboprophylaxis ylaxis in  joint replacement replacement surgery? 113. A.  A.  What are the types of bone graft that you are aware of? B.  What are the different types types of synthetic grafts grafts that you you are aware aware of? C. Compare and contrast the properties of synthetic bone grafts. D. Explain the term creeping substitution. E. Explain the process of collection and storage of donor femoral heads to be used as bone graft in future. 114. A. A.   What do you understand by by the term osteoporosis? B.  What is the WHO WHO definition for osteoporosis? osteoporosis? C.  What are the risk factors for for the development of osteoporosis? D.  What is the pathophysiology pathophysiology of osteoporosis?

E. How would you investigate a patient with suspected osteoporosis? F.  What are the changes in peak bone mass with respect to age? age? G. How do you classify osteoporosis? 115. A.  A.  How would you differentiate osteoporosis osteoporosis from osteomalacia? B.  What advice would you give a patient patient in terms of prevention prevention of osteoporosis? osteoporosis? C. Is having a national screening programme for osteoporosis worthwhile? wor thwhile? Why?

 

146

Chapter 11 Viva Questions

D. How would you treat an established case of osteoporosis? E.  What are the complications/side-effects complications/side-effects of therapy therapy with bisphosphonates? bisphosphonates? 116. A. A.   What is the pathophysiology pathophysiolo gy of Paget’s disease disease?? B.  What is the aetiology aetiology of Paget’ Paget’ss disease? C.  What is the mode mode of presentation presentation in patients with Paget’ Paget’ss disease? D.  What are the poten potential tial meta metabolic bolic comp complica lications tions in patient patientss with Pa Paget’ get’ss disease? E. How would you investigate i nvestigate a patient with Paget’ Paget’ss disease? F.  What is the treatment treatment of established established Paget’ Paget’ss disease? 117. A.  A.  What is the pathophysiology pathophysiology of rheumatoid arthritis? B.  What is the aetiology aetiology of rheumatoid rheumatoid arthritis? C.  What are the ARA criteria for the diagnosis diagnosis of rheumatoid rheumatoid arthritis? D.  What are the potential extra-articular extra-articular manifestations manifestations in patients patients with rheumatoid arthritis? E. How would you investigate a patient with rheumatoid arthritis? F.  What are the poor prognostic indicators for patients patients with rheumatoid rheumatoid arthritis? 118. A.  A.  What are the basic basic principles behind the use of a tourniquet in upper and lower limb surgery? B. How do you decide on inflation pressures in upper and lower limb surgery? C.  When would you not use a tourniquet? D.  What are the complications complications of the use of a tourniquet? E.  Wha  Whatt do do you you und unders erstan tand d by by the the ter term m post-tourniquet syndrome ? How is it treated? 119. Operating theatres  A.  Whilst designing a new theatre theatre complex, complex, what are are the different zones that one has to bear in mind and why? B.  What are the sources of contamination in an operating theatre? theatre?

C. How is the risk of this contamination reduced? D.  What are the types of ventilation ventilation system that that you are aware of and describe each in detail? E.  What do you understand by the the term "laminar air flow"? F.  What  What is is the the effe effect ct of lam lamina inarr air air flow on the risk of inf infect ection ion in joi joint nt replacements? 120. A.  A.  How would you assess that your laminar airflow operation operation theatre is suitable for performing joint replacement replacements? s? B.  What are the microbiological microbiological requirements requirements for continuing to perform joint replacements replacemen ts in your operation theatre?

 

Basic science

C.  What antibiotics would you administer as prophylaxis prophylaxis prior to performing performing a lower limb joint replacement and why? D.  What is the mechanism mechanism of action of flucloxacillin and vancomycin? E. Do pre-operative prophylactic prophylactic antibiotics reduce the risk of infection following a joint replaceme replacement? nt? F.  What is the evidence evidence for your answer to question 5? 121. A 23-year-old 23-year-old man presents with acute onset of pain and stiffness in his right elbow, in particular supination and pronation of his forearm. He had a fracture of his radial neck fixed nine years ago and the metalwork was subsequently removed (Figs 11.1a and b). Tere were other features, f eatures, which suggested acute sepsis on this presentation.  A. Describe the radiograph radiographs. s. B.  What is the differential differential diagnosis? C.  What other investigations investigations would you you arrange? D. How would you manage this patient? E. How would you classify osteomyelitis?

147

a

b

Figure 11.1a and b

F. Describe the gram-staining technique for identification of bacteria and how do youstaining? differentiate between gram-p gram-positive ositive and gram-negative bacteria on gram 122. A 64-year-old 64-year-old male who has had a total knee replacement a year ago presents presents  with a history history of pain pain and swel swelling ling with some disch discharge arge from his knee (Fi (Fig. g. 11.2) 11.2)..  A. Describe the clinical photograph. B.  What is the differential differential diagnosis? C.  What investigations investigations would you like like to arrange? D. How would you manage this patient?

 

148

Chapter 11 Viva Questions

Figure 11.2

E.  What do you understand by the the term biofilm? F. Describe the constituents of a biofilm and the stages in its i ts formation. G.  Why are antibiotics antibiotics ineffective in the presence presence of a biofilm? 123. A. A.   Describe the photograph photograph (Fig. (Fig. 11.3). 11.3). B.  What do you think is the mechanism mechanism of failure in this situation? C. How are ceramics manufactured?

D. Discuss the biomechanical properties of ceramics ceramics.. E.  What is the mode mode of failure in ceramics ceramics with respect respect to them being used as a bearing surface? F. Compare ceramic-on-polyethylene with ceramic-on-ceramic as a bearing couple.

Figure 11.3

 

Chapter 12 Viva Answers: Basic science 112A. What is your thromboprophylaxis regime for a 72-year-ol 72-year-old d man who is scheduled for a total hip replacement? Combine mechanical prophylaxis using anti-embolism stockings with pharmacological prophylaxis prophylaxis using low molecular weight heparin. This regime should continue for 4 weeks after af ter surgery. In addition, use an intra-operative intermittent calf compression device on the non-operated non- operated side. This approach approach should be tailored to each patient and all patients should undergo risk assessment as part par t of a pre-operative work-up. work-up. This should identify patients who require other prophylaxis. Be aware of NICE guidelines which outline other acceptable agents, including oral anticoagulants such as riviroxiban.

112B. What is your thromboprophylaxis regime for a 72-year-ol 72-year-old d man who is scheduled for a total knee replacement?

For elective knee replacement surgery the same protocol as above should be followed but need only be contiuned for 10–14 days.

112C. What is the evidence for your previous two answers. NICE guidelines – as above.1

112D. Is there a role for routine chemical chemical thromboprophylaxis thromboprophylaxis in patients with ankle fractures? If yes, what is your regime for the same? Consider offering pharmacological VTE prophylaxis to patients with lower limb plaster casts after evaluating the risks and benefits based on clinical discussion with the patient. Offer O ffer LMWH until lower limb plaster cast removal. Following ankle fracture surgery LMWH is generally given until the plaster is Following removed. For For ankle fractures treated non-operatively this is i s not normally the case.

112E. What is is the mechanism of action of aspirin, aspirin, low-molecularlow-molecularweight heparin and warfarin? Aspirin is non-selective and irreversible COX (cyclo-oxygenase) inhibitor. Normally COX produces produces prostaglandins, most of which are pro-inflammatory,

 

150

Chapter 12 Viva Answers

and thromboxanes, which promote clotting. Low-dose, long-term aspirin use irreversibly blocks the formation of thromboxane A2 in platelets, producing an inhibitory effect on platelet aggregation. aggregation. LMWHs inhibit the coagulation process through binding to antithrombin via a pentasaccharide sequence. This binding leads to a conformational change of antithrombin which accelerates its inhibition of thrombin (factor IIa) and activated factor X (factor Xa). Once dissociated, the LMWH is free to bind to another antithrombin molecule and subsequently inhibit more thrombin. Warfarin inhibits the vitamin K-dependent synthesis of biologically active forms of the calcium-dependent clotting factors fac tors II, VII, IX and X.

112F.. Do you know of any oral 112F oral anticoagulant that can be used as a thromboprophylactic agent in joint replacement surgery? What is the mechanism of action of this agent? Dabigatran acts by directly inhibiting the enzyme thrombin, the enzyme which converts fibrinogen into insoluble strands of fibrin (part of the clotting cascade).

112G. What are the contraindications to the use of chemical thromboprophylaxis in joint replacement surgery? Contraindications to chemical thrmoboprophylaxis include:



















Active bleeding Acquired bleeding disorders (such as acute liver failure) Concurrent use of anticoagulants known to increase the risk of bleeding (such as warfarin with international normalised ratio [INR] higher than 2) Lumbar puncture/epidural/spinal anaesthesia expected within the next 12 hours Lumbar puncture/epidural/spinal anaesthesia within the previous 4 hours Acute stroke  Thrombocytopenia (platelets (platelets less than 75 x 109 /l) Uncontrolled systolic hypertension (230/120 mmHg or higher) Untreated inherited bleeding disorders (such as haemophilia and von Willebrand’ss disease) Willebrand’

113A. What are the types of bone graft that you you are aware aware of? Bone grafts may be autografts or allografts and may consist of cancellous or cortical bone. Autografts involve utilising bone obtained from the same individual receiving the graft. Bone can be harvested from a myriad of sites. An additional surgical site is often required, in effect adding another potential location for postoperative pain and complications. Autografts are osteoinductive (able to recruit host mesenchymal cells to differentiate into osteoblasts to make new bone), osteoconductive (form a scaffold enabling blood vessel ingrowth and new bone formation) and osteogenic (the ability of cellular elements within the graft to make new bone).

 

Basic science

Allograft is harvested from an individual other than the one receiving the graft. Allograft bone is taken from cadavers. There are three types of bone allograft available, fresh or fresh-frozen fresh-frozen bone, freeze-dried bone allograft and demineralized freeze-dried bone allograft. Allografts are osteoconductive and may be osteoinductive. Cancellous bone graft is used for osteogenesis. It is commonly used for treating non-unions and cavitary defects as it is i s quickly remodelled and incorporated. Cortical bone graft is used for structural support and is slowly incorporated. Heterogenous bone from another species was trialled but with poor results thus resulting in its withdrawal. wi thdrawal.

113B. What are the different types of synthetic grafts that you are aware of? Synthetic bone grafts or graft substitutes are commercially available. The main constituents of these are either calcium triphosphate, hydroxyapatite, calcium carbonate and calcium sulphate. These often contain bone morphogenic

151

proteins (BMPs) and demineralised bone matrix. The composition varies between commercial products.

113C. Compar Compare e and contrast contrast the properties properties of synthetic bone grafts. grafts. Osteoconductive grafts include calcium sulphate, hydroxyapatite, ceramics, calcium phosphate, collagen and other polymers. Osteoinductive grafts include demineralised bone matrix, bone morphogenic proteins and growth factors. Composite grafts may combine these properties.

113D. Explain the term creeping creeping substitution. substitution. Creeping substitution is the process by which cancellous bone graft is incorporated. It refers to osteoblasts laying down new bone over the old grafted bone, which is subsequently resorbed.

113E. Explain the process of collecti collection on and storage of donor femoral heads to be used as bone graft in future. A femoral head from a living donor undergoing a total hip replacement is packaged and frozen within 24 hours of donation. Chips from the bone taken by the retrieving surgeon are placed into aerobic and anaerobic broths for culture. The product is only used if free from microbial growth. The sample is quarantined for 180 days to retest the donor for serological markers or tested at time of donation by PCR technology for Hepatitis and HIV in addition to the routine serology. It is stored at less than -40°C. This product is stored in two sterile plastic containers within an outer bag. Delivery is in a disposable transport box containing dry ice (solid carbon dioxide) validated to keep the graft frozen until the date and time provided on the box. It is delivered direct to the point of use e.g. theatre.2,3

 

152

Chapter 12 Viva Answers

114A. What do you understand by the term osteoporosis? Osteoporosis means there reduced bone mineral density (BMD) but there is no problem with bone mineralisation.

114B. What is the WHO definition for osteoporosis? Osteoporosis is defined by the World Health Organisation (WHO) as a bone mineral density that is 2.5 standard deviations or more below the mean peak bone mass of an average young, healthy adult as measured by the DEXA scan (dual energy X-ray absorptiometry).

114C. What are the risk factors for for the development development of osteoporo osteoporosis? sis? Risk factors can be non-modifiable and modifiable: •

Non-modifiable risk factors: Age - BMD decreases, and consequently the risk of osteoporosis increases with





















age. Gender - Women are at greater risk of osteoporosis as they have smaller bones and hence lower total bone mass. Additionally, women lose bone more quickly following the menopause, and typically live. Ethnicity - Afro-Caribbean women have a higher BMD than white women at all ages due to a higher peak bone mass and an d slower rate of loss. Reproduction factors - A late menopause or short time from menopause to BMD measurement are associated with higher BMD. BMD decreases most rapidly in the early postmenopausal years. Current use of oestrogen replacement therapy is associated with a higher BMD. Family Fami ly history of osteoporosis Modifiable risk factors: Weight - Higher risk with low weight. Smoking Alcohol Exercise Diet Medications - Steroids, etc.

114D. What is the pathophysiology of osteoporosis? Bone resorption is always followed by bone formation, a phenomenon referred to as coupling. In osteoporosis, this coupling mechanism is thought to be unable to keep up with the constant microtrauma to trabecular bone. Osteoblasts not only secrete and mineralise osteoid but also appear to control the bone resorption carried out by osteoclasts. Osteoclasts require weeks to resorb bone, whereas osteoblasts need months to produce new bone.  Therefore, any process that t hat increases in creases the rate of bone remodelin remodeling g results in net bone loss over time. 4

 

Basic science

114E. How would you investigate a patient with suspec suspected ted osteoporosis? Initial work up would include a full history including family history, h istory, gynaecological history if appropriate, medications taken, history of previous fractures. Full Full examination would and blood tests to rule out other bone pathologies would be undertaken (calcium, phosphate, alkaline phosphatase levels) and possible urinary tests. Once other pathologies had been excluded a DEXA scan would confirm the diagnosis of osteoporosis.

114F.. What are the changes in peak bone mass with respect to age? 114F BMD increases until 25 then remains high until 45 to 50 years of age. BMD then reduces more rapidly after 50 in women (due to menopause) and steadily

153

decrease in men.

114G. How do you classify osteoporosis?  The disease may be classified as primary type 1, primary type 2, or secondary.  The form of osteoporosis osteoporosis most common in women after menopause is referred to as primary type 1 or postmenopausal osteoporosis. Primary type 2 osteoporosis or senile osteoporosis occurs after age 75 and is seen in both females and males at a ratio of 2:1. Finally, secondary osteoporosis may arise at any age and affect men and women equally. This form of osteoporosis results from chronic predisposing medical problems or disease, or prolonged use of medications such as steroids.

115A. How would you differentiate osteoporosis from osteomalacia osteomalacia?? Both osteoporosis and osteomalacia may present with bone fractures. Typically Typically,, osteoporosis is painless and insidious until a fracture develops. It is commoner with advancing age. Characteristically, osteomalacia is a painful bone disorder at onset, which can present at any age. Osteomalacia patients may report a history hi story of renal failure, anticonvulsant use, or malabsorption. Osteoporosis typically presents with a normal serum calcium, phosphorus, alkaline phosphatase, vitamin D, and PTH. In contrast, osteomalacia is characterised by hypophosphataemia, hypocalcaemia, increased alkaline phosphatase levels, low levels of vitamin D metabolites, and secondary hyperparathyroidism. hyperparathyr oidism. Urinary calcium levels may be normal in osteoporosis but are often low in osteomalacia. Both conditions appear as low bone mass on radiographs and DEXA scan. s can. However, However, specific radiological findings unique to osteomalacia include Looser pseudofractures. On x-ray, the coarseness of the trabeculae in osteomalacia may differentiate the two diagnoses.5

115B. What advice would you give a patient in terms of prevention of osteoporosis? In both osteoporosis and osteomalacia bone mass may be decreased, but in osteoporosis mineralisation is normal, whereas it is deficient in osteomalacia.

 

154

Chapter 12 Viva Answers

Prevention of osteoporosis includes reduction of risk factors. Important Impor tant messages would be to avoid smoking, smok ing, excess alcohol and drug abuse. A healthy balanced diet and encouraging some load-bearing exercise is also important. Prevention of falls in at-risk patients may also minimise osteoporotic fragility fractures.

115C. Is having a national screening programme for osteoporosis worthwhile? Why? Osteoporosis is an important condition that causes more than 200 000 fractures

each year at a cost to the NHS of more than £940m. It would therefore seem that a national screening programme would be of benefit. However, However, according 6 to the Wilson-Jugner criteria,  screening the entire population would not be worthwhile. This This is because there is not really an early stage of osteoporosis that would require treatment to prevent a late stage. The use of bone density assessment in selected individual patients is however important in reducing the prevalence of osteoporosis, prevention and therefore th erefore reduction of fragility fractures. Those Those who would benefit from screening would be women with a premature menopause; people on steroids for an extended period; women who have suffered hip fractures and elderly people with a stoop or loss of height.

115D. How would would you treat an establis established hed case of osteoporosis?  Treating osteoporosis includes advising patients to stop smoking, excess  Treating alcohol and to ensure that they have a healthy balanced diet with moderate load-bearing exercise. Options for preventing preventing bone loss include calcium and Vitamin D supplementation, oestrogen therapy (HRT), bisphosphonates (inhibits osteoclasts), calcitonin and selective oestrogen receptor modulators (e.g. raloxifene). raloxifene). Stimulation of bone formation could include sodium fluoride (stimulate osteoblasts), recombinant PTH and strontium. The specific combination of therapies would vary from patient to patient.

115E. What are the complicatio complications/side-effects ns/side-effects of therapy with bisphosphonates?

Oral bisphosphonates are associated with gastric irritation and oesophageal ulceration. It is recommended that they are taken after food and that the patient should remain upright for 30 to 60 minutes after taking the medication. Osteonecrosis of the jaw is a complication associated with intravenous bisphosphonates and is commoner in patients having dental surgery involving the jaw. There have been reports of patients having severe muscle, joint, and/or bone pain after taking bisphosphonate medications.  This compli complicatio cation n may aris arise e days, days , months, mon ths, or even eve n years yea rs after a fter star starting ting bisphosphonate therapy. Atypical femoral fractures have been reported in patients receiving long-term bisphosphonates and any significant thigh pain should be investigated in these patients. In the USA, the FDA has commented on an association between a higher prevalence of atrial fibrillation in patients taking bisphosphonates.

 

Basic science

116A. What is the pathophysiology of Paget’s Paget’s disease?  The pathophysiology pathophysiology of Paget’s Paget’s includes increased osteoclast size and number. number.  This results in increased bone resorption resorption with haphazard osteoblastic bone formation. Bone is enlarged, hypervascular, hypervascular, deformed and biomechanically weak.

155

116B. What is the aetiology of Paget s disease? Aetiology is largely unknown, but one theory includes a viral origin as Pagetic osteoclasts have been shown to contain mRNA from paramyxoviruses paramyxoviruses and canine distemper virus.

116C. What is the mode of presentation in patients’ with Paget’s Paget’s disease? Paget’s disease is usually diagnosed in the fifth decade of life and manifests with Paget’s wi th pain. They may progress to degenerative joint disease, d isease, long bone deformity, cranial nerve compression, high output cardiac failure or a pathological fracture.

116D. What are the potential metabolic complications complications in patients with Paget’s disease?  The potential metabolic complications complications include a raised alkaline phosphatase, raised serum acid phosphatase and raise urinary urinar y hydroxyproline. hydroxyproline.

116E. How would you investigate a patient with Paget’ Paget’ss disease? Patients with suspected Paget’s are investigated investigated with a full history and examination, radiographs and possibly an isotope bone scan to determine the extent and activity activit y of the Paget’s disease process. Blood tests including a full bone and liver profile should be performed. per formed. Urinary tests for hydroxyproline should also be undertaken.

116F.. What is the treatment of establishe 116F established d Paget’ Paget’ss disease? Most patients with Paget’ Paget’ss disease require no treatment. Medical management is directed at lowering osteoclast activity and numbers. This can be done with calcitonin or bisphosphonates. Pathological fractures fractures of the lower limbs can be extremely hypervascular and targeting of osteoclast function can reduce peri-operative blood loss. Otherwise treatment is directed with dealing with the subsequent complications associated with Paget’ Paget’ss disease.

117A. What is the pathophysiology of rheumatoid arthritis? Rheumatoid arthritis (RA) is an auto-immune systemic inflammatory disorder.  There is a synovial inflammatory response (pannus) around around the joint capsule.  This inflammatory tissue leads to erosive arthritic changes within joints due to destruction of the articular cartilage.

 

156

Chapter 12 Viva Answers

117B. What is is the aetiology of rheumatoid rheumatoid arthritis?  The aetiology of disease is obscure; auto-immunity auto-immunity has a large role in development of RA. The The disease may arise as a result of:







Age: The average average age of onset of disease are about 40, but it may occur at all ages. It is less common before puberty. Gender: Females are affected three times more than males. Genetic: Individuals with HLA-DR1 or HLA-DR4 serotypes are at risk of developing RA. Concordance of RA in identical twins t wins is approximately 15%

117C. What are the ARA criteria for the diagnosis diagnosis of rheumatoid rheumatoid arthritis? Revised ARA Criteria for the Classification of Rheumatoid Arthritis (RA).7 For classification purposes, a patient is said to have RA if he or she has satisfied at least four of the following seven criteria. Criteria 1 through 4 must have been present for at least six weeks. Patients with two clinical diagnoses are not excluded. Designation as classic, definite, or probable RA is not to be made. 1. Morning stiffness: Morning stiffness in and around the joints, lasting at least hourof before improvement. 2. one Arthritis threemaximal or more more joint areas: At least three joint areas simultaneously have had soft tissue swelling or fluid (not bony overgrowth alone) observed by a physician; the 14 possible joint areas are right or left proximal interphalangeal (PIP) joints, metacarpophalangeal (MCP) joints, wrist, elbow, knee, ankle, and metatarsophalangeal (MPT) joints. 3. Arthritis of hand joints: At least one one area swollen swollen (as defined above) in a wrist, MCP or PIP joint. 4. Symmetric arthritis: Simultaneous involvement involvement of the same joint areas (see 2 above) on both sides of the body (bilateral involvement of PIPJs, PIPJs, MCPJs, MCPJs, or MTPJs MTP Js is acceptable without absolute symmetry). 5. Rheumatoid nodules: nodules: Subcutaneous Subcutaneous nodules, over bony prominences, or extensor surfaces, or in juxta-articular regions, observed by a physician. 6. Serum rheumatoid rheumatoid factor: Demonstration of abnormal amounts of serum rheumatoid factor by any method for which the result has been positive in < 5% of normal control subjects. 7. Radiographic changes: Radiographic Radiographic changes typical of RA on posteroanterior hand and wrist radiographs, which must include erosions or unequivocal bony decalcification localized to or most marked adjacent to the involved  joints (osteoarthritis changes alone do not qualify).

117D. What are the potential extra-articular extra-arti cular manifestations in patients with rheumatoid arthritis? •









Cardiovascular – atherosclerosis, pericarditis, IHD Pulmonary – pulmonary fibrosis, pleural effusions Abdominal – splenomegaly Skin – vasculitis, Raynauds syndrome, nodules, Sjögrens syndrome Eye – scleritis, episcleritis

 

Basic science

157

117E. How would you investigate a patient with rheumatoid arthritis? A blood test for rheumatoid factor (RF) is undertaken. A negative RF does not rule out RA. Other blood tests are usually done to allow for other causes of arthritis, such as lupus erythematosus. The ESR, CRP, CRP, full blood count, renal function, liver enzymes and other immunological tests (e.g. antinuclear antibody/ANA) are all performed. per formed. If there is small joint disease then radiographs of the hands or feet may be taken. There There may be no radiographic changes early on. Radiographs of other symptomatic joints would be obtained as well.

117F.. What are the poor prognostic indicators for patients with 117F rheumatoid rheumatoi d arthritis? •















Poor prognostic factors include: Persistent synovitis Early erosive disease Extra-articular findings Positive rheumatoid factor Family history of RA Poor functional status Socioeconomic factors Elevated ESR/CRP

118A. What are the basic principles principles behind the use of a tourniquet tourniquet in upper and lower limb surgery?  Tourniquets  Tourniq uets help h elp provide provid e a bloodless blo odless field during d uring surger surgery, y, by eliminating elimin ating arterial flow distal to the tourniquet. They can be non-pneumatic (used for digits) or pneumatic which can be automatic (operate from an air line or electric pump) or non-automatic (hand-operated pump). They should be well padded, of appropriate size and shielded from the surgical prep, prep, which could lead to a burn.

118B. How do you decide on inflation pressures in upper and lower limb surgery?  There is no absolute value, value, but a combination of patient patient age, soft-tissue condition, circumference circumference of limb and comorbid medical conditions should be used to guide inflation pressures. As a rule of thumb in the upper limb, a pressure of 50 mmHg more than systolic is used and in lower limb surgery a value double the systolic is used.

118C. When would would you you not use a tourniquet? tourniquet? A tourniquet is contraindicated in severe crushing injuries or poor skin quality, sickle cell disease and in i n significant peripheral vascular disease. Also the use of tourniquets during the treatment of infection, open fractures and intramedulllary nailing is often avoided.

 

158

Chapter 12 Viva Answers

118D. What are the complications of the use of a tourniquet? Complications can be divided into local and systemic:





















Local Compression neurapraxia Bone and soft-tissue necrosis Direct vascular injury Post-operative swelling/stiffness Wound haematoma/infection Systemic Cardiorespiratory Cardiorespirat ory decompensation Increased CVP Deep-vein thrombosis Cerebral infarction Altered acid-base balance

118E. What do you understand by the term  post-tourniquet syndrome syndrome? How is it treated?  This is a tourniquet-induced skeletal muscle ischaemia. Oedema, stiffness, pallor,, weakness and numbness can be noted. It is treated with removal of the pallor tourniquet and supportive measures such as ensuring that the patient is well hydrated and haemodynamically stable.

119A. While designing a new theatre complex, what are the different zones that one has to bear in mind and why? Operating theatre zones: •









An outer, or general access zone for patient reception area and general office. A clean, or limited access zone between the reception bay and theatre suite, and dispersal areas, corridors and staff rest room. Restricted access zone, for those properly clothed personnel engaged in operating theatre activities, including anaesthetic room, utility and “scrub “scrub up” rooms. An aseptic operating zone – the operating theatre. This This keeps the number of people moving through the operating zone to a minimum, as the bacteriological count is related to the number of persons and their movement.  This also allows the operating zone zone to be separate and to enable control of ventilation, air filtration and temperature, humidity and light. Disposal zone.

119B. What are the sources of contamination in an operating theatre? Sources of contamination: 1. Internal: from from the patient themselves themselves e.g. e.g. the skin of the patient and bacteraemia. 2. External: a. Airborne pathogens pathogens – airborne particles mainly from from theatre theatre personnel. personnel.

 

Basic science

b. Human – theatre theatre personnel, other than airborne airborne pathogens e.g. e.g. direct direct contact or transfer via inanimate object. c. Instruments/equipment/protheses – both disposable/single use and reusable. d. Theatre fixtures and fittings.

119C. How is the risk of this contamination reduced? In order to reduce the risk of contamination British Orthopaedic Association guidelines recommend:8 1. The level level of sterile precautions precautions required required to perform orthopaedic surgery safely are higher than those for surgery involving the bowel, infected body cavities, contaminated wounds and other soft-tissue surgery. 2. The following following measures measures must be enforced enforced when material is implanted in bone, major joints are opened or bone is exposed. 3. The use of clean air theatres, installed, installed, maintained and checked according according to NHS standards is considered essential for orthopaedic surgery. 4. All staff in the operating theatre theatre suite, suite, including the anaesthetic room and corridor, must adhere to existing high standards of theatre corridor, th eatre discipline and follow established procedures which include: a. All hair to be be kept covered at all times. b. Masks to be worn at all times within the operating theatre and lay-up room. c. Street clothes and clothes worn worn outside the operating operating theatre theatre suite, suite, including shoes, must not be worn within the theatre suite. d. Staff may only only enter or leave the operating theatre through through clearly identified doors so that air within the operating theatre is not disturbed needlessly. e. The number of people within the operating operating theatre must be kept to the minimum required to function safely s afely.. f. Tr Traffic affic from from dirty areas areas and within the lay-up room must be rigidly controlled. g. Drapes and gowns gowns must be made of impervious material. material. Thin cotton drapes and gowns have no place in orthopaedic surgery. In addition to these guidelines, the use of ultraclean air and prophylactic antibiotics greatly reduces infection rates. Lidwell et al9 in the MRC trial reported the lowest incidence of deep infection (0.06%) was achieved using ultra-clean air,, body exhaust systems and prophylactic antibiotics. air















Further measures to reduce contamination include:10 Screening and eradication of MRSA Day of surgery admission onto specific wards Body exhaust systems Double-gloving Effective surgeon scrubbing and use of skin antiseptics e.g. chlorhexidine gluconate 4% Patient skin preparation with antiseptics If the operative site is to be shaved, this should be performed per formed as close to the operation as possible

159

 

160

Chapter 12 Viva Answers





Equipment cleaning, disinfection and sterilisation Surgical technique, e.g. shorter operative time

119D. What are the types of ventilation system that you are aware of? Describe each in detail.  Types of ventilation ventilation system: Source of air for ventilation; air is usually taken in at the roof level of the theatre suite. It is drawn by a series of fans through filters capable of removing bacteria carrying particles. It is also humidified and warmed or cooled. High-efficiency particulate air (HEPA) filters are employed. These These are capable of filtering particles par ticles of 0.5 microns in size with 99.97% efficiency. Generally operating theatres are equipped with positive pressure (plenum) ventilation systems. In this system the pressure inside the theatre is greater than outside. Clean air is fed via wall or ceiling diffusers and let out of vents placed  just above floor level. level. Air also passes out around doors doors and other openings.  The opening of doors and the movement movement of personnel make this system less efficient. Standard positive pressure ventilated operating theatres deliver around 15 to 25 air changes per hour. Laminar airflow: see the next answer.

119E. What do you understand by the term “laminar air flow”? Laminar airflow: this involves the entire body of air within a designated space moving with uniform velocity in a single direction along parallel flow lines.  There are three three main types of theatre airflow: •

Horizontal laminar flow





Vertical laminar flow flow (Howorth enclosures) Ex-flow or exponential  Theatres are usually usually designed with a vertical downward airflow concept. The flow of air is around 0.3 m/s. Horizontal laminar flow; HEPA filters form a wall, or part of a wall. The The positioning of potential objects to the laminar flow is important impor tant in order to avoid obstructions (e.g. theatre personnel, image intensifiers). Vertical laminar flow; air is i s passed through HEPA filters in the ceiling and directed downwards towards the operative field in a vertical verti cal direction. The area of HEPA HEPA filters on the ceiling is enclosed by panels extending towards the floor (should extend to within 2 m of the floor). Objects such as theatre lights form obstacles and create turbulence. Also, personnel standing under the edge of the enclosure deflect the vertical airflow inwards (peripheral entrainment), also deflecting contaminated air inwards towards the operative site.

Ex flow or exponential flow (Howorth enclosures); this describes the flow of clean air downwards and outwards in the shape of an inverted trumpet. This eliminates the problem of peripheral entrainment. This system is in theory more efficient than laminar flow, and requires fewer changes of air per hour.

 

Basic science

119F.. What is the effect of laminar air flow on the risk of infection in 119F  joint replacements? replacements? In the MRC trial, Lidwell et al8 found that ultra-clean air reduced the risk of deep  joint sepsis in trial arthroplasty by a factor of 2.6 compared with controls. controls. When all groups in the were considered together the analysis showed deep sepsis after 63 out of 4133 operations in the control group (1.5%) and after 23 out of 3922 operations in the ultraclean-air groups groups (0.6%) (ratio 2.6, 95% confidence limits 1.6-4.2; p < 0.001). Fitzgerald et al11 found the incidence of deep sepsis after af ter 5,865 total hip arthroplasties performed in four centres varied from 0.5% to 2.3%.Procedures performed in a conventional operating operating room were associated with the highest incidence of deep sepsis (1.3%). The use of a vertical, unidirectional airflow system with a helmet aspirator suite was associated with the lowest incidence of deep sepsis (0.6%).

120A. How would would you assess assess that your your laminar airflow airflow operation theatre is suitable for performing joint replacements? I would ensure that the operating theatre complies with the Health Technical Technical 12 Memorandum 2025 (HTM)  with respect to ventilation in healthcare premises.

120B. What are the microbiological requirements for continuing to perform joint replacements in your operation theatre? When commissioning an ultraclean theatre by microbiological microbiological sampling 13 sampling points reproduced one at each corner of the unidirectional airflow zone perimeter, perimeter, halfway along each side of the perimeter, one at each corner of the inner zone and one in the centre. The air should contain less than 0.5 CFU/ m3, or one colony for every 2 m3 sampled (HTM 2025).12

120C. What antibiotics would you you administer as prophylaxis prior to performing a lower limb joint replacement and why? I would administer third-generation cephalosporins intravenously as they are effective against staphylococcal and uropathogens. For revision cases, specific antibiotics may been given in cases of confirmed infection, vancomycin and teicoplanin are often used in cases of presumed aseptic revision surgery.13

120D. What is the mechanism of action of flucloxacillin and vancomycin?

161

Like other β-lactam antibiotics, flucloxacillin acts by inhibiting the synthesis of bacterial cell walls. It inhibits cross-linkage between the linear peptidoglycan polymer chains that make up a major component of the cell wall of Gram-positive bacteria. Vancomycin acts by inhibiting Vancomycin i nhibiting proper cell wall synthesis in Gram-positive bacteria. Due to the different mechanism by which Gram-negative bacteria

 

162

Chapter 12 Viva Answers

produce their cell walls and the various factors related to entering the outer membrane of Gram-negative organisms, vancomycin is not active against Gramnegative bacteria (except some non-gonococcal species of Neisseria).

120E. Do pre-operative pre-operative prophylactic prophylactic antibiotics antibiotics reduce the risk of infection following a joint replacement? Yes.

120F.. What is the evidence for your answer? 120F Fogelberg et al,14 Pavel et al,15 Boxma et al16 and Gillespie and Walenkamp17  have all demonstrated that prophylactic antibiotics reduce the incidence of infection after orthopaedic surgery in patients without known infection and their use is considered routine for primary total joint arthroplasty ar throplasty..

121A. Describe the radiogr radiographs aphs  There is sclerosis and lysis in the proximal proximal radius and periosteal scalloping, scalloping, consistent with osteomyelitis.

121B. What is the differential diagnosis?  There is a broad differential differential for these lytic lesions. lesions. Osteomyelitis, tumour tumour and avascular necrosis must all be considered.

121C. What other investigations would would you arrange? Blood tests including WCC, CRP, ESR and blood cultures. MRI scan for bone marrow and soft-tissue changes. Bone scan including WCC labelled scan would be arranged as well.

121D. How would you manage this patient? History and examination and appropriate antibiotics if blood cultures yield an organism. Aspiration of the bone may be attempted in the area of maximal tenderness and swelling. The sample is sent to the laboratory for Gram stain, culture and sensitivities. Surgery and antibiotics are complementary and surgery may be required in cases refractory to antibiotic treatment. Sequestered abscesses need surgical drainage.

121E. How would you classify osteomyelitis?

Chronic osteomyelitis classified by Cierny and Mader: 18









Medullary (endosteal disease). Superficial (cortical surface sur face infected due to coverage defect). Localised (cortical sequestrum that can be excised without compromising stability). Diffuse (Features of I, II and III with instability before or after debridement).

 

Basic science







 This is further classified by assessing the physiologic status of the host: Class A denotes a normal host. Class B denotes a host with systemic compromise, local compromise, or both. Class C denotes a host for whom the morbidity morbidit y of treatment is worse than that imposed by the disease itself.

121F.. Describe the gram-staining technique for identificati 121F identification on of bacteria and how do you differentiate between gram-positive and gram-negative bacteria on gram staining? Gram-positive bacteria have a thick mesh-like cell wall made of peptidoglycan (50% to 90% of cell envelope), which are stained purple by crystal violet, whereas Gram-negative bacteria have a thinner layer (10% of cell envelope), which are stained pink by the counter-stain. There are four basic steps of the Gram stain: 1. Applying a primary stain (crystal violet) to a heat-fixed heat-fixed smear of a bacterial culture. Heat fixing kills some bacteria but is mostly used to affix the bacteria to the slide so that they don’t rinse out during the staining procedure. 2. The addition of a mordant, which binds to crystal violet and traps it in the cell (Gram’s iodine). 3. Rapid decolorisation with alcohol. 4. Counterstaining with safranin. Carbol fuchsin is sometimes substituted for safranin since it will stain the anaerobic bacteria more intensely but it is much less commonly employed as a counterstain. Crystal violet dissociates in aqueous solutions into CV + and chloride (Cl−) ions.  These ions penetrate through through the cell wall and cell membrane membrane of both Grampositive and Gram-negative cells. The CV+ ion interacts with negatively charged components of bacterial cells and stains stain s the cells purple.

122A. Describe the clinical clinical photograp photograph. h.  There is a sinus in the central portion of midline scar over over the knee. There There appears to be a significant joint effusion.

163

122B. What is the differential diagnosis? Infection, either superficial super ficial or deep. Chronic discharging sinus.

122C. What investigations would you like to arrange? Blood tests – WCC, CRP and ESR. Microbiological Mi crobiological assessment of discharged fluid. Plain radiographs to assess components for loosening.

122D. How would you manage this patient? I would have a high clinical suspicion for infection. I would like to aspirate fluid and undertake a synovial biopsy from the knee to confirm the suspicion. This

 

164

Chapter 12 Viva Answers

should be done under sterile conditions in the operating theatre. Combined aspiration and biopsy confirms infection in 90% of cases. Ideal treatment then involves a two-stage surgical procedure with extensive debridement and washout, followed by antibiotics. An articulating/non-articulating cement spacer (with antibiotics) is used prior to implantation of the revision prosthesis.  The timing of this is guided by inflammatory markers. The use of a single-stage revision is gaining popularity has been advocated in certain patients where the causative organism is known, no sinuses are present, the patient is not immunocompromised, and there is no radiological evidence of component loosening or osteitis.19

122E. What do you understand by the term biofilm? Biofilm is a complex aggregation of microorganisms microorganisms in which cells adhere to each other on a solid substrate.

122F.. Describe the constituents of a biofilm and the stages in its 122F formation. Constituents of biofilm include the complex association of microorganisms and a matrix of extracellular polymeric substance (EPS) which is produced by the microorganisms themselves. This EPS is a polymeric conglomeration of extracellular DNA, proteins and polysaccharides. There are five stages in biofilm formation; 1. Initial attachment 2. Irreversible attachment 3. Initial maturation 4. Further maturation 5. Dispersion

122G. Why are antibiotics ineffective in the presence of a biofilm? Biofilm cannot be treated by antibiotics as the dense extracellular matrix and

View more...

Comments

Copyright ©2017 KUPDF Inc.
SUPPORT KUPDF